Combo with "Nclex Review: Hypertension" and 12 others

Réussis tes devoirs et examens dès maintenant avec Quizwiz!

To asses for JVD in a client with congestive heart failure, the nurse will:

observe the vertical distension of the vein as the patient gradually sits up.

Which of the following may cause sinus bradycardia?

Intrinsic sinus node disease

The position of comfort for a patient with left ventricular failure is usually:

Sitting with legs dependent

Thrombolytic therapy for MI

Streptokinase - TPA - may be prescribed within first 6 hours

"It is important to try to stop smoking."

When developing a teaching plan for a patient newly diagnosed with PAD, which information should the nurse include?

The nurse is monitoring a patient with possible coronary artery disease who is undergoing exercise (stress) testing on a treadmill. The symptom that has the most immediate implications for the patient's care during the exercise testing is a. the BP rising from 134/68 to 150/80 mm Hg. b. the heart rate (HR) increasing from 80 to 96 beats/min. c. the patient complaining of feeling short of breath. d. the ECG indicating the presence of coronary ischemia.

Correct Answer: D Rationale: ECG changes associated with coronary ischemia (such as T-wave inversions and ST segment depression) indicate that the myocardium is not getting adequate oxygen delivery and that the exercise test should be immediately terminated. Increases in BP and HR are normal responses to aerobic exercise. Shortness of breath is also normal as the intensity of exercise increases during the stress testing. Cognitive Level: Application Text Reference: pp. 753, 757 Nursing Process: Assessment NCLEX: Physiological Integrity

The nurse is planning patient teaching for a patient who has just been diagnosed with hypertension and has a new prescription for captopril (Capoten). Which information is important to include when teaching the patient? a. To increase fluid intake if dryness of the mouth is a problem b. To check BP daily before taking the medication c. To include high-potassium foods such as citrus fruits in the diet d. To change position slowly to help prevent dizziness and falls

Correct Answer: D Rationale: The ACE inhibitors frequently cause orthostatic hypotension, and patients should be taught to change position slowly to allow the vascular system time to compensate for the position change. Increasing fluid intake may counteract the effect of the medication, and the patient is taught to use gum or hard candy to relieve dry mouth. The BP does not need to be checked at home by the patient before taking the medication. Because ACE inhibitors cause potassium retention, increased intake of high-potassium foods is inappropriate.

A patient with no history of health problems and a BP of 210/142 is admitted to the ICU with a diagnosis of hypertensive crisis. The clinical manifestation that will require the most immediate action by the nurse is that a. the patient complains of a severe headache with pain at level 9/10 (0-10 scale). b. tremors are present in the fingers when the arms are extended. c. the urine output is 90 ml over the first 2 hours after the patient is admitted. d. the patient is unable to move the left leg when asked to do so.

Correct Answer: D Rationale: The patient's inability to more the left leg indicates that a hemorrhagic stroke may be occurring and will require immediate action to prevent further neurologic damage. The other clinical manifestations also are likely caused by the hypertension and will require rapid nursing actions, but they do not require action as urgently as the neurologic changes. Cognitive Level: Analysis Text Reference: pp. 767, 778 Nursing Process: Assessment NCLEX: Physiological Integrity

20. A patient with a DVT is started on IV heparin and oral warfarin (Coumadin). The patient asks the nurse why two medications are necessary. The nurse's best response to the patient is, a. "Heparin will start to dissolve the clot, and Coumadin will prevent any more clots from occurring." b. "Because of the potential for a pulmonary embolism, it is important for you to have more than one anticoagulant." c. "The heparin will work immediately, but the Coumadin takes several days to have an effect on coagulation." d. "Administration of two anticoagulants reduces the risk for recurrent deep vein thrombosis."

C Rationale: IV heparin is used because of the immediate effect on coagulation and discontinued once the international normalized ratio (INR) value indicates that the warfarin has reached a therapeutic level. Heparin is not a thrombolytic drug. The use of two anticoagulants is not related to the risk for pulmonary embolism, and two are not necessary to reduce the risk for another DVT. Cognitive Level: Application Text Reference: pp. 912, 913 Nursing Process: Implementation NCLEX: Physiological Integrity

4. The nurse is caring for a patient receiving IV furosemide (Lasix) 40 mg and enalapril (Vasotec) 5 mg PO bid for ADHF with severe orthopnea. When evaluating the patient response to the medications, the best indicator that the treatment has been effective is a. weight loss of 2 pounds overnight. b. improvement in hourly urinary output. c. reduction in systolic BP. d. decreased dyspnea with the head of the bed at 30 degrees.

D Rationale: Because the patient's major clinical manifestation of ADHF is orthopnea (caused by the presence of fluid in the alveoli), the best indicator that the medications are effective is a decrease in crackles. The other assessment data also may indicate that diuresis or improvement in cardiac output have occurred but are not as useful in evaluating this patient's response. Cognitive Level: Application Text Reference: p. 825 Nursing Process: Evaluation NCLEX: Physiological Integrity

18. A 21-year-old woman is scheduled for an open mitral valve commissurotomy for treatment of mitral stenosis. When explaining the advantage of valve repair instead of valve replacement to the patient, the nurse will include the information that a. mechanical mitral valves require replacement about every 10 years. b. no antibiotic prophylaxis to prevent endocarditis is needed after valve repair. c. biologic replacement valves require the use of life-long immunosuppressive drugs. d. long-term anticoagulation is necessary after mechanical valve replacement.

D Rationale: Long-term anticoagulation therapy is needed after mechanical valve replacement, and this would restrict decisions about career and childbearing in this patient. Mechanical valves are durable and typically last longer than 10 years. Patients require prophylactic antibiotics when having invasive procedures after either valve repair or replacement. Biologic valves do not activate the immune system, and immunosuppressive therapy is not needed. Cognitive Level: Application Text Reference: p. 882 Nursing Process: Implementation NCLEX: Physiological Integrity

12. When teaching the patient with heart failure about a 2000-mg sodium diet, the nurse explains that foods to be restricted include a. eggs and other high-cholesterol foods. b. canned and frozen fruits. c. fresh or frozen vegetables. d. milk, yogurt, and other milk products.

D Rationale: Milk and yogurt naturally contain a significant amount of sodium, and intake of these should be limited for patients on a diet that limits sodium to 2000 mg daily. Other milk products, such as processed cheeses, have very high levels of sodium and are not appropriate for a 2000-mg sodium diet. The other foods listed have minimal levels of sodium and can be eaten without restriction. Cognitive Level: Application Text Reference: p. 833 Nursing Process: Implementation NCLEX: Health Promotion and Maintenance

14. The nurse establishes the nursing diagnosis of ineffective therapeutic regimen management related to lack of knowledge concerning long-term management of rheumatic fever when a patient recovering from rheumatic fever says, a. "I will need to have monthly antibiotic injections for at 5 years or longer." b. "I will call the doctor if I develop excessive fatigue or difficulty breathing." c. "I will need to let my dentist know that I have had this rheumatic fever." d. "I will be immune to further episodes of rheumatic fever after this infection."

D Rationale: Patients with a history of rheumatic fever are more susceptible to a second episode. The other patient statements are correct and would not support the nursing diagnosis of ineffective therapeutic regimen management related to lack of knowledge. Cognitive Level: Application Text Reference: p. 878 Nursing Process: Diagnosis NCLEX: Health Promotion and Maintenance

Which action will the scrub nurse use to maintain aseptic technique during surgery? A) a. Use waterproof shoe covers. B) b. Wear personal protective equipment. C) c. Insist that all operating room (OR) staff perform a surgical scrub. D) d. Change gloves after touching the upper arm of the surgeon's gown.

D) d. Change gloves after touching the upper arm of the surgeon's gown.

Which outcome measure will be best for the operating room (OR) nurse manager to use in determining the effectiveness of the physical environment and traffic control measures in the operating room? A) a. Smooth functioning of the OR team B) b. Effective protection of patient privacy C) c. Rapid completion of surgical procedure D) d. Low incidence of perioperative infection

D) d. Low incidence of perioperative infection

2) Which patient should the nurse attend to first? A. Diabetic patient experiencing an increased blood sugar at 8am in the morning. B. Influenza patient experiencing a fever spike of 100.0 F C. Pneumonia patient experiencing productive cough with green sputum. D. Heart failure patient experiencing abnormal (symptomatic) Bradycardia at rest.

D. Heart failure patient experiencing abnormal (symptomatic) Bradycardia at rest.

The most common cause of death following myocardial infarction is:

Fatal dysrhythmia

Which of the following medications would be the MOST acceptable alternative to morphine for analgesia in patients with acute coronary syndrome

Fentanyl

46. When administering aspirin to a patient with an acute coronary syndrome, you should: a. First check to make sure the patient is not overly hypertensive b. Administer half the usual dose if the patient has a history of stroke c. Have him or her chew and swallow 160 to 325 mg of baby aspirin d. Give up to 325 mg of enteric-coated aspirin for the patient to swallow

Have him or her chew and swallow 160 to 325 mg of baby aspirin

4. A loud S3 heart sound, when heard in older adults, often signifies: a. Emphysema b. Valve rupture c. Heart failure d. Pulmonary hypertension

Heart failure

the MOST significant risk of associated with the use of fibrinolytic therapy is:

Hemorrhage

"Heparin will help prevent blood clots from forming in your heart chambers."

Heparin is prescribed for a patient who has dilated cardiomyopathy has been admitted to the hospital with fatigue and orthopnea. Which statement is appropriate for the nurse to use in patient teaching about anticoagulation therapy?

You are treating a patient with blood pressure of 200 over 140. The patient initially complained of headache and nausea. During your 3-hour transport, the patient began to seize and is now unresponsive to any stimulus. You suspect the patient has:

Hypertensive encephalopathy

9. Which of the following drugs is contraindicated in patients with bronchospastic disease? a. Albuterol b. Adrenaline c. Proventil d. Labetalol

Labetalol

Right ventricular failure most often results from:

Left ventricular failure

Which of the following home medicines would indicate that your patient has a strong risk factor for heart disease?

Metformin

Pain management for MI

Morphine - increases oxygen to myocardium - 2-5mg IV push - decreases preload

ability to do daily activities without chest discomfort.

Nadolol (Corgard) is prescribed for a patient with angina. In evaluating the effectiveness of the drug, the nurse will monitor for _________________

Chemo receptors

Nerve endings located in the aortic arch and carotid bodies that are stimulated by hypoxemia and that subsequently transmit impulses to the CNS.

Atropine works by inhibiting

Parasympathetic response

25. A cardiac patient with respiratory distress should be assumed to be experiencing: a. Severe pain and anxiety b. Heart failure with pulmonary edema c. Bronchospasm due to lower airway constriction d. Cor pulmonale due to pulmonary hypertension

Severe pain and anxiety

Conductivity

The ability of the heart muscle fibers to propagate electrical impulses along and across cell membranes.

If the left ventricle loses 25% of its muscle mass due to myocardial infarction:

The heart can still pump effectively

start basic cardiopulmonary resuscitation (CPR).

The nurse hears the cardiac monitor alarm and notes that the patient has a cardiac pattern of undulations of varying contours and amplitude with no measurable ECG pattern. The patient is unconscious with no pulse or respirations. After calling for assistance, the nurse should _______________

While caring for a patient with and acute myocardial infarction, the nurse monitors the patient carefully for the most common complication of:

arrhythmias

A client has been admitted to the hospital with a diagnosis of suspected bacterial endocarditis. The complication the nurse will constantly observe for is: a. Presence of heart murmur b. Systemic emboli c. Fever d. Congestive heart failure

b. Emboli are the major problem; those arising in the right heart chambers will terminate in the lungs and left chamber emboli may travel anywhere in the arteries. Heart murmurs, fever, and night sweats may be present, but do not indicate a problem with emboli. CHF may be a result, but this is not as dangerous an outcome as emboli.

When teaching a patient with coronary artery disease about the disease's relationship to elevated serum lipid levels, the nurse explains that

each stage in the development of atherosclerotic lesions involves the presence and effects of fat substances in the blood

Mitral stenosis can lead to...

eventually increased pressure in left atrium causes dilation and eventual CHF - also causes Afib

When performing CPR on an adult, you would compress the chest to a depth of _____ inches.

1 1/2 to 2

serum potassium level is 3.0 mEq/L after 1 week of therapy.

A home health care patient has recently started taking oral digoxin (Lanoxin) and furosemide (Lasix) for control of heart failure. The patient data that will require the most immediate action by the nurse is if the patient's _________________

acute pain related to myocardial ischemia.

A patient admitted to the coronary care unit (CCU) with an MI and frequent premature ventricular contractions (PVCs) has health care provider orders for continuous amiodarone infusion, IV nitroglycerin infusion, and morphine sulfate 2 mg IV every 10 minutes until there is relief of pain. The patient says, "This is the worst pain I have ever had. Am I going to die?" Based on these data, the nurse identifies a priority nursing diagnosis of ___________________

the patient will be able to receive outpatient IV antibiotic therapy if complications such as heart failure do not develop.

A patient hospitalized with a streptococcal infective endocarditis tells the nurse," I know that I need antibiotics, but I do not want to be hospitalized for very long." The nurse explains that _______________

administer IV morphine sulfate 2 mg.

A patient in the intensive care unit with ADHF complains of severe dyspnea and is anxious, tachypneic, and tachycardic. All these medications have been ordered for the patient. The first action by the nurse will be to _______________

21. The nurse has initiated discharge teaching for a patient who is to be maintained on warfarin (Coumadin) following hospitalization for DVT. The nurse determines that additional teaching is needed when the patient says, a. "I should wear a Medic Alert bracelet to indicate I am on anticoagulant therapy." b. "I should change my diet to include more green, leafy vegetables." c. "I will check with my health care provider before I begin or stop any medication." d. "I will need to have blood tests routinely to monitor the effects of the Coumadin."

B Rationale: Patients taking Coumadin are taught to follow a consistent diet with regard to foods that are high in vitamin K, such as green leafy vegetables. The other patient statements are accurate. Cognitive Level: Application Text Reference: p. 917 Nursing Process: Evaluation NCLEX: Physiological Integrity

Three days after a myocardial infarction (MI), the patient develops chest pain that increases when taking a deep breath and is relieved by leaning forward. Which action should the nurse take next? A) a. Palpate the radial pulses bilaterally. B) b. Assess the feet for peripheral edema. C) c. Auscultate for a pericardial friction rub. D) d. Check the cardiac monitor for dysrhythmias.

C) c. Auscultate for a pericardial friction rub.

. Which diagnostic test will be most useful to the nurse in determining whether a patient admitted with acute shortness of breath has heart failure? A) a. Serum creatine kinase (CK) B) b. Arterial blood gases (ABGs) C) c. B-type natriuretic peptide (BNP) D) d. 12-lead electrocardiogram (ECG)

C) c. B-type natriuretic peptide (BNP)

Which topic is most important for the nurse to discuss preoperatively with a patient who is scheduled for a colon resection? A) a. Care for the surgical incision B) b. Medications used during surgery C) c. Deep breathing and coughing techniques D) d. Oral antibiotic therapy after discharge home

C) c. Deep breathing and coughing techniques

9. Which of these assessment data obtained by the nurse when assessing a patient with acute pericarditis should be reported immediately to the health care provider? a. Blood pressure (BP) of 166/96 b. Jugular vein distension (JVD) to the level of the jaw c. Pulsus paradoxus 8 mm Hg d. Level 6/10 chest pain with deep inspiration

B Rationale: The JVD indicates that the patient may have developed cardiac tamponade and may need rapid intervention to maintain adequate cardiac output. Hypertension would not be associated with complications of pericarditis, and the BP is not high enough to indicate that there is any immediate need to call the health care provider. A pulsus paradoxus of 8 mm Hg is normal. Level 6/10 chest pain should be treated but is not unusual with pericarditis. Cognitive Level: Application Text Reference: p. 872 Nursing Process: Assessment NCLEX: Physiological Integrity

Before the administration of preoperative medications, the nurse is preparing to witness the patient signing the operative consent form when the patient says, "I do not really understand what the doctor said." Which action is best for the nurse to take? A) a. Provide an explanation of the planned surgical procedure. B) b. Notify the surgeon that the informed consent process is not complete. C) c. Administer the prescribed preoperative antibiotics and withhold any ordered sedative medications. D) d. Notify the operating room staff that the surgeon needs to give a more complete explanation of the procedure.

B) b. Notify the surgeon that the informed consent process is not complete.

When the nurse caring for a patient before surgery has a question about a sedative medication to be given before sending the patient to the surgical suite, the nurse will communicate with the A) a. surgeon. B) b. anesthesiologist. C) c. circulating nurse. D) d. registered nurse first assistant (RNFA).

B) b. anesthesiologist.

A 42-year-old patient is recovering from anesthesia in the postanesthesia care unit (PACU). On admission to the PACU, the blood pressure (BP) is 124/70. Thirty minutes after admission, the blood pressure falls to 112/60, with a pulse of 72 and warm, dry skin. The most appropriate action by the nurse at this time is to A) a. increase the rate of the IV fluid replacement. B) b. continue to take vital signs every 15 minutes. C) c. administer oxygen therapy at 100% per mask. D) d. notify the anesthesia care provider (ACP) immediately.

B) b. continue to take vital signs every 15 minutes.

After delivering five shocks, an implantable cardioverter-defibrillator will:

Continue to deliver shocks at the same joule setting

Prinzmetal angina occurs when:

Coronary arteries spasm

Most myocardial infarctions are caused by:

Coronary embolism

A new patient is seen at an outpatient clinic for a routine health examination. To determine the patient's baseline blood pressure (BP) accurately, the nurse will a. have the patient sit with the arm supported at the level of the heart and measure the BP in each arm. b. obtain the BP readings in both arms and average the results. c. measure the BP in both the supine and upright positions. d. take additional measurements if there is a 10 mm Hg difference between BP readings taken 5 minutes apart.

Correct Answer: A Rationale: To obtain the baseline BP, the patient's arm should be at the level of the heart. The BP is obtained in both arms; if there is a difference, the arm with the higher pressure should be used to monitor BP. The BP can be obtained with the patient in the supine position, but the arm should be positioned at heart level. A 10 mm Hg difference between BP taken 5 minutes apart is within the range of normal and would not require additional measurements. Cognitive Level: Comprehension Text Reference: pp. 778-779 Nursing Process: Assessment NCLEX: Health Promotion and Maintenance

Laboratory testing is ordered for a patient during a clinic visit for routine assessment of hypertension. When monitoring for target organ damage, the nurse will be most concerned about a. blood urea nitrogen (BUN) of 15 mg/dl (5.4 mmol/L). b. serum hemoglobin of 14.7 g/dl (135 g/L). c. serum creatinine of 2.6 mg/dl (230 mmol/L). d. serum potassium of 3.8 mEq/L (3.2 mmol/L).

Correct Answer: C Rationale: BUN and creatinine are useful in determining whether renal failure is developing as a result of hypertension. The BUN level is normal. The serum creatinine is elevated and will require further investigation. The serum potassium level and hemoglobin level are normal. Cognitive Level: Application Text Reference: pp. 767, 768 Nursing Process: Assessment NCLEX: Physiological Integrity

When performing an assessment of a newly admitted patient, the nurse notes a thrill along the left sternal border. To obtain more information about the cause of the thrill, which action will the nurse take next? a. Palpate the quality of the peripheral pulses. b. Compare the apical and radial pulse rates. c. Assess for murmurs. d. Locate the PMI.

Correct Answer: C Rationale: Both thrills and murmurs are caused by turbulent blood flow, such as occurs when blood flows through a damaged valve. Relevant information includes the quality of the murmur, where in the cardiac cycle the murmur is heard, and where on the thorax the murmur is heard best. The other information is also important in the cardiac assessment but will not provide information that is relevant to the thrill. Cognitive Level: Application Text Reference: pp. 748-749, 751 Nursing Process: Assessment NCLEX: Physiological Integrity

To auscultate for S3 or S4 gallops in the mitral area, the nurse listens with the a. diaphragm of the stethoscope with the patient in a reclining position. b. diaphragm of the stethoscope with the patient lying flat on the left side. c. bell of the stethoscope with the patient in the left lateral position. d. bell of the stethoscope with the patient sitting and leaning forward.

Correct Answer: C Rationale: Gallop rhythms generate low-pitched sounds and are most easily heard with the bell of the stethoscope. Sounds associated with the mitral valve are accentuated by turning the patient to the left side, which brings the heart closer to the chest wall. The diaphragm of the stethoscope is best to use for the higher-pitched sounds such as S1 and S2. Cognitive Level: Application Text Reference: pp. 750-751 Nursing Process: Assessment NCLEX: Health Promotion and Maintenance

. When administering IV nitroglycerin (Tridil) to a patient with a myocardial infarction (MI), which action will the nurse take to evaluate the effectiveness of the medication? A) a. Check blood pressure. B) b. Monitor apical pulse rate. C) c. Monitor for dysrhythmias. D) d. Ask about chest discomfort.

D Ask about chest discomfort.

If the paddle positions are switched (if the apex paddle is applied to the sternum and the sternum paddle to the apex) during defibrillation:

Defibrillation will occur as usual

14. A patient with orthopnea a. Experiences dyspnea during periods of exertion b. Prefers a semi sitting position to facilitate breathing c. Experiences worsened dyspnea while lying down d. Sleeps in a recliner due to severe right heart failure

Experiences worsened dyspnea while lying down

PR interval.

In analyzing a patient's electrocardiographic (ECG) rhythm strip, the nurse uses the knowledge that the time of the conduction of an impulse through the Purkinje fibers is represented by the __________________

34. In contrast to stable angina, unstable angina: a. Occurs following periods of strenuous exertion b. Often awakens the patient from his or her sleep c. Indicates that myocardial necrosis has occurred d. Is less frequent but is associated with more pain

Indicates that myocardial necrosis has occurred

a systolic BP <90 mm Hg.

Intravenous sodium nitroprusside (Nipride) is ordered for a patient with acute pulmonary edema. During the first hours of administration, the nurse will need to adjust the Nipride rate if the patient develops ____________

29. Myocardial ischemia occurs when the heart muscle: a. Is deprived of oxygen because of a blocked coronary artery b. Undergoes necrosis because of prolonged oxygen deprivation c. Suffers oxygen deprivation secondary to coronary vasodilation d. Experiences a decreased oxygen demand and an increased supply

Is deprived of oxygen because of a blocked coronary artery

Signs of cardiac tamponade include:

Muffled heart tones

The first medication a paramedic should administer to a patient with angina is:

Oxygen

First degree heart block

PR interval > .20 seconds - conduction is delayed at AV node - related to digoxin or vagal stimulation

11. Common complaints in patients experiencing an acute coronary syndrome (ACS) include all of the following EXCEPT: a. Fatigue b. Headache c. Chest pain d. Palpitations

Palpitations

30. Atherosclerosis is a process in which: a. The outer wall of the coronary artery becomes lined with masses of fatty tissue b. Calcium precipitates into the arterial walls, greatly reducing the artery's elasticity c. Plaque infiltrates the arterial wall, decreasing its elasticity and narrowing its lumen d. Plaque ruptures from a distant location and lodges in one of the coronary arteries

Plaque infiltrates the arterial wall, decreasing its elasticity and narrowing its lumen

Signs & Symptoms CAD

Possibly normal findings during asymptomatic periods, chest pain, palpitations, dyspnea, syncope, cough, hemoptysis, excessive fatigue

16. A pulse that alternates in strength from one beat to the next beat is called: a. Pulse deficit b. Pulsus alternans c. Pulsus paradoxus d. Paradoxical pulse

Pulsus alternans

Ischemia caused by unstable angina:

Responds well to treatment with antiplatelet agents

Myocardial Infarction

Results from sustained cardiac ischemia causing cellular death (after 20 min) and necrosis of myocardium - scar tissue replaces necrotic tissue after 6 weeks

The right coronary artery and the left anterior descending artery supply most of the blood to the:

Right atrium and ventricle

Patients usually describe the pain of an aortic dissection as:

Ripping or tearing

The sound heard when the AV valves close during ventricular systole is:

S1

Conduction system of heart

SA node --> AV node --> Bundle of His --> Purkinje fibers

STEMI

ST elevation on EKG - indicates MI

28. Which of the following clinical findings is LEAST suggestive of left side heart failure? a. An S3 gallop b. Sacral edema c. Crackles in the lungs d. Shortness of breath

Sacral edema

22. When reviewing a cardiac patient's medication list, you note that she is taking acebutolol. You recognize that this drug is also called __________, and is classified as a/an: a. Sectral, beta blocker b. Betapace, antiarrhythmic c. Ticlid, antiplatelet agent d. Norvasc, calcium channel blocker

Sectral, beta blocker

blood urea nitrogen (BUN) and creatinine.

Several hours following a surgical repair of an abdominal aortic aneurysm, the patient develops left flank pain and a urinary output of 20 ml/hr for 2 hours. The nurse notifies the health care provider and anticipates orders for a(n) _____________

teaching people to seek medical diagnosis and treatment for streptococcal pharyngitis.

The community health nurse involved in programs to prevent rheumatic fever knows that the most important intervention to decrease the incidence of the disease is ________________

obtaining the blood cultures.

The health care provider writes the following admitting orders for a patient with suspected IE who has fever and chills: ceftriaxone (Rocephin) 1.0 g intravenous piggyback (IVPB) q12hr, acetylsalicylic acid (ASA) for temperature above 102° F (38.9° C), and blood cultures ∗ 2, complete blood cell count (CBC), and electrocardiogram (ECG). When admitting the patient, the nurse gives the highest priority to ____________

Absolute bradycardia means that

The heart rate is less than 60 beats per minute

relief of chest discomfort.

The nurse administers IV nitroglycerin to a patient with an MI. In evaluating the effect of this intervention, the nurse should monitor for _______________

A patient whose ICD fired three times today who is scheduled for a dose of amiodarone (Cordarone)

The nurse has received change-of-shift report about all of these patients on the telemetry unit. Which patient should the nurse see first?

administer IV fluids at a rate to keep the arterial BP within a normal range.

The nurse identifies a nursing diagnosis of risk for altered peripheral tissue perfusion related to bypass graft thrombosis for a patient following an abdominal aneurysm repair. An appropriate intervention to prevent this problem in the immediate postoperative period is to _______________

an oxygen saturation of 88% on room air.

The nurse identifies the collaborative problem of potential complication: pulmonary edema for a patient in ADHF. When assessing the patient, the nurse will be most concerned about ______________

The patient's 29-year-old brother has had a sudden cardiac arrest.

The nurse is taking a health history from a 24-year-old patient with hypertrophic cardiomyopathy (HC); which information obtained by the nurse is most relevant?

The lungs have crackles audible to the midline.

The nurse obtains the following data when caring for a patient who experienced an AMI 2 days previously. Which information is most important to report to the health care provider?

notify the health care provider immediately if nausea or difficulty breathing occurs.

The nurse plans discharge teaching for a patient with chronic heart failure who has prescriptions for digoxin (Lanoxin), hydrochlorothiazide (HydroDIURIL), and a potassium supplement. Appropriate instructions for the patient include _______________

When monitoring a patient's cardiac rhythm it is MOST important to remember that:

The presence of a QRS complex correlates with the patient's pulse

"I will miss being able to eat peanut butter sandwiches."

Which of these statements made by a patient after the nurse has completed teaching about the TLC diet indicates that further teaching is needed?

What criteria should the nurse use to determine normal sinus rhythm for a client on a cardiac monitor? Check all that apply. a. The RR intervals are relatively consistent b. One P wave precedes each QRS complex c. Four to eight complexes occur in a 6-second strip d. The ST segment is higher than the PR interval e. The QRS complex ranges from 0.12 to 0.2 seconds

a and b. (a) The consistency of the RR interval indicates a regular rhythm. (b) A normal P wave before each complex indicates the impulse originated in the SA node. (c) The number of complexes in a 6-second strip is multiplied by 10 to approximate the heart rate; normal sinus rhythm is 60 to 100. (d) Elevation of the ST segment is a sign of cardiac ischemia and is unrelated to the rhythm. (e) The QRS duration should be less than 0.12 seconds; the PR interval should be 0.12 to 0.2 seconds.

A patient is receiving a drug that decreases peripheral arterial resistance. The nurse anticipates that the effect of this drug on the patient's cardiac function will result in

a decrease in afterload

You are caring for a patient on cardiac monitoring. The monitor show ventricular fibrillation. You are aware that this is associated with the absence of:

a palpable pulse respirations QRS complexes all of the above

Three days after a myocardial infarction, the patient develops chest pain that radiates to the back and left arm and is relieved by sitting in a forward position. On auscultation of the patient's chest, the nurse would expect to hear

a pericardial friction rub

Regurgitant valve

a valve that cannot close completely - sometimes called insufficient

Stenotic valve

a valve that cannot open fully

Which of the following instructions should be included in the discharge teaching for a patient discharged with a transdermal nitroglycerin patch? a. "Apply the patch to a nonhairy, nonfatty area of the upper torso or arms." b. "Apply the patch to the same site each day to maintain consistent drug absorption." c. "If you get a headache, remove the patch for 4 hours and then reapply." d. "If you get chest pain, apply a second patch right next to the first patch."

a. A nitroglycerin patch should be applied to a nonhairy, nonfatty area for the best and most consistent absorption rates. Sites should be rotated to prevent skin irritation, and the drug should be continued if headache occurs because tolerance will develop. Sublingual nitroglycerin should be used to treat chest pain.

Which of the following terms describes the force against which the ventricle must expel blood? a. Afterload b. Cardiac output c. Overload d. Preload

a. Afterload refers to the resistance normally maintained by the aortic and pulmonic valves, the condition and tone of the aorta, and the resistance offered by the systemic and pulmonary arterioles. Cardiac output is the amount of blood expelled from the heart per minute. Overload refers to an abundance of circulating volume. Preload is the volume of blood in the ventricle at the end of diastole.

A 60-year-old male client comes into the emergency department with complaints of crushing chest pain that radiates to his shoulder and left arm. The admitting diagnosis is acute myocardial infarction. Immediate admission orders include oxygen by NC at 4L/minute, blood work, chest x-ray, an ECG, and 2mg of morphine given intravenously. The nurse should first: a. Administer the morphine. b. Obtain a 12-lead ECG. c. Obtain the lab work. d. Order the chest x-ray.

a. Although obtaining the ECG, chest x-ray, and blood work are all important, the nurse's priority action would be to relieve the crushing chest pain.

Following a treadmill test and cardiac catheterization, the client is found to have coronary artery disease, which is inoperative. He is referred to the cardiac rehabilitation unit. During his first visit to the unit he says that he doesn't understand why he needs to be there because there is nothing that can be done to make him better. The best nursing response is: a. "Cardiac rehabilitation is not a cure but can help restore you to many of your former activities." b. "Here we teach you to gradually change your lifestyle to accommodate your heart disease." c. "You are probably right but we can gradually increase your activities so that you can live a more active life." d. "Do you feel that you will have to make some changes in your life now?"

a. Such a response does not have false hope to the client but is positive and realistic. The answer tells the client what cardiac rehabilitation is and does not dwell upon his negativity about it.

A client with a myocardial infarction (MI) develops pulmonary crackles and dyspnea. A chest X-ray shows evidence of pulmonary edema. The specific type of MI the client had is most probably: a. anterior. b. posterior. c. lateral. d. inferior.

a. anterior. An anterior MI causes left ventricular dysfunction and can lead to manifestations of heart failure, which include pulmonary crackles and dyspnea. The other types of MI aren't usually associated with heart failure.

A client is in hemorrhagic shock. To determine the effectiveness of fluid replacement therapy, the nurse should monitor the client's: a. blood pressure. b. hemoglobin level. c. temperature. d. heart rate.

a. blood pressure. With adequate fluid replacement, fluid volume in the intravascular space expands, raising the client's blood pressure. The hemoglobin level reflects red blood cell concentration, not overall fluid status. Temperature and heart rate aren't directly related to fluid status.

When the appropriate complex follows the pacing spike, it is said to be a. captured. b. nonsynchronous. c. inhibited. d. triggered.

a. captured.

A client with chronic heart failure is receiving digoxin (Lanoxin), 0.25 mg by mouth (P.O.) daily, and furosemide (Lasix), 20 mg P.O. twice daily. The nurse instructs the client to notify the physician if nausea, vomiting, diarrhea, or abdominal cramps occur because these signs and symptoms may signal digitalis toxicity. Digitalis toxicity also may cause: a. visual disturbances. b. taste and smell alterations. c. dry mouth and urine retention. d. nocturia and sleep disturbances.

a. visual disturbances. Digitalis toxicity may cause visual disturbances (such as flickering light flashes, colored or halo vision, photophobia, blurring, diplopia, and scotomata), central nervous system abnormalities (such as headache, fatigue, lethargy, depression, irritability and, if profound, seizures, delusions, hallucinations, and memory loss), and cardiovascular abnormalities (abnormal heart rate and arrhythmias). Taste and smell alterations aren't associated with digitalis toxicity. Dry mouth and urine retention typically occur with anticholinergic agents, not inotropic agents such as digoxin. Nocturia and sleep disturbances are adverse effects of furosemide — especially if the client takes the second daily dose in the evening, which may cause diuresis at night.

Atherosclerosis

an accumulation of lipid-containing plaque in the arteries

Interventions for Angina

assess pain (1-10), administer O2, vital signs, administer nitrates, obtain 12 lead EKG

Atherosclerosis impedes coronary blood flow by which of the following mechanisms? a. Plaques obstruct the vein b. Plaques obstruct the artery c. Blood clots form outside the vessel wall d. Hardened vessels dilate to allow blood to flow through

b. Arteries, not veins, supply the coronary arteries with oxygen and other nutrients. Atherosclerosis is a direct result of plaque formation in the artery. Hardened vessels can't dilate properly and, therefore, constrict blood flow.

The physician refers the client with unstable angina for a cardiac catherization. The nurse explains to the client that this procedure is being used in this specific case to: a. Open and dilate the blocked coronary arteries b. Assess the extent of arterial blockage c. Bypass obstructed vessels d. Assess the functional adequacy of the valves and heart muscle.

b. Cardiac catherization is done in clients with angina primarily to assess the extent and severity of the coronary artery blockage, A decision about medical management, angioplasty, or coronary artery bypass surgery will be based on the catherization results.

After extensive cardiac bypass surgery, a client returns to the intensive care unit on dobutamine (Dobutrex), 5 mcg/kg/minute I.V. Which classification best describes dobutamine? a. Indirect-acting dual-active agent b. Direct-acting beta-active agent c. Indirect-acting beta-active agent d. Direct-acting alpha-active agent

b. Direct-acting beta-active agent Dobutamine acts directly on beta receptors. Thus, the drug can be described as a direct-acting beta-active agent. Adrenergic agents are classified according to their method of action and the type of receptor they act on. Direct-acting agents act directly on the sympathetically innervated organ or tissue, whereas indirect-acting agents trigger the release of a neurotransmitter, usually norepinephrine. Dual-acting agents combine direct and indirect actions. Adrenergic agents act on alpha, beta, and dopamine receptors.

During CPR for ventricular fibrillation, which drug would an RN most like prepare first? a. Atropine sulfate b. Epinephrine c. Furosemide (Lasix) d. Lidocaine

b. Epinephrine - first line drug used during CPR to stimulate cardiac activity Atropine sulfate is used for bradycardia and first-degree AV blocks. Furosemide is a diuretic. Lidocaine is used for ventricular arrhythmias, specifically PVCs.

A client receives a pacemaker to treat a recurring arrhythmia. When monitoring the cardiac rhythm strip, the nurse observes extra pacemaker spikes that aren't followed by a beat. Which condition should the nurse suspect? a. Failure to pace b. Failure to capture c. Failure to sense d. Asystole

b. Failure to capture Extra pacemaker spikes that aren't followed by a beat may indicate failure to capture, in which the pacemaker fires but the heart doesn't conduct the beat. In failure to pace, the pacemaker doesn't fire when it should, causing hypotension and other signs of low cardiac output, accompanied by bradycardia or a heart rate slower than the pacemaker's preset rate. In failure to sense, the pacemaker can't sense the client's intrinsic heartbeat; on the rhythm strip, spikes may fall on T waves, or they may fall regularly but at points where they shouldn't appear. Asystole is characterized by an absent heart rate or rhythm as reflected by a flat line on the rhythm strip.

A nurse is caring for a client with unstable ventricular tachycardia. The nurse instructs the client to do which of the following, if prescribed, during an episode of ventricular tachycardia? a. Breathe deeply, regularly, and easily. b. Inhale deeply and cough forcefully every 1 to 3 seconds. c. Lie down flat in bed d. Remove any metal jewelry

b. Inhale deeply and cough forcefully every 1 to 3 seconds. Cough cardiopulmonary resuscitation (CPR) sometimes is used in the client with unstable ventricular tachycardia. The nurse tells the client to use cough CPR, if prescribed, by inhaling deeply and coughing forcefully every 1 to 3 seconds. Cough CPR may terminate the dysrhythmia or sustain the cerebral and coronary circulation for a short time until other measures can be implemented.

The nurse is obtaining a history from a new client in the cardiovascular clinic. When investigating for childhood diseases and disorders associated with structural heart disease, the nurse should consider which finding significant? a. Croup b. Rheumatic fever c. Severe staphylococcal infection d. Medullary sponge kidney

b. Rheumatic fever Childhood diseases and disorders associated with structural heart disease include rheumatic fever and severe streptococcal (not staphylococcal) infections. Croup — a severe upper airway inflammation and obstruction that typically strikes children ages 3 months to 3 years — may cause latent complications, such as ear infection and pneumonia. However, it doesn't affect heart structures. Likewise, medullary sponge kidney, characterized by dilation of the renal pyramids and formation of cavities, clefts, and cysts in the renal medulla, eventually may lead to hypertension but doesn't damage heart structures.

When administered a thrombolytic drug to the client experiencing an MI, the nurse explains to him that the purpose of this drug is to: a. Help keep him well hydrated. b. Dissolve clots he may have. c. Prevent kidney failure. d. Treat potential cardiac arrhythmias.

b. Thrombolytic drugs are administered within the first 6 hours after onset of a MI to lyse clots and reduce the extent of myocardial damage.

A nurse notes that a client with sinus rhythm has a premature ventricular contraction that falls on the T wave of the preceding beat. The client's rhythm suddenly changes to one with no P waves or definable QRS complexes. Instead there are coarse wavy lines of varying amplitude. The nurse assesses this rhythm to be: a. Ventricular tachycardia b. Ventricular fibrillation c. Atrial fibrillation d. Asystole

b. Ventricular fibrillation Ventricular fibrillation is characterized by irregular, chaotic undulations of varying amplitudes. Ventricular fibrillation has no measurable rate and no visible P waves or QRS complexes and results from electrical chaos in the ventricles.

A nurse is watching the cardiac monitor and notices that the rhythm suddenly changes. There are no P waves, the QRS complexes are wide, and the ventricular rate is regular but over 100. The nurse determines that the client is experiencing: a. Premature ventricular contractions b. Ventricular tachycardia c. Ventricular fibrillation d. Sinus tachycardia

b. Ventricular tachycardia Ventricular tachycardia is characterized by the absence of P waves, wide QRS complexes (usually greater than 0.14 second), and a rate between 100 and 250 impulses per minute. The rhythm is usually regular.

A client with chest pain receives nitroglycerin on the way to the acute care facility. Based on an electrocardiogram obtained on admission, the physician suspects a myocardial infarction (MI) and prescribes I.V. morphine to relieve continuing pain. A primary goal of nursing care for this client is to recognize life-threatening complications of an MI. The major cause of death after an MI is: a. cardiogenic shock. b. cardiac arrhythmia. c. heart failure. d. pulmonary embolism.

b. cardiac arrhythmia. Cardiac arrhythmias cause roughly 40% to 50% of deaths after MI. Heart failure, in contrast, accounts for 33% and cardiogenic shock for 9% of post-MI deaths. Pulmonary embolism, another potential complication of an MI, is less common.

A client is recovering from surgical repair of a dissecting aortic aneurysm. The nurse should evaluate the client for signs of bleeding or recurring dissection. These signs include: a. hematuria and decreased urine output. b. hypotension and tachycardia. c. increased urine output and bradycardia. d. hypotension and bradycardia.

b. hypotension and tachycardia. When caring for a client recovering from surgical repair of a dissecting aortic aneurysm, the nurse must monitor for hypotension with reflex tachycardia, decreased urine output, and unequal or absent peripheral pulses — all potential signs of bleeding or recurring dissection. Hematuria, increased urine output, and bradycardia aren't signs of bleeding from aneurysm repair or recurring dissection.

Vasoactive drugs, which cause the arteries and veins to dilate thereby shunting much of the intravascular volume to the periphery and causing a reduction in preload and afterload, include agents such as a. furosemide (Lasix) b. sodium nitoprusside (Nipride) c. dopamine (Inotropin) d. norepinephrine (Levophed)

b. sodium nitoprusside (Nipride)

Creatine kinase level

blood test used to diagnose MI - level rises within 6 hours after onset of chest pain and peaks within 18 hours

CK-MB

blood test used to diagnose MI - peak elevation occurs 18 hours after onset of chest pain - level returns to normal 48 - 72 hours later

Fibrinolytic medications are beneficial to certain patients with an acute myocardial infarction because they:

break down the plasmin concentration inside a blood clot

During a cardiac catherization blood samples from the right atrium, right ventricle, and pulmonary artery are analyzed for their oxygen content. Normally: a. All contain less CO2 than does pulmonary vein blood b. All contain more oxygen than does pulmonary vein blood c. The samples of blood all contain about the same amount of oxygen d. Pulmonary artery blood contains more oxygen than the other samples

c. Blood samples from the right atrium, right ventricle, and pulmonary artery would all be about the same with regard to oxygen concentration. Such blood contains slightly less oxygen than does systemic arterial blood.

A client enters the ER complaining of severe chest pain. A myocardial infarction is suspected. A 12 lead ECG appears normal, but the doctor admits the client for further testing until cardiac enzyme studies are returned. All of the following will be included in the nursing care plan. Which activity has the highest priority? a. Monitoring vital signs b. Completing a physical assessment c. Maintaining cardiac monitoring d. Maintaining at least one IV access site

c. Even though initial tests seem to be within normal range, it takes at least 3 hours for the cardiac enzyme studies to register. In the meantime, the client needs to be watched for bradycardia, heart block, ventricular irritability, and other arrhythmias. Other activities can be accomplished around the MI monitoring.

Which of the following blood tests is most indicative of cardiac damage? a. Lactate dehydrogenase b. Complete blood count (CBC) c. Troponin I d. Creatine kinase (CK)

c. Troponin I levels rise rapidly and are detectable within 1 hour of myocardial injury. Troponin levels aren't detectable in people without cardiac injury.

When assessing a client with left-sided heart failure, the nurse expects to note: a. ascites. b. jugular vein distention. c. air hunger. d. pitting edema of the legs.

c. air hunger. With left-sided heart failure, the client typically has air hunger and other signs of pulmonary congestion. Ascites, jugular vein distention, and pitting edema of the legs are signs of right-sided heart failure.

Which complication of cardiac surgery occurs when there is fluid and clot accumulation in the pericardial sac, which compresses the heart, preventing blood from filling the ventricles? a. hypothermia b. hypertension c. cardiac tamponade d. fluid overload

c. cardiac tamponade

Signs and symptoms of angina

chest pain/pressure, pain may radiate, pain relieved by nitroglycerin and rest, dyspnea, pallor/diaphoresis, indigestion, "impending doom"

A client in the emergency department complains of squeezing substernal pain that radiates to the left shoulder and jaw. He also complains of nausea, diaphoresis, and shortness of breath. What should the nurse do? a. Complete the client's registration information, perform an electrocardiogram, gain I.V. access, and take vital signs. b. Alert the cardiac catheterization team, administer oxygen, attach a cardiac monitor, and notify the physician. c. Gain I.V. access, give sublingual nitroglycerin, and alert the cardiac catheterization team. d. Administer oxygen, attach a cardiac monitor, take vital signs, and administer sublingual nitroglycerin.

d. Administer oxygen, attach a cardiac monitor, take vital signs, and administer sublingual nitroglycerin. Cardiac chest pain is caused by myocardial ischemia. Administering supplemental oxygen increases the myocardial oxygen supply. Cardiac monitoring helps detect life-threatening arrhythmias. Ensure that the client isn't hypotensive before giving sublingual nitroglycerin for chest pain. Registration information may be delayed until the client is stabilized. Alerting the cardiac catheterization team before completing the initial assessment is premature.

white male, age 43, with a tentative diagnosis of infective endocarditis is admitted to an acute care facility. His medical history reveals diabetes mellitus, hypertension, and pernicious anemia; he underwent an appendectomy 20 years ago and an aortic valve replacement 2 years ago. Which history finding is a major risk factor for infective endocarditis? a. Race b. Age c. History of diabetes mellitus d. History of aortic valve replacement

d. History of aortic valve replacement A heart valve prosthesis, such as an aortic valve replacement, is a major risk factor for infective endocarditis. Other risk factors include a history of heart disease (especially mitral valve prolapse), chronic debilitating disease, I.V. drug abuse, and immunosuppression. Although race, age, and a history of diabetes mellitus may predispose a person to cardiovascular disease, they aren't major risk factors for infective endocarditis.

A client with chest pain, dyspnea, and an irregular heartbeat comes to the emergency department. An electrocardiogram shows a heart rate of 110 beats/minute (sinus tachycardia) with frequent premature ventricular contractions. Shortly after admission, the client has ventricular tachycardia and becomes unresponsive. After successful resuscitation, the client is taken to the intensive care unit (ICU). Which nursing diagnosis is appropriate at this time? a. Deficient knowledge (disease process) related to interventions used to treat acute illness b. Impaired physical mobility related to complete bed rest c. Social isolation related to restricted visiting hours in the ICU d. Ineffective tissue perfusion (cardiopulmonary) related to arrhythmia

d. Ineffective tissue perfusion (cardiopulmonary) related to arrhythmia The client suffered a lethal arrhythmia, requiring immediate resuscitation. This arrhythmia was caused by ineffective perfusion to the heart. Therefore, the client should have the nursing diagnosis Ineffective tissue perfusion (cardiopulmonary). Client teaching should be limited to clear, concise explanations that reduce anxiety and promote cooperation. An anxious client has difficulty learning, so the knowledge deficit would continue despite attempts at teaching. Impaired physical mobility and Social isolation are necessitated by the client's critical condition; therefore, they are considered therapeutic, not problems warranting nursing diagnoses.

In a client with chronic bronchitis, which sign would lead the nurse to suspect right-sided heart failure? a. Cyanosis of the lips b. Bilateral crackles c. Productive cough d. Leg edema

d. Leg edema Right-sided heart failure is characterized by signs of circulatory congestion, such as leg edema, neck vein distention, and hepatomegaly. Left-sided heart failure is characterized by circumoral cyanosis, crackles, and a productive cough.

A client with a history of I.V. drug abuse is admitted to the medical-surgical unit for evaluation for infective endocarditis. Nursing assessment is most likely to reveal that this client has: a. retrosternal pain that worsens during supine positioning. b. pulsus paradoxus. c. a scratchy pericardial friction rub. d. Osler's nodes and splinter hemorrhages.

d. Osler's nodes and splinter hemorrhages. Infective endocarditis occurs when an infectious agent enters the bloodstream, such as from I.V. drug abuse or during an invasive procedure or dental work. Typical assessment findings in clients with this disease include Osler's nodes (red, painful nodules on the fingers and toes), splinter hemorrhages, fever, diaphoresis, joint pain, weakness, abdominal pain, a new or altered heart murmur, and Janeway's lesions (small, hemorrhagic areas on the fingers, toes, ears, and nose). The other options are common findings in clients with pericarditis, not infective endocarditis.

Calcium channel blockers (CAD/Angina)

dilate coronary arteries and reduce vasospasm

Causes for sinus tachycardia

fever acute blood loss anemia shock exercise CHF pain anxiety drugs

Nitrates and MI

given IV dilates coronary arteries decreases SVR/afterload decreases preload

The MOST immediate forms of reprofusion therapy for an injured myocardium is:

high flow oxygen and infusion of nitroglycerin

A client with diabetes mellitus is admitted unresponsive to the ED. Initial labs reveal: K+ 2.8, Na 138, Cl 90, glucose 628. Cardiac monitoring shows multifocal PVCs. The nurse understands that the PVCs are most likely from:

hypokalemia

ST depression

indicated ventricles are having difficulty returning to resting state

Pericarditis

inflammation of pericardium - inflammation causes layers to rub and can result in accumulation of fluid in sac

Endocarditis

inflammation of the inner lining of the heart and valves - occurs primarily in IV drug users, valve replacements disease, and invasive procedures (portal of entry) - eroded areas in endocardium attract sediment which attracts bacteria - lesions grow and destroy endocardium and valves -eventually leads to obstruction of blood flow through valves

A patient being treated with lisinopril (Zestril, Prinivil) asks the nurse how the drug lowers the blood pressure. The nurse explains that this drug

inhibits the effects of a substance in the blood that causes vasoconstriction and sodium and water retention

Conduct a nursing assessment of the vascular and lymphatic systems.

left atrium

Splinter hemmorhages

longitudinal black lines in nail beds - associated with endocarditis

A patient admitted to the hospital with an exacerbation of her chronic congestive heart failure tells the nurse she was fine when she went to bed but woke up feeling as if she were suffocating. The nurse explains that the onset of these symptoms in the middle of the night is not unusual because

lying down promotes fluid reabsorption from her legs and when it returns to the heart, it is too much for the heart to pump out

patients experiencing a right ventricular infarction

may present with hypotension

Collateral circulation

more than one artery supplying a muscle with blood, develops when chronic ischemia occurs to meet the metabolic demands; therefore an occlusion of a coronary artery in a younger individual is more likely to be lethal than in an older individual

A patient with acute congestive heart failure has severe dyspnea and is extremely anxious. The nurse anticipates that increased cardiac output and decreased anxiety may be promoted by the intravenous administration of

morphine

During the nursing assessment of any patient with a valvular disorder, the nurse would expect to find

murmurs

Coronary Artery Disease (definition)

narrowing or obstruction of one or more coronary arteries as a result of atherosclerosis causing decreased perfusion of myocardial tissue and inadequate myocardial oxygen supply leading to hypertension, angina, dysrhythmias, MI, heart failure, and death

Janeway lesions

nontender hemorrhagic lesions - fingers, toes, nose, earlobes - associated with endocarditis

PR interval

normal is .12 - .20 seconds - represents

pace maker - capture

pacemaker provides adequate stimulus to produce ventricular contraction

Pericarditis - signs and symptoms

pain radiating to neck or back, increased pain on inspiration, increased pain when supine and decrease when leaning forward - friction rub on auscultation near lower left sternal border, elevated WBC and sed rate, fever, + blood culture, STEMI or inverted T wave, echo showing pericardial effusion, narrow pulse pressure

Oselr's nodes

reddish tender lesions on pads of fingers, hands, and toes - associated with endocarditis

Beta blockers (CAD/Angina)

reduces BP - prevents adrenaline from causing tachycardia and vasoconstriction - desired heart rate 50-60 to enhane cardiac output

Heart murmur

reflects turbulent blood flow through valve - valve can be normal or abnormal

The _____ follows depolarization of the myocardium and represents a period during which no new cardiac potential can be propagated.

refractory period

Aortic insuffciency/regurgitation

related to Marfan's syndrome and bacterial endocarditis - backflow of blood from aorta to left ventricle during diastole leads to dilation of ventricle - usually asymptomatic for many years

Mitral insufficiency/regurgitation

related to RHD - backflow of blood into left atrium during systole because valve does not fully close

P wave

represents atrial depolarization/contraction

A patient with chronic congestive heart failure tells the nurse at the clinic that he has gained 5 pounds in the last 3 days, even though he has continued to follow a low-sodium diet. The nurse recognizes the patient

should be assessed for other symptoms that would indicate an exacerbation of congestive heart failure

Digoxin - how does it work?

slows heart rate by increasing block at AV node and increases force of myocardial contraction

Symptoms of CAD occur when...

the coronary artery is occluded to the point that inadequate blood supply to the muscle occurs, causing ischemia

During postoperative teaching with a patient who has had a mitral valve replacement with a mechanical valve, the nurse instructs the patient regarding

the need for anticoagulation therapy for the duration of the valve

Unstable angina

unpredictable - increases in occurrence, duration, and severity over time - may not be relieved with nitroglycerine and may require hospitalization

Causes for sinus bradycardia

vagal stimulation digitalis toxicity (>2.4) Increased ICP MI trained athletes severe pain beta blockers hypothermia

32. Acute coronary syndrome (ACS) is a term used to describe: a. Acute chest pressure or discomfort that subsides with rest or nitroglycerin b. A clinical condition in which patients experience chest pain during exertion c. Any group of clinical symptoms consistent with acute myocardial ischemia d. A sudden cardiac rhythm disturbance that causes a decrease in cardiac output

Any group of clinical symptoms consistent with acute myocardial ischemia

To help reduce impedance to electrical current:

Apply 25 pounds of pressure with the paddles against the chest wall

In preparing a patient for discharge from the hospital following a myocardial infarction, the nurse determines that further instruction is needed when the patient says

"My heart will be as good as new when I finish a cardiac rehabilitation program."

When teaching a client about propranolol hydrochloride, the nurse should base the information on the knowledge that propranolol: 1. Blocks beta-adrenergic stimulation and thus causes decreased heart rate, myocardial contractility, and conduction. 2. Increases norepinephrine secretion and thus decreases blood pressure and heart rate. 3. Is a potent arterial and venous vasodilator that reduces peripheral vascular resistance and lowers blood pressure. 4. Is an angiotensin-converting enzyme inhibitor that reduces blood pressure by blocking the conversion of angiotensin I to angiotensin II.

1. Propranolol is a beta-adrenergic blocking agent. Actions of propranolol include reducing heart rate, decreasing myocardial contractility, and slowing conduction. Propranolol does not increase norepinephrine secretion, cause vasodilation, or block conversion of angiotensin I to angiotensin II.

The client has had hypertension for 20 years. The nurse should assess the client for? 1. Renal insufficiency and failure. 2. Valvular heart disease. 3. Endocarditis. 4. Peptic ulcer disease.

1. Renal disease, including renal insufficiency and failure, is a complication of hypertension. Effective treatment of hypertension assists in preventing this complication. Valvular heart disease, endocarditis, and peptic ulcer disease are not complications of hypertension.

Metoprolol (Toprol XL) is added to the pharmacologic therapy of a diabetic female diagnosed with stage 2 hypertension initially treated with Furosemide (Lasix) and Ramipril (Altace). An expected therapeutic effect is: 1. Decrease in heart rate. 2. Lessening of fatigue. 3. Improvement in blood sugar levels. 4. Increase in urine output.

1. The effect of a beta blocker is a decrease in heart rate, contractility, and afterload, which leads to a decrease in blood pressure. The client at first may have an increase in fatigue when starting the beta blocker. The mechanism of action does not improve blood sugar or urine output.

A client treated for hypertension with furosemide (Lasix), atenolol (Tenormin), and ramipril (Altace) develops a second degree heart block Mobitz type 1. Which of the following actions should the nurse take? 1. Administer a 250 mL fluid bolus. 2. Withhold the atenolol. 3. Prepare for cardioversion. 4. Set up for an arterial line.

2. The client may be asymptomatic and the underlying cause should be assessed. Drugs that block the AV node should be avoided, such as beta blockers (Atenolol), calcium channel blockers, digoxin, and amiodarone. Symptomatic clients are treated with atropine and transcutaneous pacing. There is no indication for a fluid bolus, cardioversion, or arterial line.

The most important long-term goal for a client with hypertension would be to: 1. Learn how to avoid stress. 2. Explore a job change or early retirement. 3. Make a commitment to long-term therapy. 4. Lose weight.

3. Compliance is the most critical element of hypertension therapy. In most cases, hypertensive clients require lifelong treatment and their hypertension cannot be managed successfully without drug therapy. Stress management is an important component of hypertension therapy, but the priority goal is related to compliance. It is not necessary for the client to change jobs or retire, but rather to learn to manage stress if the job is stressful. Losing weight may be necessary and will contribute to lower blood pressure, but the client must first accept the need for a lifelong management plan to control the hypertension.

The nurse teaches a client, who has recently been diagnosed with hypertension, about dietary restrictions: a low-calorie, low-fat, low-sodium diet. Which of the following menu selections would best meet the client's needs? 1. Mixed green salad with blue cheese dressing, crackers, and cold cuts. 2. Ham sandwich on rye bread and an orange. 3. Baked chicken, an apple, and a slice of white bread. 4. Hot dogs, baked beans, and celery and carrot sticks.

3. Processed and cured meat products, such as cold cuts, ham, and hot dogs, are all high in both fat and sodium and should be avoided on a low-calorie, low-fat, low-salt diet. Dietary restrictions of all types are complex and difficult to implement with clients who are basically asymptomatic.

A client's job involves working in a warm, dry room, frequently bending and crouching to check the underside of a high-speed press, and wearing eye guards. Given this information, the nurse should assess the client for which of the following? 1. Muscle aches. 2. Thirst. 3. Lethargy. 4. Orthostatic hypotension.

4. Possible dizziness from orthostatic hypotension when rising from a crouched or bent position increases the client's risk of being injured by the equipment. The nurse should assess the client's blood pressure in all three positions (lying, sitting, and standing) at all routine visits. The client may experience muscle aches, or thirst from working in a warm, dry room, but these are not as potentially dangerous as orthostatic hypotension. The client should not be experiencing lethargy.

An exercise program is prescribed for the client with hypertension. Which intervention would be most likely to assist the client in maintaining an exercise program? 1. Giving the client a written exercise program. 2. Explaining the exercise program to the client's spouse. 3. Reassuring the client that he or she can do the exercise program. 4. Tailoring a program to the client's needs and abilities.

4. Tailoring or individualizing a program to the client's lifestyle has been shown to be an effective strategy for changing health behaviors. Providing a written program, explaining the program to the client's spouse, and reassuring the client that he or she can do the program may be helpful but are not as likely to promote adherence as individualizing the program.

A 68 year old female is receiving digoxin (Lanoxin). The nurse should notify the practitioner and withhold the medication if the apical pulse is less than which of the following?

60

15. A 36-year-old patient who has a history of thromboangiitis obliterans (Buerger's disease) is admitted to the hospital with a gangrenous lesion of the right small toe. When the nurse is planning expected outcomes for the patient, which outcome has the highest priority for this patient? a. Cessation of smoking b. Maintenance of appropriate weight c. Control of serum lipid levels d. Demonstration of meticulous foot care

A Rationale: Absolute cessation of nicotine use is needed to reduce the risk for amputation in patients with Buerger's disease. Other therapies have limited success in treatment of this disease. Cognitive Level: Application Text Reference: p. 908 Nursing Process: Planning NCLEX: Physiological Integrity

28. Heparin is prescribed for a patient who has dilated cardiomyopathy has been admitted to the hospital with fatigue and orthopnea. Which statement is appropriate for the nurse to use in patient teaching about anticoagulation therapy? a. "Heparin will help prevent blood clots from forming in your heart chambers." b. "Heparin is used to improve the circulation to the muscles in your arms and legs." c. "Heparin has been prescribed to stop blood clots from traveling to your lungs." d. "Heparin makes it easier for your heart to pump and will decrease your symptoms."

A Rationale: Decreased blood flow through the heart causes blood stasis and the formation of blood clots in the ventricles, which then may embolize. Anticoagulant therapy will not improve circulation to the skeletal muscles. The patient with dilated cardiomyopathy who is inactive may be at risk for deep-vein thrombosis and pulmonary emboli, but this is not the usual reason for anticoagulation. There is no indication in the stem that the patient is immobile. Heparin will not decrease cardiac workload or decrease the patient's fatigue or orthopnea. Cognitive Level: Application Text Reference: p. 886 Nursing Process: Implementation NCLEX: Physiological Integrity

19. The nurse has identified the collaborative problem of potential complication: pulmonary embolism for a patient with left-calf DVT. Which nursing action is appropriate to include in the plan of care? a. Maintain bed rest as ordered. b. Administer oxygen to keep O2 saturation >90%. c. Apply compression gradient stockings. d. Remind the patient to dorsiflex the feet and rotate the ankles.

A Rationale: Decreasing muscle activity of the leg will help prevent thrombus dislodgement. There is no need to administer oxygen unless the patient develops a pulmonary embolism. Compression gradient stockings are ordered after resolution of the DVT to prevent further DVT. Exercising the muscles in the legs may prevent a new DVT, but it may dislodge the current thrombus if the patient has a DVT. Cognitive Level: Application Text Reference: p. 912 Nursing Process: Planning NCLEX: Physiological Integrity

18. The health care provider orders a continuous IV heparin infusion for a patient with swelling and pain of the upper leg caused by a DVT. While the patient is receiving the heparin infusion, the nurse should a. avoid any IM medications to prevent localized bleeding. b. notify the health care provider if the partial thromboplastin time (PTT) value is greater than 50 seconds. c. have vitamin K available in case reversal of the heparin is needed. d. monitor posterior tibial and dorsalis pedis pulses with the Doppler.

A Rationale: IM injections are avoided in patients receiving anticoagulation. A PTT of 50 seconds is within the therapeutic range. Vitamin K is used to reverse warfarin. Pulse quality is not affected by DVT. Cognitive Level: Application Text Reference: pp. 913, 916 Nursing Process: Planning NCLEX: Physiological Integrity

6. After repair of an abdominal aortic aneurysm, the nurse notes that the patient does not have popliteal, posterior tibial, or dorsalis pedis pulses. The legs are cool and mottled. Which action is appropriate for the nurse to take first? a. Review the preoperative assessment form for data about the pulses. b. Notify the surgeon and anesthesiologist. c. Document that the pulses are absent and recheck in 30 minutes. d. Elevate the lower extremities on pillows.

A Rationale: Many patients with aortic aneurysms also have peripheral arterial disease, so the nurse should check the preoperative assessment to determine whether pulses were present before surgery before notifying the health care providers about the absent pulses. Because the patient's symptoms suggest graft occlusion or multiple emboli and a possible need to return to surgery, it is not appropriate to wait 30 minutes before taking action. Elevating the legs will decrease blood flow. Cognitive Level: Application Text Reference: p. 898 Nursing Process: Implementation NCLEX: Physiological Integrity

6. A patient in the intensive care unit with ADHF complains of severe dyspnea and is anxious, tachypneic, and tachycardic. All these medications have been ordered for the patient. The first action by the nurse will be to a. administer IV morphine sulfate 2 mg. b. give IV diazepam (Valium) 2.5 mg. c. increase dopamine (Intropin) infusion by 2 mcg/kg/min. d. increase nitroglycerin (Tridil) infusion by 5 mcg/min.

A Rationale: Morphine improves alveolar gas exchange, improves cardiac output by reducing ventricular preload and afterload, decreases anxiety, and assists in reducing the subjective feeling of dyspnea. Diazepam may decrease patient anxiety, but it will not improve the cardiac output or gas exchange. Increasing the dopamine may improve cardiac output but will also increase the heart rate and myocardial oxygen consumption. Nitroglycerin will improve cardiac output and may be appropriate for this patient, but it will not directly reduce anxiety and will not act as quickly as morphine to decrease dyspnea. Cognitive Level: Analysis Text Reference: pp. 828-829 Nursing Process: Implementation NCLEX: Physiological Integrity

21. A patient with ADHF who is receiving nesiritide (Natrecor) asks the nurse how the medication will work to help improve the symptoms of dyspnea and orthopnea. The nurse's reply will be based on the information that nesiritide will a. dilate arterial and venous blood vessels, decreasing ventricular preload and afterload. b. improve the ability of the ventricular myocardium to contract, strengthening contractility. c. enhance the speed of impulse conduction through the heart, increasing the heart rate. d. increase calcium sensitivity in vascular smooth muscle, boosting systemic vascular resistance.

A Rationale: Nesiritide, a recombinant form of BNP, causes both arterial and venous vasodilation, leading to reductions in preload and afterload. Inotropic medications, such as dopamine and dobutamine, may be used in ADHF to improve ventricular contractility. Nesiritide does not increase impulse conduction or calcium sensitivity in the heart. Cognitive Level: Application Text Reference: p. 829 Nursing Process: Implementation NCLEX: Physiological Integrity

8. To assess the patient with pericarditis for the presence of a pericardial friction rub, the nurse should a. place the diaphragm of the stethoscope at the lower left sternal border of the chest. b. ask the patient to stop breathing during auscultation to distinguish the sound from a pleural friction rub. c. use the diaphragm of the stethoscope to listen for a rumbling, low-pitched, systolic sound. d. feel the precordial area with the palm of the hand to detect vibration with cardiac contraction.

A Rationale: Pericardial friction rubs are heard best with the diaphragm at the lower left sternal border. Because dyspnea is one clinical manifestation of pericarditis, the nurse should time the friction rub with the pulse rather than ask the patient to stop breathing during auscultation. Friction rubs are not typically low pitched or rumbling and are not confined to systole. Rubs are not assessed by palpation. Cognitive Level: Comprehension Text Reference: p. 872 Nursing Process: Assessment NCLEX: Physiological Integrity

7. A patient is admitted to the hospital with possible acute pericarditis. The nurse will plan to teach the patient about the purpose of a. multiple ECGs. b. daily blood cultures. c. cardiac catheterization. d. pericardiocentesis.

A Rationale: Pericarditis causes changes such as ST segment elevation in multiple leads on the ECG, which evolve over the course of the inflammatory process. Blood cultures are not indicated unless the patient has evidence of sepsis. Cardiac catheterization is not a diagnostic procedure for pericarditis. Pericardiocentesis will not be done unless the patient has symptoms of cardiac tamponade. Cognitive Level: Application Text Reference: p. 872 Nursing Process: Planning NCLEX: Physiological Integrity

20. While caring for a patient with aortic stenosis, the nurse establishes a nursing diagnosis of pain related to decreased coronary blood flow. An appropriate intervention by the nurse is to a. promote rest to decrease myocardial oxygen demand. b. teach the patient to use sublingual nitroglycerin for chest pain. c. educate the patient about the need for anticoagulant therapy. d. elevate the head of the bed 40 degrees to decrease venous return.

A Rationale: Rest is recommended to balance myocardial oxygen supply and demand and to decrease chest pain. The patient with aortic stenosis requires higher preload to maintain cardiac output, so nitroglycerin and measures to decrease venous return are contraindicated. Anticoagulation is not recommended unless the patient has atrial fibrillation. Cognitive Level: Application Text Reference: p. 880 Nursing Process: Implementation NCLEX: Physiological Integrity

13. A patient with rheumatic fever has subcutaneous nodules, erythema marginatum, and polyarthritis. An appropriate nursing diagnosis based on these findings is a. activity intolerance related to fatigue and arthralgia. b. risk for infection related to open skin lesions. c. risk for impaired skin integrity related to pruritus. d. impaired physical mobility related to permanent joint fixation.

A Rationale: The clinical manifestations of rheumatic fever include fatigue and arthralgia. The skin lesions seen in rheumatic fever are not open or pruritic. Joint inflammation is a temporary clinical manifestation of rheumatic fever. Cognitive Level: Application Text Reference: p. 877 Nursing Process: Diagnosis NCLEX: Physiological Integrity

15. The community health nurse involved in programs to prevent rheumatic fever knows that the most important intervention to decrease the incidence of the disease is a. teaching people to seek medical diagnosis and treatment for streptococcal pharyngitis. b. providing prophylactic antibiotics to people with a family history of rheumatic fever. c. immunizing susceptible groups of people with streptococcal vaccine. d. promoting hygienic measures to prevent the transmission of streptococcal infections.

A Rationale: The incidence of rheumatic fever is decreased by treatment of streptococcal infections with antibiotics. Family history is not a risk factor for rheumatic fever. There is no immunization that is effective in decreasing the incidence of rheumatic fever. Hygienic measures may help to decrease the spread of streptococcal infections but is not the primary means of decreasing risk for rheumatic fever. Cognitive Level: Comprehension Text Reference: p. 878 Nursing Process: Planning NCLEX: Health Promotion and Maintenance

19. An outpatient who has developed heart failure after having an acute myocardial infarction has a new prescription for carvedilol (Coreg). After 2 weeks, the patient returns to the clinic. The assessment finding that will be of most concern to the nurse is that the patient a. has BP of 88/42. b. has an apical pulse rate of 56. c. complains of feeling tired. d. has 2+ pedal edema.

A Rationale: The patient's BP indicates that the dose of carvedilol may need to be decreased because the mean arterial pressure is only 57. Bradycardia is a frequent adverse effect of -Adrenergic blockade, but the rate of 56 is not as great a concern as the hypotension. -adrenergic blockade will initially worsen symptoms of heart failure in many patients, and patients should be taught that some increase in symptoms, such as fatigue and edema, is expected during the initiation of therapy with this class of drugs. Cognitive Level: Application Text Reference: p. 832 Nursing Process: Assessment NCLEX: Analysis

1. A patient with a history of a 4-cm abdominal aortic aneurysm is admitted to the emergency department with severe back pain and bilateral flank ecchymoses. The vital signs are blood pressure (BP) 90/58, pulse 138, and respirations 34. The nurse plans interventions for the patient based on the expectation that treatment will include a. immediate surgery. b. a STAT angiogram. c. a paracentesis when vital signs are stabilized with fluid replacement. d. admission to intensive care for observation and diagnostic testing.

A Rationale: The patient's history and clinical manifestations are consistent with rupture into the retroperitoneal space, and the patient will need immediate surgery to have a chance at survival. The other listed treatments will all be too time consuming. Cognitive Level: Application Text Reference: p. 895 Nursing Process: Planning NCLEX: Physiological Integrity

2. The health care provider writes the following admitting orders for a patient with suspected IE who has fever and chills: ceftriaxone (Rocephin) 1.0 g intravenous piggyback (IVPB) q12hr, acetylsalicylic acid (ASA) for temperature above 102° F (38.9° C), and blood cultures 2, complete blood cell count (CBC), and electrocardiogram (ECG). When admitting the patient, the nurse gives the highest priority to a. obtaining the blood cultures. b. initiating the IV antibiotic. c. scheduling the ECG. d. administering the ASA.

A Rationale: Treatment of the IE with antibiotics should be started as quickly as possible, but it is essential to obtain blood cultures before initiating antibiotic therapy to obtain accurate sensitivity results. The ECG and ASA should also be accomplished rapidly, but the blood cultures (and then administration of the antibiotic) have highest priority. Cognitive Level: Application Text Reference: p. 867 Nursing Process: Planning NCLEX: Physiological Integrity

1. A patient with a history of chronic heart failure is admitted to the emergency department with severe dyspnea and a dry, hacking cough. The patient has pitting edema in both ankles, blood pressure (BP) of 170/100, an apical pulse rate of 92, and respirations 28. The most important assessment for the nurse to accomplish next is to a. auscultate the lung sounds. b. assess the orientation. c. check the capillary refill. d. palpate the abdomen.

A Rationale: When caring for a patient with severe dyspnea, the nurse should use the ABCs to guide initial care. This patient's severe dyspnea and cough indicate that acute decompensated heart failure (ADHF) is occurring. ADHF usually manifests as pulmonary edema, which should be detected and treated immediately to prevent ongoing hypoxemia and cardiac/respiratory arrest. The other assessments will provide useful data about the patient's volume status and should also be accomplished rapidly, but detection (and treatment) of fluid-filled alveoli is the priority. Cognitive Level: Application Text Reference: pp. 824-825 Nursing Process: Assessment NCLEX: Physiological Integrity

After the nurse teaches the patient about the use of atenolol (Tenormin) in preventing anginal episodes, which statement by a patient indicates that the teaching has been effective? A) a. "It is important not to suddenly stop taking the atenolol." B) b. "Atenolol will increase the strength of my heart muscle." C) c. "I can expect to feel short of breath when taking atenolol." D) d. "Atenolol will improve the blood flow to my coronary arteries."

A "It is important not to suddenly stop taking the atenolol."

Allow the student to participate on the swim team.

A 19-year-old student has a mandatory ECG before participating on a college swim team and is found to have sinus bradycardia, rate 52. BP is 114/54, and the student denies any health problems. What action by the nurse is appropriate?

Question the patient about current stress level and coffee use.

A 21-year-old college student arrives at the student health center at the end of the quarter complaining, "My heart is skipping beats." The nurse obtains an ECG and notes the presence of occasional PVCs. What action should the nurse take first?

long-term anticoagulation is necessary after mechanical valve replacement.

A 21-year-old woman is scheduled for an open mitral valve commissurotomy for treatment of mitral stenosis. When explaining the advantage of valve repair instead of valve replacement to the patient, the nurse will include the information that _____________

Cessation of smoking

A 36-year-old patient who has a history of thromboangiitis obliterans (Buerger's disease) is admitted to the hospital with a gangrenous lesion of the right small toe. When the nurse is planning expected outcomes for the patient, which outcome has the highest priority for this patient?

the use of antihypertensive medications to lower the risk of further dissection or bleeding.

A 72-year-old patient is hospitalized for an aortic dissection of the abdominal aorta that stabilizes with treatment. The nurse develops a teaching plan for the patient's discharge that includes information about _______________

When reviewing the 12-lead electrocardiograph (ECG) for a healthy 86-year-old patient who is having an annual physical examination, which of the following will be of most concern to the nurse? A) a. The heart rate (HR) is 43 beats/minute. B) b. The PR interval is 0.21 seconds. C) c. There is a right bundle-branch block. D) d. The QRS duration is 0.13 seconds.

A The heart rate (HR) is 43 beats/minute.

To auscultate for S3 or S4 gallops in the mitral area, the nurse listens with the A) a. bell of the stethoscope with the patient in the left lateral position. B) b. bell of the stethoscope with the patient sitting and leaning forward. C) c. diaphragm of the stethoscope with the patient in a reclining position. D) d. diaphragm of the stethoscope with the patient lying flat on the left side.

A bell of the stethoscope with the patient in the left lateral position.

36. Infarctions of the inferior myocardial wall are MOST often caused by: a. Blockage of the left coronary artery b. Acute spasm of the circumflex artery c. Occlusion of the right coronary artery d. A blocked left anterior descending artery

A blocked left anterior descending artery

15. In the context of cardiac compromise, syncope occurs due to: a. An increase in vagal tone b. A drop in cerebral perfusion c. A sudden cardiac dysrhythmia d. An acute increase in heart rate

A drop in cerebral perfusion

Auscultate the heart sounds.

A few days after an acute MI, a patient complains of stabbing chest pain that increases with deep breathing. Which action will the nurse take first?

Allow the use of denial as a coping mechanism until the patient begins asking questions about the MI.

A few days after experiencing an MI, the patient states, "I just had a little chest pain. As soon as I get out of here, I'm going for my vacation as planned." Which nursing intervention is appropriate to include in the nursing care plan?

"I will call for help when I need to get up to the bathroom."

A hospitalized patient with heart failure has a new order for captopril (Capoten) 12.5 mg PO. After administering the first dose and teaching the patient about captopril, which statement by the patient indicates that teaching has been effective?

titrate oxygen to keep O2 saturation greater than 90%.

A nursing action that is indicated for the collaborative problem of potential complication: cardiac dysrhythmia in a patient who has had a repair of a descending thoracic aortic aneurysm is to ______________

put one pillow under the thighs and two pillows under the lower legs.

A patient admitted to the hospital with DVT has health care provider's orders for bed rest with the feet elevated. The best method for the nurse to use in elevating the patient's feet is to _____________

paroxysmal nocturnal dyspnea.

A patient admitted to the hospital with an exacerbation of chronic heart failure tells the nurse, "I felt fine when I went to bed, but I woke up in the middle of the night feeling like I was suffocating!" The nurse can best document this assessment information as _______________

Obtain and apply the transcutaneous pacemaker (TCP).

A patient develops sinus bradycardia at a rate of 32 beats/min, has a BP of 80/36 mm Hg, and is complaining of feeling faint. Which action should the nurse take?

third-degree AV block.

A patient experiences dizziness and shortness of breath for several days. During cardiac monitoring in the ED, the nurse obtains the following ECG tracing. The nurse interprets this cardiac rhythm as _______________

difficulty swallowing.

A patient has a 5-cm thoracic aortic aneurysm that was discovered during a routine chest x-ray. When obtaining a nursing history from the patient, the nurse will ask the patient about _________

MCL1

A patient has a dysrhythmia that requires careful monitoring of atrial activity. Which lead will be best to use for continuous monitoring?

40-60

A patient has a junctional escape rhythm on the monitor. The nurse would expect the patient to have a pulse rate of ____ beats/min.

document the finding and continue to monitor the patient.

A patient has a normal cardiac rhythm strip except that the PR interval is 0.34 seconds. The appropriate intervention by the nurse is to ________________

"I won't lift the arm on the pacemaker side up very high until I see the doctor."

A patient has received instruction on the management of a new permanent pacemaker before discharge from the hospital. The nurse recognizes that teaching has been effective when the patient tells the nurse, _______________

visualize any coronary artery blockages and dilate any obstructed arteries.

A patient is admitted to the ED after an episode of severe chest pain, and the physician schedules the patient for coronary angiography and possible percutaneous coronary intervention (PCI). The nurse prepares the patient for the procedure by explaining that it is used to ________________________

"I can't get my shoes on at the end of the day."

A patient is admitted to the hospital with a diagnosis of chronic venous insufficiency. Which of these statements by the patient is most consistent with the diagnosis?

multiple ECGs.

A patient is admitted to the hospital with possible acute pericarditis. The nurse will plan to teach the patient about the purpose of ______________

No change in the patient's chest pain

A patient is receiving fibrinolytic therapy 2 hours after developing an AMI. Which assessment information will be of most concern to the nurse?

keep the patient in bed in the supine position.

A patient recovering on a general surgical unit from an aortic valve replacement 1 week ago develops sudden severe pain, pulselessness, pallor, and coolness in the left leg. The nurse should notify the health care provider and _______________

if the ICD fires and the patient loses consciousness, 911 should be called.

A patient who has a history of sudden cardiac death has an ICD inserted. When performing discharge teaching with the patient, it is important for the nurse to instruct the patient and family that ______________

"You had a serious abnormal heart rhythm, which treatment was able to reverse."

A patient who has been successfully resuscitated after developing ventricular fibrillation asks the nurse about what happened. The most appropriate response by the nurse is, _____________

ECG

A patient who has chest pain is admitted to the ED, and all the following diagnostic tests are ordered. Which one will the nurse arrange to be completed first?

Notify the doctor about any symptoms of heart failure such as shortness of breath.

A patient who has developed acute pulmonary edema is hospitalized and diagnosed with dilated cardiomyopathy. Which information will the nurse plan to include when teaching the patient about management of this disorder?

The LPN/LVN has the patient sit in a bedside chair for 90 minutes.

A patient who has had a femoral-popliteal bypass graft to the right leg is being cared for on the surgical unit. Which action by an LPN/LVN caring for the patient requires the RN to intervene?

Oral ibuprofen (Motrin) 800 mg

A patient who has had recent cardiac surgery develops pericarditis and complains of severe chest pain with deep breathing. Which of these ordered PRN medications should the nurse administer?

Troponin levels

A patient who has had severe chest pain for the last 4 hours is admitted with a diagnosis of possible AMI. Which of these ordered laboratory tests should the nurse monitor to help determine whether the patient has had an MI?

Generalized muscle aches and pains

A patient who has recently started taking rosuvastatin (Crestor) and niacin (Nicobid) reports all the following symptoms to the nurse. Which is most important to communicate to the health care provider?

sildenafil (Viagra)

A patient who is being admitted to the emergency department with severe chest pain gives the following list of medications taken at home to the nurse. Which of the medications has the most immediate implications for the patient's care?

Obtain further information about possible causes for the heart rate.

A patient who is complaining of a "racing" heart and nervousness comes to the emergency department. The patient's blood pressure (BP) is 102/68. The nurse places the patient on a cardiac monitor and obtains the following ECG tracing. Which action should the nurse take next?

Monitor the patient's BP every hour.

A patient who is receiving dobutamine (Dobutrex) for the treatment of ADHF has all of the following nursing actions included in the plan of care. Which action will be best for the RN to delegate to an experienced LPN/LVN?

immune disorders.

A patient who is seen in the clinic tells the health care provider about experiencing cold, numb fingers when running during the winter and is diagnosed with Raynaud's phenomenon. The nurse will anticipate teaching the patient about tests for ______________

dilate arterial and venous blood vessels, decreasing ventricular preload and afterload.

A patient with ADHF who is receiving nesiritide (Natrecor) asks the nurse how the medication will work to help improve the symptoms of dyspnea and orthopnea. The nurse's reply will be based on the information that nesiritide will ___________

"Call if you notice that your stools are black or have blood in them."

A patient with PAD has a new prescription for clopidogrel (Plavix). Which information should the nurse include when teaching the patient about this medication?

"What time did your chest pain begin?"

A patient with ST-segment elevation in several ECG leads is admitted to the ED and diagnosed as having an AMI. Which question should the nurse ask to determine whether the patient is a candidate for fibrinolytic therapy?

"The heparin will work immediately, but the Coumadin takes several days to have an effect on coagulation."

A patient with a DVT is started on IV heparin and oral warfarin (Coumadin). The patient asks the nurse why two medications are necessary. The nurse's best response to the patient is, __________

immediate surgery.

A patient with a history of a 4-cm abdominal aortic aneurysm is admitted to the emergency department with severe back pain and bilateral flank ecchymoses. The vital signs are blood pressure (BP) 90/58, pulse 138, and respirations 34. The nurse plans interventions for the patient based on the expectation that treatment will include ____________

auscultate the lung sounds.

A patient with a history of chronic heart failure is admitted to the emergency department with severe dyspnea and a dry, hacking cough. The patient has pitting edema in both ankles, blood pressure (BP) of 170/100, an apical pulse rate of 92, and respirations 28. The most important assessment for the nurse to accomplish next is to ________________

Heparin will prevent the development of clots in the coronary arteries.

A patient with a non-ST segment elevation myocardial infarction (NSTEMI) is receiving heparin. What is the purpose of the heparin?

assess the patient for clinical manifestations of acute heart failure because an exacerbation of the chronic heart failure may be occurring.

A patient with chronic heart failure who has been following a low-sodium diet tells the nurse at the clinic about a 5-pound weight gain in the last 3 days. The nurse's first action will be to _______________

the cardiac monitor shows a heart rate of 45.

A patient with chronic stable angina is being treated with metoprolol (Lopressor). The nurse will suspect that the patient is experiencing a side effect of the metoprolol if __________________

hypokalemia.

A patient with diabetes mellitus is admitted unresponsive to the emergency department (ED). Initial laboratory findings are serum potassium 2.8 mEq/L (2.8 mmol/L), serum sodium 138 mEq/L (138 mmol/L), serum chloride 90 mEq/L (90 mmol/L), and blood glucose 628 mg/dl (34.9 mmol/L). Cardiac monitoring shows multifocal PVCs. The nurse understands that the patient's PVCs are most likely caused by _______________

anticoagulant therapy with warfarin (Coumadin).

A patient with dilated cardiomyopathy has an atrial fibrillation that has been unresponsive to drug therapy for several days. The nurse anticipates that further treatment of the patient will require ____________

Give the patient's other medications 2 hours after the Welchol.

A patient with hyperlipidemia has a new order for the bile-acid sequestrant medication colesevelam (Welchol). Which nursing action is appropriate when giving the medication?

increase in the patient's heart rate.

A patient with myocardial infarction develops symptomatic hypotension. The monitor shows a type 1, second-degree AV block with a heart rate of 30. The nurse administers IV atropine as prescribed. The nurse determines that the drug has been effective on finding a(n) _________________

activity intolerance related to fatigue and arthralgia.

A patient with rheumatic fever has subcutaneous nodules, erythema marginatum, and polyarthritis. An appropriate nursing diagnosis based on these findings is ______________

administer a sedative before the procedure is begun.

A patient with supraventricular tachycardia (SVT) is hemodynamically stable and requires cardioversion. The nurse will plan to _______________

Hold the ordered metoprolol (Lopressor) and call the health care provider.

A patient's sinus rhythm rate is 62. The PR interval is 0.18 seconds at 1:00 AM, 0.20 seconds at 12:30 PM, and 0.23 seconds at 4:00 PM. Which action should the nurse take?

preoperative patient in the holding area asks the nurse, "Will the doctor put me to sleep with a mask over my face?" The most appropriate response by the nurse is, A) a. "A drug will be given to you through your IV line, which will cause you to go to sleep almost immediately." B) b. "Only your surgeon can tell you for sure what method of anesthesia will be used. Should I ask your surgeon?" C) c. "General anesthesia is now given by injecting medication into your veins, so you will not need a mask over your face." D) d. "Masks are not used anymore for anesthesia. A tube will be inserted into your throat to deliver a gas that will put you to sleep."

A) a. "A drug will be given to you through your IV line, which will cause you to go to sleep almost immediately"

A patient with heart failure has a new order for captopril (Capoten) 12.5 mg PO. After administering the first dose and teaching the patient about captopril, which statement by the patient indicates that teaching has been effective? A) a. "I will call for help when I need to get up to use the bathroom." B) b. "I will be sure to take the medication after eating something." C) c. "I will need to include more high-potassium foods in my diet." D) d. "I will expect to feel more short of breath for the next few days."

A) a. "I will call for help when I need to get up to use the bathroom."

During the preoperative interview, a patient scheduled for an elective hysterectomy tells the nurse, "I am afraid that I will die in surgery like my mother did!" Which response by the nurse is most appropriate? A) a. "Tell me more about what happened to your mother." B) b. "You will receive medications to reduce your anxiety." C) c. "You should talk to the doctor again about the surgery." D) d. "Surgical techniques have improved a lot in recent years."

A) a. "Tell me more about what happened to your mother."

A 19-year-old has a mandatory electrocardiogram (ECG) before participating on a college swim team and is found to have sinus bradycardia, rate 52. BP is 114/54, and the student denies any health problems. What action by the nurse is appropriate? A) a. Allow the student to participate on the swim team. B) b. Refer the student to a cardiologist for further assessment. C) c. Obtain more detailed information about the student's health history. D) d. Tell the student to stop swimming immediately if any dyspnea occurs.

A) a. Allow the student to participate on the swim team.

On the day of surgery, the nurse is admitting a patient with a history of cigarette smoking. Which action is most important at this time? A) a. Auscultate for adventitious breath sounds. B) b. Ask whether the patient has smoked recently. C) c. Remind the patient about harmful effects of smoking. D) d. Calculate the cigarette smoking history in pack-years.

A) a. Auscultate for adventitious breath sounds.

A patient who is just waking up after having a general anesthetic is agitated and confused. Which action should the nurse take first? A) a. Check the O2 saturation. B) b. Administer the ordered opioid. C) c. Take the blood pressure and pulse. D) d. Notify the anesthesia care provider.

A) a. Check the O2 saturation.

. A patient's cardiac monitor has a pattern of undulations of varying contours and amplitude with no measurable ECG pattern. The patient is unconscious and pulseless. Which action should the nurse take first? A) a. Defibrillate at 360 joules. B) b. Give O2 per bag-valve-mask. C) c. Give epinephrine (Adrenalin) IV. D) d. Prepare for endotracheal intubation.

A) a. Defibrillate at 360 joules.

Which description best defines the role of the nurse anesthetist as a member of the surgical team? A) a. Functions independently in the administration of anesthetics B) b. Has the same credentials and responsibilities as an anesthesiologist C) c. Is responsible for intraoperative administration of anesthetics ordered by the anesthesiologist D) d. Requires supervision by the anesthesiologist or surgeon while administering anesthesia to a patient

A) a. Functions independently in the administration of anesthetics

Which action should the postanesthesia care unit (PACU) nurse delegate to nursing assistive personnel (NAP) who help with the transfer of a patient to the surgical unit? A) a. Help with the transfer of the patient onto a stretcher. B) b. Give a verbal report to the surgical unit charge nurse. C) c. Document the appearance of the patient's incision in the chart. D) d. Ensure that the receiving nurse understands the postoperative orders.

A) a. Help with the transfer of the patient onto a stretcher.

A patient with a dislocated shoulder is prepared for a closed, manual reduction of the dislocation with monitored anesthesia care (MAC). The nurse anticipates the administration of A) a. IV midazolam (Versed). B) b. inhaled desflurane (Suprane). C) c. epidural lidocaine (Xylocaine). D) d. eutectic mixture of local anesthetics (EMLA).

A) a. IV midazolam (Versed).

After the nurse administers IV atropine to a patient with symptomatic type 1, second-degree atrioventricular (AV) block, which finding indicates that the medication has been effective? A) a. Increase in the patient's heart rate B) b. Decrease in premature contractions C) c. Increase in peripheral pulse volume D) d. Decrease in ventricular ectopic beats

A) a. Increase in the patient's heart rate

Four days after having a myocardial infarction (MI), a patient who is scheduled for discharge asks for assistance with all the daily activities, saying, "I am too nervous to take care of myself." Based on this information, which nursing diagnosis is appropriate? A) a. Ineffective coping related to anxiety B) b. Activity intolerance related to weakness C) c. Denial related to lack of acceptance of the MI D) d. Social isolation related to lack of support system

A) a. Ineffective coping related to anxiety

Which action will the nurse include in the plan of care when caring for a patient admitted with acute decompensated heart failure (ADHF) who is receiving nesiritide (Natrecor)? A) a. Monitor blood pressure frequently. B) b. Encourage patient to ambulate in room. C) c. Titrate nesiritide rate slowly before discontinuing. D) d. Teach patient about safe home use of the medication.

A) a. Monitor blood pressure frequently.

To determine whether there is a delay in impulse conduction through the atria, the nurse will measure the length of the patient's A) a. P wave. B) b. PR interval. C) c. QT interval. D) d. QRS complex.

A) a. P wave.

The clinic nurse reviews the complete blood cell count (CBC) results for a patient who is scheduled for surgery in a few days. The results are white blood cell count (WBC) 10.2 ´ 103/µL; hemoglobin 15 g/dL; hematocrit 45%; platelets 150 ´ 103/µL. Which action should the nurse take? A) a. Send the CBC results to the surgery facility. B) b. Call the surgeon and anesthesiologist immediately. C) c. Ask the patient about any symptoms of a recent infection. D) d. Discuss the possibility of blood transfusion with the patient

A) a. Send the CBC results to the surgery facility.

Which action by a new nurse who is caring for a patient who has just had an implantable cardioverter-defibrillator (ICD) inserted indicates a need for more education about care of patients with ICDs? A) a. The nurse assists the patient to do active range of motion exercises for all extremities. B) b. The nurse assists the patient to fill out the application for obtaining a Medic Alert ID and bracelet. C) c. The nurse gives atenolol (Tenormin) to the patient without consulting first with the health care provider. D) d. The nurse teaches the patient that sexual activity usually can be resumed once the surgical incision is healed.

A) a. The nurse assists the patient to do active range of motion exercises for all extremities.

13. Paroxysmal nocturnal dyspnea (PND) is MOST accurately defined as: a. Dyspnea that is brought on by excessive movement during sleep b. Sitting upright in a chair in order to facilitate effective breathing c. The inability to function at night due to severe difficulty breathing d. Acute shortness of breath that suddenly awakens a person from sleep

Acute shortness of breath that suddenly awakens a person from sleep

If a patient with cardiac tamponade becomes hypotensive in the field, you should:

Administer a fluid bolus

"Sexual activity can be gradually resumed like other activity. A good comparison of energy expenditure is climbing two flights of stairs."

After having an AMI, a 62-year-old patient tells the nurse, "I guess having sex again will be too hard on my heart." The nurse's best response is ______________

The patient exercises indoors during the winter months.

After teaching a patient with newly diagnosed Raynaud's phenomenon about how to manage the condition, which behavior by the patient indicates that the teaching has been effective?

"I will stop what I am doing and sit down before I put the nitroglycerin under my tongue."

After the nurse teaches a patient with chronic stable angina about how to use the prescribed nitrates, which statement by the patient indicates that the teaching has been effective?

"It is important not to suddenly stop taking the atenolol."

After the nurse teaches the patient about the use of atenolol (Tenormin) in preventing anginal episodes, which statement by a patient indicates that the teaching has been effective?

21. A patient with an elevated cholesterol level would MOST likely take: a. Inderal b. Altacor c. Isordil d. Diovan

Altacor

has BP of 88/42.

An outpatient who has developed heart failure after having an acute myocardial infarction has a new prescription for carvedilol (Coreg). After 2 weeks, the patient returns to the clinic. The assessment finding that will be of most concern to the nurse is that the patient ______________

26. Which statement by a patient who is being discharged 5 days after an abdominal aortic aneurysm repair and graft indicates that the discharge teaching has been effective? a. "I will call the doctor if my temperature is higher than 101° F." b. "I will tell my dentist about this surgery the next time I have an appointment." c. "I should not need to take anything but acetaminophen (Tylenol) for my pain." d. "I am eager to get home so that I can pick up my 6-year-old granddaughter."

B Rationale: Prophylactic antibiotics may be ordered to prevent graft infection when the patient has any invasive procedures, including dental procedures. The patient is instructed to call if the temperature is higher than 100° F. After abdominal surgery, patients may need to use opioid pain medications on a PRN basis. Because heavy lifting is avoided for at least 4 to 6 weeks after surgery, the patient should not pick up a 6-year-old child. Cognitive Level: Application Text Reference: p. 898 Nursing Process: Evaluation NCLEX: Physiological Integrity 27. A patient with PAD has a new prescription for clopidogrel (Plavix). Which information should the nurse include when teaching the patient about this medication? a. "Call if you notice that your stools are black or have blood in them." b. "Take the Plavix on an empty stomach as soon as you get up." c. "Change position slowly to avoid dizziness while you are taking Plavix." d. "You should never use aspirin while you are taking the Plavix." Correct Answer: A Rationale: Clopidogrel inhibits platelet function and increases the risk for GI bleeding. It can be taken without regard to food. Orthostatic hypotension is not an expected side effect of the medication. Aspirin may be prescribed concurrently with clopidogrel for some patients. Cognitive Level: Application Text Reference: p. 902 Nursing Process: Implementation NCLEX: Physiological Integrity

7. Intravenous sodium nitroprusside (Nipride) is ordered for a patient with acute pulmonary edema. During the first hours of administration, the nurse will need to adjust the Nipride rate if the patient develops a. a drop in heart rate to 54 beats/min. b. a systolic BP <90 mm Hg. c. any symptoms indicating cyanide toxicity. d. an increased amount of ventricular ectopy.

B Rationale: Sodium nitroprusside is a potent vasodilator, and the major adverse effect is severe hypotension. After 48 hours of continuous use, cyanide toxicity is a possible (though rare) adverse effect. Reflex tachycardia (not bradycardia) is another adverse effect of this medication. Nitroprusside does not cause increased ventricular ectopy. Cognitive Level: Application Text Reference: p. 828 Nursing Process: Evaluation NCLEX: Physiological Integrity

10. The nurse working in the heart failure clinic will know that teaching for a 74-year-old patient with newly diagnosed heart failure has been effective when the patient a. says that the nitroglycerin patch will be used for any chest pain that develops. b. calls when the weight increases from 124 to 130 pounds in a week. c. tells the home care nurse that furosemide (Lasix) is taken daily at bedtime. d. makes an appointment to see the doctor at least once yearly.

B Rationale: Teaching for a patient with heart failure includes information about the need to weigh daily and notify the health care provider about an increase of 3 pounds in 2 days or 5 pounds in a week. Nitroglycerin patches are used primarily to reduce preload (not to prevent chest pain) in patients with heart failure and should be used daily, not on an "as necessary" basis. Diuretics should be taken earlier in the day to avoid nocturia and sleep disturbance. Heart failure is a chronic condition that will require frequent follow-up rather than an annual health care provider examination. Cognitive Level: Application Text Reference: pp. 826, 833-834, 838 Nursing Process: Evaluation NCLEX: Health Promotion and Maintenance

15. While admitting an 80-year-old patient with heart failure to the medical unit, the nurse obtains the information that the patient lives alone and sometimes confuses the "water pill" with the "heart pill." The nurse makes a note that discharge planning for the patient will need to include a. transfer to a dementia care service. b. referral to a home health care agency. c. placement in a long-term-care facility. d. arrangements for around-the-clock care.

B Rationale: The data about the patient suggest that assistance in developing a system for taking medications correctly at home is needed. A home health nurse will assess the patient's home situation and help the patient to develop a method for taking the two medications as directed. There is no evidence that the patient requires services such as dementia care, long-term-care, or around-the-clock home care. Cognitive Level: Application Text Reference: pp. 836-837 Nursing Process: Assessment NCLEX: Health Promotion and Maintenance

23. A patient who has had recent cardiac surgery develops pericarditis and complains of severe chest pain with deep breathing. Which of these ordered PRN medications should the nurse administer? a. Oral acetaminophen (Tylenol) 650 mg b. Oral ibuprofen (Motrin) 800 mg c. IV morphine sulfate 6 mg d. Fentanyl 2 mg IV

B Rationale: The pain associated with pericarditis is caused by inflammation, so nonsteroidal antiinflammatory medications like ibuprofen are most effective. The patient does not need opioid pain medication. Cognitive Level: Application Text Reference: p. 873 Nursing Process: Implementation NCLEX: Physiological Integrity

8. When the nurse is caring for a patient on the first postoperative day after an abdominal aortic aneurysm repair, the information that is most significant when the nurse is assessing for the return of peristalsis is a. absence of abdominal distention. b. passing of flatus with ambulation. c. dark brown nasogastric (NG) tube drainage. d. moderate abdominal tenderness.

B Rationale: The passing of flatus is the best indicator of returning bowel function. Because the patient usually has an NG tube in place, the absence of abdominal distention is not indicative of gastrointestinal (GI) function. The quality of NG tube drainage may indicate the presence of complications such as GI bleeding but does not provide useful information about GI function. Moderate abdominal tenderness is expected after abdominal surgery. Cognitive Level: Application Text Reference: p. 898 Nursing Process: Assessment NCLEX: Physiological Integrity

14. A patient who has had a femoral-popliteal bypass graft to the right leg is being cared for on the surgical unit. Which action by an LPN/LVN caring for the patient requires the RN to intervene? a. The LPN/LVN administers the ordered aspirin 160 mg after breakfast. b. The LPN/LVN has the patient sit in a bedside chair for 90 minutes. c. The LPN/LVN assists the patient to ambulate 40 feet in the hallway. d. The LPN/LVN places the patient in a Fowler's position for meals.

B Rationale: The patient should avoid sitting for long periods because of the increased stress on the suture line caused by leg edema and because of the risk for DVT. The other actions by the LPN/LVN are appropriate. Cognitive Level: Application Text Reference: p. 906 Nursing Process: Implementation NCLEX: Safe and Effective Care Environment

26. A patient who has developed acute pulmonary edema is hospitalized and diagnosed with dilated cardiomyopathy. Which information will the nurse plan to include when teaching the patient about management of this disorder? a. Careful compliance with diet and medications will control the patient's symptoms. b. Notify the doctor about any symptoms of heart failure such as shortness of breath. c. No more than one or two alcoholic drinks daily are permitted. d. Elevating the legs above the heart will help relieve angina.

B Rationale: The patient should be instructed to notify the health care provider about any worsening of heart failure symptoms. Because dilated cardiomyopathy does not respond well to therapy, even patients with good compliance with therapy may have recurrent episodes of heart failure. The patient is instructed to avoid alcoholic beverages. Elevation of the legs above the heart will worsen symptoms (although this approach is appropriate for a patient with hypertrophic cardiomyopathy). Cognitive Level: Application Text Reference: pp. 886, 888 Nursing Process: Planning NCLEX: Physiological Integrity

22. A few days after an acute MI, a patient complains of stabbing chest pain that increases with deep breathing. Which action will the nurse take first? a. Notify the patient's health care provider. b. Auscultate the heart sounds. c. Check the patient's oral temperature. d. Give the ordered acetaminophen (Tylenol).

B Rationale: The patient's clinical manifestations and history are consistent with pericarditis, and the first action by the nurse should be to listen for a pericardial friction rub. Checking the temperature, giving acetaminophen (Tylenol), and notifying the health care provider are also appropriate actions but would not be done before listening for a rub. Cognitive Level: Application Text Reference: p. 872 Nursing Process: Implementation NCLEX: Physiological Integrity

2. A 69-year old patient is admitted to the hospital for elective repair of an abdominal aortic aneurysm. The history includes hypertension for 25 years, hyperlipidemia for 15 years, and smoking for 50 years. The patient asks the nurse what caused the aneurysm. The nurse's best response includes the information that a. congenital weakness of arterial walls eventually results in an aneurysm. b. atherosclerotic plaques damage the artery and may lead to aneurysms. c. chronic infections of blood vessel walls may have contributed to the aneurysm. d. uncontrolled hypertension, hyperlipidemia, and smoking caused the aneurysm.

B Rationale: The patient's gender, age, and risk factor history indicate that the aneurysm was likely caused by atherosclerosis rather than a congenital weakness or chronic infection. Although the patient's BP, elevated lipids, and smoking undoubtedly have contributed to the atherosclerotic process, it is not appropriate for the nurse to imply that the patient is to blame. Cognitive Level: Application Text Reference: p. 894 Nursing Process: Implementation NCLEX: Physiological Integrity

17. When caring for a patient with mitral valve stenosis, it is most important that the nurse assess for a. angina and syncope. b. dyspnea and hemoptysis. c. JVD and peripheral edema. d. hypotension and paroxysmal nocturnal dyspnea (PND).

B Rationale: The pressure gradient changes in mitral stenosis lead to fluid backup into the lungs, resulting in dyspnea and hemoptysis. Angina and syncope are associated with aortic stenosis. JVD and peripheral edema are more common in right-sided valvular disorders. Hypotension and PND suggest aortic regurgitation. Cognitive Level: Application Text Reference: p. 879 Nursing Process: Assessment NCLEX: Physiological Integrity

A patient develops sinus bradycardia at a rate of 32 beats/minute, has a BP of 80/36 mm Hg, and is complaining of feeling faint. Which action should the nurse take? A) a. Continue to monitor the rhythm and BP. B) b. Apply the transcutaneous pacemaker (TCP). C) c. Have the patient perform the Valsalva maneuver. D) d. Give the scheduled dose of diltiazem (Cardizem).

B) b. Apply the transcutaneous pacemaker (TCP).

During recovery from anesthesia in the postanesthesia care unit (PACU), a patient's vital signs are blood pressure 118/72, pulse 76, respirations 12, and SpO2 91%. The patient is sleepy but awakens easily. Which action should the nurse take at this time? A) a. Place the patient in a side-lying position. B) b. Encourage the patient to take deep breaths. C) c. Prepare to transfer the patient from the PACU. D) d. Increase the rate of the postoperative IV fluids.

B) b. Encourage the patient to take deep breaths.

A postoperative patient has not voided for 7 hours after return to the postsurgical unit. Which action should the nurse take first? A) a. Notify the surgeon. B) b. Perform a bladder scan. C) c. Assist the patient to ambulate to the bathroom. D) d. Insert a straight catheter as indicated on the PRN order.

B) b. Perform a bladder scan.

A patient arrives at the ambulatory surgery center for a scheduled outpatient surgery. Which information is of most concern to the nurse? A) a. The patient has not had outpatient surgery before. B) b. The patient is planning to drive home after surgery. C) c. The patient's insurance does not cover outpatient surgery. D) d. The patient had a glass of water a few hours before arriving.

B) b. The patient is planning to drive home after surgery.

. A 36-year-old woman is admitted for an outpatient surgery. Which information obtained by the nurse during the preoperative assessment is most important to report to the anesthesiologist before surgery? A) a. The patient's lack of knowledge about postoperative pain control measures B) b. The patient's statement that her last menstrual period was 8 weeks previously C) c. The patient's history of a postoperative infection following a prior cholecystectomy D) d. The patient's concern that she will be unable to care for her children postoperatively

B) b. The patient's statement that her last menstrual period was 8 weeks previously

The nurse notes that a patient's cardiac monitor shows that every other beat is earlier than expected, has no P wave, and has a QRS complex with a wide and bizarre shape. How will the nurse document the rhythm? A) a. Ventricular couplets B) b. Ventricular bigeminy C) c. Ventricular R-on-T phenomenon D) d. Ventricular multifocal contractions

B) b. Ventricular bigeminy

Data that were obtained during the perioperative nurse's assessment of a patient in the preoperative holding area that would indicate a need for special protection techniques during surgery include A) a. a stated allergy to cats and dogs. B) b. a history of spinal and hip arthritis. C) c. verbalization of anxiety by the patient. D) d. having a sip of water 2 hours previously.

B) b. a history of spinal and hip arthritis.

When caring for a patient who has survived a sudden cardiac death (SCD) event and has no evidence of an acute myocardial infarction (AMI), the nurse will anticipate teaching the patient A) a. that sudden cardiac death events rarely reoccur. B) b. about the purpose of outpatient Holter monitoring. C) c. how to self-administer low-molecular-weight heparin. D) d. to limit activities after discharge to prevent future events.

B) b. about the purpose of outpatient Holter monitoring.

A 75-year-old is to be discharged from the ambulatory surgical unit following left eye surgery. The patient tells the nurse, "I do not know if I can take care of myself with this patch over my eye." The most appropriate nursing action is to A) a. refer the patient for home health care services. B) b. discuss the specific concerns regarding self-care. C) c. give the patient written instructions regarding care. D) d. assess the patient's support system for care at home.

B) b. discuss the specific concerns regarding self-care.

A patient is seen at the health care provider's office several weeks before hip surgery for preoperative assessment. The patient reports use of echinacea, saw palmetto, and glucosamine/chondroitin. The nurse should A) a. ascertain that there will be no interactions with anesthetic agents. B) b. discuss the supplement use with the patient's health care provider. C) c. teach the patient that these products may be continued preoperatively. D) d. advise the patient to stop the use of all herbs and supplements at this time.

B) b. discuss the supplement use with the patient's health care provider.

A diabetic patient who uses insulin to control blood glucose has been NPO since midnight before having a mastectomy. The nurse will anticipate the need to A) a. withhold the usual scheduled insulin dose because the patient is NPO. B) b. obtain a blood glucose measurement before any insulin administration. C) c. give the patient the usual insulin dose because stress will increase the blood glucose. D) d. administer a lower dose of insulin because there will be no oral intake before surgery.

B) b. obtain a blood glucose measurement before any insulin administration.

Ten minutes after receiving the ordered preoperative opioid by intravenous (IV) injection, the patient asks to get up to go to the bathroom to urinate. The most appropriate action by the nurse is to A) a. assist the patient to the bathroom and stay with the patient to prevent falls. B) b. offer a urinal or bedpan and position the patient in bed to promote voiding. C) c. allow the patient up to the bathroom because the onset of the medication takes more than 10 minutes. D) d. ask the patient to wait because catheterization is performed at the beginning of the surgical procedure.

B) b. offer a urinal or bedpan and position the patient in bed to promote voiding.

An 83-year-old who had a surgical repair of a hip fracture 2 days previously has restrictions on ambulation. Based on this information, the nurse identifies the priority collaborative problem for the patient as A) a. potential complication: hypovolemic shock. B) b. potential complication: venous thromboembolism. C) c. potential complication: fluid and electrolyte imbalance. D) d. potential complication: impaired surgical wound healing.

B) b. potential complication: venous thromboembolism.

While admitting an 80-year-old with heart failure to the hospital, the nurse learns that the patient lives alone and sometimes confuses the "water pill" with the "heart pill." When planning for the patient's discharge the nurse will facilitate A) a. transfer to a dementia care service. B) b. referral to a home health care agency. C) c. placement in a long-term care facility. D) d. arrangements for around-the-clock care.

B) b. referral to a home health care agency.

As the nurse prepares a patient the morning of surgery, the patient refuses to remove a wedding ring, saying, "I have never taken it off since the day I was married." The nurse should A) a. have the patient sign a release and leave the ring on. B) b. tape the wedding ring securely to the patient's finger. C) c. tell the patient that the hospital is not liable for loss of the ring. D) d. suggest that the patient give the ring to a family member to keep.

B) b. tape the wedding ring securely to the patient's finger.

3) What statement by a pre-operative patient indicates the need for further teaching by the nurse? A. Someone will help take care of my home. B. I can drive myself home after surgery. C. My brother will bring his pet gerbil to keep me entertained. D. I'll notify the health care provider if I develop a fever.

B. I can drive myself home after surgery.

4) To improve gas exchange and oxygenation for a patient with heart failure, what nursing management should be implemented? A. Check vital signs B. Place patient in high Fowlers position. C. Place patient in semi-Fowlers position. D. Administer diuretic.

B. Place patient in high Fowlers position.

2. A patient with chronic heart failure who has been following a low-sodium diet tells the nurse at the clinic about a 5-pound weight gain in the last 3 days. The nurse's first action will be to a. ask the patient to recall the dietary intake for the last 3 days because there may be hidden sources of sodium in the patient's diet. b. instruct the patient in a low-calorie, low-fat diet because the weight gain has likely been caused by excessive intake of inappropriate foods. c. assess the patient for clinical manifestations of acute heart failure because an exacerbation of the chronic heart failure may be occurring. d. educate the patient about the use of diuretic therapy because it is likely that the patient will need medications to reduce the hypervolemia.

C Rationale: The 5-pound weight gain over 3 days indicates that the patient's chronic heart failure may be worsening; it is important that the patient be immediately assessed for other clinical manifestations of decompensation, such as lung crackles. A dietary recall to detect hidden sodium in the diet and teaching about diuretic therapy are appropriate interventions but are not the first nursing actions indicated. There is no evidence that the patient's weight gain is caused by excessive dietary intake of fat or calories, so the answer beginning "instruct the patient in a low-calorie, low-fat diet" describes an inappropriate action. Cognitive Level: Application Text Reference: p. 826 Nursing Process: Assessment NCLEX: Physiological Integrity

27. The nurse is taking a health history from a 24-year-old patient with hypertrophic cardiomyopathy (HC); which information obtained by the nurse is most relevant? a. The patient reports using cocaine once at age 16. b. The patient has a history of a recent upper respiratory infection. c. The patient's 29-year-old brother has had a sudden cardiac arrest. d. The patient has a family history of coronary artery disease (CAD).

C Rationale: About half of all cases of HC have a genetic basis, and HC is the most common cause of sudden cardiac death in otherwise healthy young people; the information about the patient's brother will be helpful in planning care (such as an automatic implantable cardioverter-defibrillator [AICD]) for the patient and in counseling other family members. The patient should be counseled against use of stimulant drugs, but the one-time use indicates that the patient is not at current risk for cocaine use. Viral infections and CAD are risk factors for dilated cardiomyopathy, but not for HC. Cognitive Level: Application Text Reference: pp. 886, 888 Nursing Process: Assessment NCLEX: Physiological Integrity

5. The nurse identifies a nursing diagnosis of risk for altered peripheral tissue perfusion related to bypass graft thrombosis for a patient following an abdominal aneurysm repair. An appropriate intervention to prevent this problem in the immediate postoperative period is to a. use a cooling blanket to maintain the patient's temperature within a normal range to prevent hypercoagulability. b. place the patient in Trendelenburg position to reduce pressure at the suture line and prevent leaking of blood at the site. c. administer IV fluids at a rate to keep the arterial BP within a normal range. d. perform passive range-of-motion (ROM) exercises to the legs hourly to promote venous return.

C Rationale: Administration of IV fluids to maintain BP within normal range will allow adequate blood flow to prevent thrombosis while preventing the risk of suture damage that can occur with high BP. Coagulation is not usually affected by temperature. The Trendelenburg position compromises respiratory status in postoperative patients. Passive ROM will not improve arterial blood flow through the graft. Cognitive Level: Application Text Reference: p. 897 Nursing Process: Planning NCLEX: Physiological Integrity

23. In planning care for a patient with a venous stasis ulcer on the right lower leg, the nurse understands that the most important intervention in promoting healing of the ulcer is a. adequate dietary intake of proteins and vitamins. b. prevention of infection with prophylactic antibiotics. c. application of external compression to the lower leg. d. keeping the ulcer moist with hydrocolloid dressings.

C Rationale: Although all the interventions are used, the most essential is compression of the leg to prevent the ulcer from becoming wider and deeper. Cognitive Level: Application Text Reference: p. 919 Nursing Process: Planning NCLEX: Physiological Integrity

25. Which of these nursing actions in the care plan for a patient who had a repair of an abdominal aortic aneurysm 5 days previously is most appropriate for the nurse to delegate to an experienced nursing assistant? a. Teach the patient the signs of possible wound infection. b. Monitor the quality and presence of the pedal pulses. c. Assist the patient in using a pillow to splint while coughing. d. Check the lower extremity strength and movement.

C Rationale: Assisting a patient who has already been taught how to cough is part of routine postoperative care and within the education and scope of practice for an experienced nursing assistant. Patient teaching and assessment of essential postoperative functions such as circulation and movement are more appropriate for the RN-level education and scope of practice. Cognitive Level: Application Text Reference: pp. 897-898 Nursing Process: Planning NCLEX: Safe and Effective Care Environment

20. An elderly patient with a 40-pack-year history of smoking and a recent myocardial infarction is admitted to the medical unit with acute shortness of breath; the nurse need to rule out pneumonia versus heart failure. The diagnostic test that the nurse will monitor to help in determining whether the patient has heart failure is a. 12-lead electrocardiogram (ECG). b. arterial blood gases (ABGs). c. B-type natriuretic peptide (BNP). d. serum creatine kinase (CK).

C Rationale: BNP is secreted when ventricular pressures increase, as with heart failure, and elevated BNP indicates a probable or very probable diagnosis of heart failure. 12-lead ECGs, ABGs, and CK may also be used in determining the causes or effects of heart failure but are not as clearly diagnostic of heart failure as BNP. Cognitive Level: Application Text Reference: p. 827 Nursing Process: Assessment NCLEX: Physiological Integrity

24. A patient is admitted to the hospital with a diagnosis of chronic venous insufficiency. Which of these statements by the patient is most consistent with the diagnosis? a. "I have burning leg pains after I walk three blocks." b. "I wake up during the night because my legs hurt." c. "I can't get my shoes on at the end of the day." d. "I can never seem to get my feet warm enough."

C Rationale: Because the edema associated with venous insufficiency increases when the patient has been standing, shoes will feel tighter at the end of the day. The other patient statements are characteristic of PAD. Cognitive Level: Application Text Reference: p. 920 Nursing Process: Implementation NCLEX: Physiological Integrity

13. The nurse identifies the nursing diagnosis of ineffective peripheral perfusion related to decreased arterial blood flow for a patient with chronic PAD. In evaluating the patient outcomes following patient teaching, the nurse determines a need for further instruction when the patient says, a. "I will have to buy some loose clothing that does not bind across my legs or waist." b. "I will change my position every hour and avoid long periods of sitting with my legs down." c. "I will use a heating pad on my feet at night to increase the circulation and warmth in my feet." d. "I will walk to the point of pain, rest, and walk again until I develop pain for a half hour daily."

C Rationale: Because the patient has impaired circulation and sensation to the feet, the use of a heating pad could lead to burns. The other patient statements are correct and indicate that teaching has been successful. Cognitive Level: Application Text Reference: p. 905 Nursing Process: Evaluation NCLEX: Physiological Integrity

12. The nurse performing an assessment with a patient who has chronic peripheral arterial disease (PAD) of the legs would expect to find a. swollen, dry, scaly ankles. b. a positive Homans' sign. c. prolonged capillary refill. d. a draining ulcer on the heel.

C Rationale: Capillary refill is prolonged in PAD because of the slower and decreased blood flow to the periphery. The other listed clinical manifestations are consistent with chronic venous disease. Cognitive Level: Application Text Reference: p. 901 Nursing Process: Assessment NCLEX: Physiological Integrity

4. The nurse identifies the nursing diagnosis of decreased cardiac output related to valvular insufficiency for the patient with IE based on the assessment finding of a. petechiae of the buccal mucosa and conjunctiva. b. fever, chills, and diaphoresis. c. urine output less than 30 ml/hr. d. an increase in pulse rate of 15 beats/min with activity.

C Rationale: Decreased renal perfusion caused by inadequate cardiac output will lead to poor urine output. Petechiae, fever, chills, and diaphoresis are symptoms of IE but are not caused by decreased cardiac output. An increase in pulse rate of 15 beats/min is normal with exercise Cognitive Level: Application Text Reference: p. 870 Nursing Process: Diagnosis NCLEX: Physiological Integrity

1. The nurse obtains a health history from a patient with a prosthetic mitral valve who has symptoms of infective endocarditis. Which question by the nurse is most appropriate? a. "Do you have a history of a heart attack?" b. "Have you any recent immunizations?" c. "Have you been to the dentist lately?" d. "Is there a family history of endocarditis?"

C Rationale: Dental procedures place the patient with a prosthetic mitral valve at risk for infectious endocarditis (IE). Myocardial infarction (MI), immunizations, and a family history of endocarditis are not risk factors for IE. Cognitive Level: Application Text Reference: pp. 867, 869 Nursing Process: Assessment NCLEX: Physiological Integrity

25. When caring for the patient with infective endocarditis of the tricuspid valve, the nurse will plan to monitor the patient for a. flank pain. b. hemiparesis. c. dyspnea. d. splenomegaly.

C Rationale: Embolization from the tricuspid valve would cause symptoms of pulmonary embolus. Flank pain, hemiparesis, and splenomegaly would be associated with embolization from the left-sided valves. Cognitive Level: Application Text Reference: p. 867 Nursing Process: Planning NCLEX: Physiological Integrity

16. A home health care patient has recently started taking oral digoxin (Lanoxin) and furosemide (Lasix) for control of heart failure. The patient data that will require the most immediate action by the nurse is if the patient's a. weight increases from 120 pounds to 122 pounds over 3 days. b. liver is palpable 2 cm below the ribs on the right side. c. serum potassium level is 3.0 mEq/L after 1 week of therapy. d. has 1 to 2+ edema in the feet and ankles in the morning.

C Rationale: Hypokalemia potentiates the actions of digoxin and increases the risk for digoxin toxicity, which can cause life-threatening dysrhythmias. The other data indicate that the patient's heart failure requires more effective therapies, but they do not require nursing action as rapidly as the low serum potassium level. Cognitive Level: Application Text Reference: pp. 832-833 Nursing Process: Assessment NCLEX: Physiological Integrity

10. Cardiac tamponade is suspected in a patient who has acute pericarditis. To assess for the presence of pulsus paradoxus, the nurse should a. subtract the diastolic blood pressure (DBP) from the systolic blood pressure (SBP). b. auscultate for a pericardial friction rub that increases in volume during inspiration. c. note when Korotkoff sounds are audible during both inspiration and expiration. d. check the ECG for variations in rate in relation to inspiration and expiration.

C Rationale: Pulsus paradoxus exists when there is a gap of greater than 10 mm Hg between when Korotkoff sounds can be heard during only expiration and when they can be heard throughout the respiratory cycle. The other methods described would not be useful in determining the presence of pulsus paradoxus. Cognitive Level: Comprehension Text Reference: p. 873 Nursing Process: Assessment NCLEX: Physiological Integrity

11. The nurse has identified a nursing diagnosis of acute pain related to inflammatory process for a patient with acute pericarditis. The most appropriate intervention by the nurse for this problem is to a. force fluids to 3000 ml/day to decrease fever and inflammation. b. teach the patient to take deep, slow respirations to control the pain. c. position the patient in Fowler's position, leaning forward on the overbed table. d. remind the patient to ask for the opioid pain medication every four hours.

C Rationale: Sitting upright and leaning forward frequently will decrease the pain associated with pericarditis. Forcing fluids will not decrease the inflammation or pain. Taking deep respirations tends to increase pericardial pain. Opioids are not very effective at controlling pain caused by acute inflammatory conditions and are usually ordered PRN. The patient would receive scheduled doses of a nonsteroidal antiinflammatory drug (NSAID). Cognitive Level: Application Text Reference: p. 874 Nursing Process: Planning NCLEX: Physiological Integrity

30. When developing a teaching plan for a patient newly diagnosed with PAD, which information should the nurse include? a. "Try to keep your legs elevated whenever you are sitting." b. "Exercise only if you do not experience pain." c. "It is important to try to stop smoking." d. "Put on support hose early in the day before swelling occurs."

C Rationale: Smoking cessation is essential for slowing the progression of PAD to critical limb ischemia and reducing the risk of myocardial infarction and death. Circulation to the legs will decrease if the legs are elevated. Patients with PAD are taught to exercise to feeling pain, rest, and then resume walking. Support hose are not used for patients with PAD. Cognitive Level: Application Text Reference: p. 901 Nursing Process: Planning NCLEX: Physiological Integrity

3. During assessment of a 72-year-old with ankle swelling, the nurse notes jugular venous distention (JVD) with the head of the patient's bed elevated 45 degrees. The nurse knows this finding indicates a. decreased fluid volume. b. incompetent jugular vein valves. c. elevated right atrial pressure. d. jugular vein atherosclerosis.

C Rationale: The jugular veins empty into the superior vena cava and then into the right atrium, so JVD with the patient sitting at a 45-degree angle reflects elevated right atrial pressure. JVD is an indicator of excessive fluid volume (increased preload), not decreased fluid volume; it is not caused by incompetent jugular vein valves or atherosclerosis. Cognitive Level: Comprehension Text Reference: p. 825 Nursing Process: Assessment NCLEX: Physiological Integrity

9. During a visit to an elderly patient with chronic heart failure, the home care nurse finds that the patient has severe dependent edema and that the legs appear to be weeping serous fluid. Based on these data, the best nursing diagnosis for the patient is a. activity intolerance related to venous congestion. b. disturbed body image related to massive leg swelling. c. impaired skin integrity related to peripheral edema. d. impaired gas exchange related to chronic heart failure.

C Rationale: The patient's findings of severe dependent edema and weeping serous fluid from the legs support the nursing diagnosis of impaired skin integrity. There is less evidence for the nursing diagnoses of activity intolerance, disturbed body image, and impaired gas exchange, although the nurse will further assess the patient to determine whether there are other clinical manifestations of heart failure to indicate that these diagnoses are appropriate. Cognitive Level: Application Text Reference: p. 836 Nursing Process: Diagnosis NCLEX: Physiological Integrity

10. A patient recovering on a general surgical unit from an aortic valve replacement 1 week ago develops sudden severe pain, pulselessness, pallor, and coolness in the left leg. The nurse should notify the health care provider and a. elevate the left leg on a pillow. b. apply an elastic wrap to the leg. c. keep the patient in bed in the supine position. d. assist the patient in gently exercising the leg.

C Rationale: The patient's history and clinical manifestations are consistent with acute arterial occlusion, and resting the leg will decrease the oxygen demand of the tissues and minimize ischemic damage until circulation can be restored. Elevating the leg or applying an elastic wrap will further compromise blood flow to the leg. Exercise with increase oxygen demand for the tissues of the leg. Cognitive Level: Application Text Reference: p. 907 Nursing Process: Implementation NCLEX: Physiological Integrity

5. A patient hospitalized with a streptococcal infective endocarditis tells the nurse," I know that I need antibiotics, but I do not want to be hospitalized for very long." The nurse explains that a. after 2 weeks of IV antibiotic therapy, the patient may be discharged with oral antibiotics to take for another 4 weeks. b. hospitalization for 4 to 6 weeks will be necessary to prevent a relapse while receiving IV antibiotic therapy. c. the patient will be able to receive outpatient IV antibiotic therapy if complications such as heart failure do not develop. d. hospitalization for IV antibiotics is necessary until the fever is resolved, but then the patient can be discharged on oral antibiotics.

C Rationale: Treatment for IE involves 4 to 6 weeks of IV antibiotic therapy in order to eradicate the bacteria, but patients frequently receive IV antibiotics on an outpatient basis. Oral antibiotics do not result in the consistently high antibiotic level that is needed to eradicate the infective bacteria. The resolution of the fever is not an indication that the patient can be discharged or that oral antibiotics are adequate for treatment. Cognitive Level: Application Text Reference: p. 871 Nursing Process: Implementation NCLEX: Physiological Integrity

Which of these statements made by a patient with coronary artery disease after the nurse has completed teaching about the therapeutic lifestyle changes (TLC) diet indicates that further teaching is needed? A) a. "I will switch from whole milk to 1% or nonfat milk." B) b. "I like fresh salmon and I will plan to eat it more often." C) c. "I will miss being able to eat peanut butter sandwiches." D) d. "I can have a cup of coffee with breakfast if I want one."

C "I will miss being able to eat peanut butter sandwiches."

To assist the patient with coronary artery disease (CAD) in making appropriate dietary changes, which of these nursing interventions will be most effective? A) a. Instruct the patient that a diet containing no saturated fat and minimal sodium will be necessary. B) b. Emphasize the increased risk for cardiac problems unless the patient makes the dietary changes. C) c. Assist the patient to modify favorite high-fat recipes by using monosaturated oils when possible. D) d. Provide the patient with a list of low-sodium, low-cholesterol foods that should be included in the diet.

C Assist the patient to modify favorite high-fat recipes by using monosaturated oils when possible.

A patient who has had severe chest pain for several hours is admitted with a diagnosis of possible acute myocardial infarction (AMI). Which of these ordered laboratory tests should the nurse monitor to help determine whether the patient has had an AMI? A) a. Homocysteine B) b. C-reactive protein C) c. Cardiac-specific troponin I and troponin T D) d. High-density lipoprotein (HDL) cholesterol

C Cardiac-specific troponin I and troponin T

Which action will the nurse in the hypertension clinic take in order to obtain an accurate baseline blood pressure (BP) for a new patient? A) a. Obtain a BP reading in each arm and average the results. B) b. Deflate the BP cuff at a rate of 5 to 10 mm Hg per second. C) c. Have the patient sit in a chair with the feet flat on the floor. D) d. Assist the patient to the supine position for BP measurements.

C Have the patient sit in a chair with the feet flat on the floor.

While assessing a patient who was admitted with heart failure, the nurse notes that the patient has jugular venous distention (JVD) when lying flat in bed. Which action should the nurse take next? A) a. Use a ruler to measure the level of the JVD. B) b. Document this finding in the patient's record. C) c. Observe for JVD with the head at 30 degrees. D) d. Have the patient perform the Valsalva maneuver.

C Observe for JVD with the head at 30 degrees.

During change-of-shift report, the nurse obtains this information about a hypertensive patient who received the first dose of propranolol (Inderal) during the previous shift. Which information indicates that the patient needs immediate intervention? A) a. The patient's most recent BP reading is 156/94 mm Hg. B) b. The patient's pulse has dropped from 64 to 58 beats/minute. C) c. The patient has developed wheezes throughout the lung fields. D) d. The patient complains that the fingers and toes feel quite cold

C The patient has developed wheezes throughout the lung fields.

A patient is scheduled for a cardiac catheterization with coronary angiography. Before the test, the nurse informs the patient that A) a. electrocardiographic (ECG) monitoring will be required for 24 hours after the test. B) b. it will be important to lie completely still during the procedure. C) c. a warm feeling may be noted when the contrast dye is injected. D) d. monitored anesthesia care will be provided during the procedure.

C a warm feeling may be noted when the contrast dye is injected

The nurse teaches the patient being evaluated for rhythm disturbances with a Holter monitor to A) a. exercise more than usual while the monitor is in place. B) b. remove the electrodes when taking a shower or tub bath. C) c. keep a diary of daily activities while the monitor is worn. D) d. connect the recorder to a telephone transmitter once daily.

C keep a diary of daily activities while the monitor is worn.

A patient has received instruction on the management of a new permanent pacemaker before discharge from the hospital. The nurse recognizes that teaching has been effective when the patient tells the nurse, A) a. "It will be 6 weeks before I can take a bath or return to my usual activities." B) b. "I will notify the airlines when I make a reservation that I have a pacemaker." C) c. "I won't lift the arm on the pacemaker side up very high until I see the doctor." D) d. "I must avoid cooking with a microwave oven or being near a microwave in use."

C) c. "I won't lift the arm on the pacemaker side up very high until I see the doctor."

A patient has a junctional escape rhythm on the monitor. The nurse will expect the patient to have a heart rate of how many beats/minute? A) a. 15 to 20 B) b. 20 to 40 C) c. 40 to 60 D) d. 60 to 100

C) c. 40 to 60

In intervening to promote ambulation, coughing, deep breathing, and turning by a postoperative patient on the first postoperative day, which action by the nurse is most helpful? A) a. Discuss the complications of immobility and poor cough effort. B) b. Teach the patient the purpose of respiratory care and ambulation. C) c. Administer ordered analgesic medications before these activities. D) d. Give the patient positive reinforcement for accomplishing these activities.

C) c. Administer ordered analgesic medications before these activities

A patient who was admitted with a myocardial infarction experiences a 50-second episode of ventricular tachycardia, then converts to sinus rhythm with a heart rate of 98 beats/minute. Which action should the nurse take next? A) a. Notify the health care provider. B) b. Perform synchronized cardioversion. C) c. Administer the PRN IV lidocaine (Xylocaine). D) d. Document the rhythm and monitor the patient.

C) c. Administer the PRN IV lidocaine (Xylocaine).

After removal of the nasogastric (NG) tube on the second postoperative day, the patient is placed on a clear liquid diet. Four hours later, the patient complains of sharp, cramping gas pains. Which action should the nurse take? A) a. Reinsert the NG tube. B) b. Give the PRN IV opioid. C) c. Assist the patient to ambulate. D) d. Place the patient on NPO status.

C) c. Assist the patient to ambulate.

Following gallbladder surgery, a patient's T-tube is draining dark green fluid. Which action should the nurse take? A) a. Place the patient on bed rest. B) b. Notify the patient's surgeon. C) c. Document the color and amount of drainage. D) d. Irrigate the T-tube with sterile normal saline.

C) c. Document the color and amount of drainage.

Following an acute myocardial infarction (AMI), a patient ambulates in the hospital hallway. When the nurse is evaluating the patient's response, which of these assessment data would indicate that the exercise level should be decreased? A) a. BP changes from 118/60 to 126/68 mm Hg. B) b. Oxygen saturation drops from 100% to 98%. C) c. Heart rate increases from 66 to 90 beats/minute. D) d. Respiratory rate goes from 14 to 22 breaths/minute.

C) c. Heart rate increases from 66 to 90 beats/minute.

Which action will the nurse include in the plan of care immediately after surgery for a patient who received ketamine (Ketalar) as an anesthetic agent? A) a. Administer larger doses of analgesic agents. B) b. Monitor for severe slowing of the heart rate. C) c. Provide a quiet environment in the postanesthesia care unit. D) d. Avoid the use of benzodiazepines in the postoperative period.

C) c. Provide a quiet environment in the postanesthesia care unit.

An alert 82-year-old who has poor hearing and vision is receiving preoperative teaching from the nurse. His wife answers most questions directed to the patient. Which action should the nurse take when doing the teaching? A) a. Use printed materials for instruction so that the patient will have more time to review the material. B) b. Direct the teaching toward the wife because she is the obvious support and caregiver for the patient. C) c. Provide additional time for the patient to understand preoperative instructions and carry out procedures. D) d. Ask the patient's wife to wait in the hall in order to focus preoperative teaching with the patient himself.

C) c. Provide additional time for the patient to understand preoperative instructions and carry out procedures.

The nurse needs to estimate quickly the heart rate for a patient with a regular heart rhythm. Which method will be best to use? A) a. Print a 1-minute electrocardiogram (ECG) strip and count the number of QRS complexes. B) b. Count the number of large squares in the R-R interval and divide by 300. C) c. Use the 3-second markers to count the number of QRS complexes in 6 seconds and multiply by 10. D) d. Calculate the number of small squares between one QRS complex and the next and divide into 1500.

C) c. Use the 3-second markers to count the number of QRS complexes in 6 seconds and multiply by 10.

Which action by an inexperienced member of the surgical team requires rapid intervention by the charge nurse? A) a. Wearing street clothes into the nursing station B) b. Wearing a surgical mask into the holding room C) c. Walking into the hallway outside an operating room without the hair covered D) d. Putting on a surgical mask, cap, and scrubs before entering the operating room

C) c. Walking into the hallway outside an operating room without the hair covered

During the administration of the fibrinolytic agent to a patient with an acute myocardial infarction (AMI), the nurse should stop the drug infusion if the patient experiences A) a. bleeding from the gums. B) b. surface bleeding from the IV site. C) c. a decrease in level of consciousness. D) d. a nonsustained episode of ventricular tachycardia.

C) c. a decrease in level of consciousness.

Intravenous sodium nitroprusside (Nipride) is ordered for a patient with acute pulmonary edema. During the first hours of administration, the nurse will need to adjust the nitroprusside rate if the patient develops A) a. a dry, hacking cough. B) b. any ventricular ectopy. C) c. a systolic BP <90 mm Hg. D) d. a heart rate <50 beats/minute.

C) c. a systolic BP <90 mm Hg.

A patient with dilated cardiomyopathy has an atrial fibrillation that has been unresponsive to drug therapy for several days. The nurse anticipates that the patient may need teaching about A) a. electrical cardioversion. B) b. IV adenosine (Adenocard). C) c. anticoagulant therapy with warfarin (Coumadin). D) d. insertion of an implantable cardioverter-defibrillator (ICD).

C) c. anticoagulant therapy with warfarin (Coumadin).

When teaching the patient with heart failure about a 2000-mg sodium diet, the nurse explains that foods to be restricted include A) a. canned and frozen fruits. B) b. fresh or frozen vegetables. C) c. milk, yogurt, and other milk products. D) d. eggs and other high-cholesterol foods.

C) c. milk, yogurt, and other milk products.

The nurse plans discharge teaching for a patient with chronic heart failure who has prescriptions for digoxin (Lanoxin) and hydrochlorothiazide (HydroDIURIL). Appropriate instructions for the patient include A) a. avoid dietary sources of potassium. B) b. take the hydrochlorothiazide before bedtime. C) c. notify the health care provider about any nausea. D) d. never take digoxin if the pulse is below 60 beats/minute

C) c. notify the health care provider about any nausea.

The nurse from the general surgical unit is asked to bring the patient's hearing aid to the surgical suite. The nurse will take the hearing aid to the A) a. clean core. B) b. scrub sink areas. C) c. nursing station or information desk. D) d. corridors of the operating room area.

C) c. nursing station or information desk.

The nurse evaluates that the interventions for the nursing diagnosis of ineffective airway clearance in a postoperative patient have been successful when the A) a. patient drinks 2 to 3 L of fluid in 24 hours. B) b. patient uses the spirometer 10 times every hour. C) c. patient's breath sounds are clear to auscultation. D) d. patient's temperature is less than 100.4° F orally.

C) c. patient's breath sounds are clear to auscultation.

During the preoperative assessment of a patient scheduled for a colon resection, the patient tells the nurse about using St. John's wort to prevent depression. The nurse should alert the staff in the postanesthesia recovery area that the patient may A) a. experience increased pain. B) b. have hypertensive episodes. C) c. take longer to recover from the anesthesia. D) d. have more postoperative bleeding than expected.

C) c. take longer to recover from the anesthesia.

After a new nurse has been oriented to the postanesthesia care unit (PACU), the charge nurse will evaluate that the orientation has been successful when the new nurse A) a. places a patient in the Trendelenburg position when the blood pressure (BP) drops. B) b. assists a patient to the prone position when the patient is nauseated. C) c. turns an unconscious patient to the side when the patient arrives in the PACU. D) d. positions a newly admitted unconscious patient supine with the head elevated.

C) c. turns an unconscious patient to the side when the patient arrives in the PACU.

6) Which of the following is not a correct nursing and collaborative management action for heart failure? A. High Fowlers position B. Improve gas exchange and oxygenation C. Increase fluid intake D. ECG monitoring

C. Increase fluid intake

Which action will be included in the plan of care when the nurse is caring for a patient who is receiving sodium nitroprusside (Nipride) to treat a hypertensive emergency? A) a. Organize nursing activities so that the patient has undisturbed sleep for 6 to 8 hours at night. B) b. Assist the patient up in the chair for meals to avoid complications associated with immobility. C) c. Use an automated noninvasive blood pressure machine to obtain frequent BP measurements. D) d. Place the patient on NPO status to prevent aspiration caused by nausea and the associated vomiting.

C. Use an automated noninvasive blood pressure machine to obtain frequent BP measurements.

note when Korotkoff sounds are audible during both inspiration and expiration.

Cardiac tamponade is suspected in a patient who has acute pericarditis. To assess for the presence of pulsus paradoxus, the nurse should _____________

42. Death in the prehospital setting following an acute myocardial infarction is MOST often the result of: a. Asystole b. Myocardial rupture c. Cardiogenic shock d. Ventricular fibrillation

Cardiogenic shock

17. Which of the following medications is a calcium channel blocker? a. Lanoxin b. Cardizem c. Tenormin d. Capoten

Cardizem

The nurse has received the laboratory results for a patient who developed chest pain 2 hours ago and may be having a myocardial infarction. The most important laboratory result to review will be a. troponins T and I. b. creatine kinase-MB. c. LDL cholesterol. d. C-reactive protein.

Correct Answer: A Rationale: Cardiac troponins start to elevate 1 hour after myocardial injury and are specific to myocardium. Creatine kinase (CK-MB) is specific to myocardial injury and infarction, but it does not increase until 4 to 6 hours after the infarction occurs. LDL cholesterol and C-reactive protein are useful in assessing cardiovascular risk but are not helpful in determining whether a patient is having an acute myocardial infarction. Cognitive Level: Application Text Reference: pp. 751-752 Nursing Process: Assessment NCLEX: Physiological Integrity

A patient is diagnosed with hypertension, and first-line drug therapy with a β-adrenergic blocking agent is planned. After reviewing the patient's history, the nurse consults with the health care provider about the use of this drug upon finding a history of a. asthma. b. peptic ulcer disease. c. alcohol dependency. d. myocardial infarction (MI).

Correct Answer: A Rationale: Cardioselective β-adrenergic blocking agents block β1-adrenergic receptors. Nonselective agents block β1- and β2-adrenergic receptors. Nonselective agents may cause bronchospasm, especially in patients with a history of asthma. β-blockers will have no effect on the patient's peptic ulcer disease or alcohol dependency. The use of a β-blocker for a hypertensive patient who has had an MI is appropriate because β-blocker therapy is recommended after MI. Cognitive Level: Application Text Reference: p. 774 Nursing Process: Assessment NCLEX: Physiological Integrity

The nurse is evaluating the response to treatment for a patient has recently started taking furosemide (Lasix) to treat stage 2 hypertension. The information that will require the nurse to act most rapidly is a(n) a. blood potassium level of 3.0 mEq/L. b. blood glucose level of 180 mg/dl. c. BP reading of 164/96. d. orthostatic decrease of 12 mm Hg.

Correct Answer: A Rationale: Hypokalemia is a frequent adverse effect of the loop diuretics and can cause life-threatening dysrhythmias. The health care provider should be notified of the potassium level immediately and administration of potassium supplements initiated. The elevated blood glucose and BP also indicate a need for collaborative interventions but will not require action as urgently as the hypokalemia. An orthostatic drop of 12 mm Hg is common and will require intervention only if the patient is symptomatic. Cognitive Level: Application Text Reference: pp. 772-773 Nursing Process: Evaluation NCLEX: Physiological Integrity

A patient with hypertension asks the nurse why lifestyle changes are needed when the patient has no symptoms from the high BP. The response by the nurse that is most likely to improve patient compliance with therapy is that hypertension a. damages the blood vessels leading to risk for heart attack, stroke, and kidney failure. b. increases blood flow to the kidneys leading to increased workload for the renal system. c. may not cause any problems for some people but does cause symptoms in many others. d. is probably causing symptoms but the patient does not recognize that they are occurring.

Correct Answer: A Rationale: Teaching the patient that hypertension can damage blood vessels and eventually cause severe health problems is most likely to improve patient compliance with needed lifestyle changes. The increased renal blood flow caused by hypertension does not damage the kidneys. Teaching the patient that hypertension may cause symptoms in some people or that the patient has unrecognized symptoms is unlikely to provide the motivation for behavior changes.

The charge nurse observes a new RN doing discharge teaching for a hypertensive patient who has a new prescription for enalapril (Vasotec). The charge nurse will need to intervene if the new RN tells the patient to a. increase the dietary intake of high potassium foods. b. move slowly when moving from lying to standing. c. check the BP with a home BP monitor every day. d. make an appointment with the dietitian for teaching about a low-sodium diet.

Correct Answer: A Rationale: The ACE inhibitors cause retention of potassium by the kidney, so hyperkalemia is a possible adverse effect. The other teaching by the new RN is appropriate for a patient with newly diagnosed hypertension who has just started therapy with enalapril. Cognitive Level: Application Text Reference: p. 772 Nursing Process: Implementation NCLEX: Physiological Integrity

The nurse teaches the patient with stage 1 hypertension about diet modifications that should be implemented to manage BP. Which diet choice indicates that the teaching has been effective? a. The patient has a glass of low-fat milk with each meal. b. The patient has only one cup of coffee in the morning. c. The patient restricts intake of dietary protein. d. The patient has tomato juice and bacon for breakfast.

Correct Answer: A Rationale: The DASH recommendations for prevention of hypertension include increasing the intake of calcium-rich foods. Caffeine restriction and decreased protein intake are not included in the recommendations. Tomato juice is very high in sodium and bacon is very high in sodium and fat, both of which should be restricted in patients with hypertension. Cognitive Level: Application Text Reference: p. 770 Nursing Process: Evaluation NCLEX: Health Promotion and Maintenance

The nurse hears a murmur between the S1 and S2 heart sounds at the patient's left 5th intercostal space and midclavicular line. The best way to record this information is a. "systolic murmur heard at mitral area." b. "diastolic murmur heard at aortic area." c. "systolic murmur heard at Erb's point." d. "diastolic murmur heard at tricuspid area."

Correct Answer: A Rationale: The S1 sound is created by closure of the mitral and tricuspid valves and signifies the onset of ventricular systole. S2 is caused by the closure of the aortic and pulmonic valves and signifies the onset of diastole. A murmur occurring between these two sounds is a systolic murmur. The mitral area is the intersection of the left 5th intercostal space and the midclavicular line. The aortic area is located at the 2nd intercostal space along the right sternal border. Erb's point is located at the 3rd intercostal space along the left sternal border. The tricuspid area is located at the 5th intercostal space along the left sternal border.

The nurse obtains the following information about hypertension risk factors from a patient with prehypertension. The risk factor that will be most important to address with the patient is that the patient a. gets no regular aerobic exercise. b. is 5 pounds over the ideal weight. c. has a low dietary fiber intake. d. drinks wine with dinner once a week.

Correct Answer: A Rationale: The recommendations for preventing hypertension include exercising aerobically for 30 minutes most days of the week. A weight that is 5 pounds over the ideal body weight is not a risk factor for hypertension. The DASH diet is high in fiber, but increasing fiber alone will not prevent hypertension from developing. The dietary recommendation for alcohol is for no more than one drink a day for women and small adults or two drinks a day for men. Cognitive Level: Application Text Reference: pp. 768, 770, 779 Nursing Process: Planning NCLEX: Health Promotion and Maintenance

uring physical examination of a thin 72-year-old patient, the nurse observes pulsation of the abdominal aorta in the epigastric area just below the xiphoid process. The nurse teaches the patient that this is a. a normal assessment finding for a thin individual. b. likely to be caused by age-related sclerosis and inelasticity of the aorta. c. an indication that an abdominal aortic aneurysm has probably developed. d. evidence of elevated systemic arterial pressure.

Correct Answer: A Rationale: Visible pulsation of the abdominal aorta is commonly observed in the epigastric area for thin individuals. More data would be needed to support a diagnosis of aortic sclerosis, aortic aneurysm, or hypertension. Cognitive Level: Application Text Reference: p. 750 Nursing Process: Assessment NCLEX: Physiological Integrity

A patient with stage 1 hypertension who received a new prescription for atenolol (Tenormin) returns to the health clinic after 2 weeks for a follow-up visit. BP is unchanged from the previous clinic visit. The nurse's first action will be to a. ask the patient about whether the medication is actually being taken. b. teach the patient about the reasons for an increase in the medication dose. c. provide information about the use of multiple drugs to treat hypertension. d. remind the patient that lifestyle changes are also important in BP control.

Correct Answer: A Rationale: β-adrenergic blockers cause adverse effects (such as erectile dysfunction, fatigue, and depression) in some patients, leading to noncompliance. It is important to determine whether the patient has stopped taking the medication before initiating any changes in therapy, such as increasing the atenolol dose or adding a second medication. Whereas reinforcement of the continued need for lifestyle changes is appropriate, a decrease in BP would be expected after initiation of medication therapy even if lifestyle changes had not occurred.

When auscultating over the patient's abdominal aorta, the nurse hears a humming sound. The nurse documents this finding as a a. thrill. b. bruit. c. heave. d. murmur.

Correct Answer: B Rationale: A bruit is the sound created by turbulent blood flow in an artery. Thrills are palpable vibrations felt when there is turbulent blood flow through the heart or in a blood vessel. Heaves are sustained lifts over the precordium that can be observed or palpated. A murmur is the sound caused by turbulent blood flow through the heart. Cognitive Level: Comprehension Text Reference: pp. 748, 750 Nursing Process: Assessment NCLEX: Physiological Integrity

The RN has developed a care plan for a patient with a hypertensive crisis who is receiving sodium nitroprusside (Nipride). Which of the following nursing actions should not be delegated to an LPN/LVN who is working in the ICU? a. Reposition the patient every 2 hours. b. Titrate nitroprusside to maintain BP 160/100 mm Hg. c. Check and document urine output hourly. d. Monitor oxygen saturation every 2 hours.

Correct Answer: B Rationale: LPN/LVN education and scope of practice include nursing tasks such as repositioning patients, monitoring and documenting urine output and oxygen saturation, and administration of oral medications. Titration of vasoactive medications requires RN-level education and scope of practice. Cognitive Level: Application Text Reference: pp. 781-782 Nursing Process: Evaluation NCLEX: Physiological Integrity

A 62-year-old patient who has just arrived in the emergency department complaining of a sudden-onset severe headache and nausea has a BP of 240/118 mm Hg. The patient gives a history of taking clonidine (Catapres) and hydrochlorothiazide (HydroDIURIL) for 10 years for hypertension. The most appropriate question by the nurse at this time is a. Have you recently taken any antihistamine medications? b. Have you been taking the Catapres and HydroDIURIL lately? c. Do you have any recent stressful events in your life? d. Did you take any acetaminophen (Tylenol) yet today?

Correct Answer: B Rationale: Sudden withdrawal of antihypertensive medications can cause rebound hypertension and hypertensive crisis. Although many over-the-counter medications can cause hypertension, antihistamines and acetaminophen do not increase BP. Stressful events will increase BP, but not usually to the level in this patient. Cognitive Level: Analysis Text Reference: p. 779 Nursing Process: Assessment NCLEX: Physiological Integrity

During a physical examination of a patient, the nurse palpates the PMI in the sixth intercostal space lateral to the midclavicular line. The most appropriate action for the nurse to take next will be to a. document that the PMI is in the normal location. b. assess the patient for symptoms of left ventricular hypertrophy. c. ask the patient about risk factors for coronary artery disease. d. auscultate both the carotid arteries for a bruit.

Correct Answer: B Rationale: The PMI should be felt at the intersection of the 5th intercostal space and the midclavicular line. A PMI located outside these landmarks indicates possible cardiac enlargement, such as with left ventricular hypertrophy. Cardiac enlargement is not necessarily associated with coronary or carotid artery disease. Cognitive Level: Application Text Reference: pp. 750-751 Nursing Process: Assessment NCLEX: Physiological Integrity

The RN is observing a student nurse who is doing a physical assessment on a patient. The RN will need to intervene immediately if the student nurse a. presses on the skin over the tibia for 10 seconds to check for edema. b. palpates both carotid arteries simultaneously to compare pulse quality. c. places the patient in the left lateral position to check for the PMI. d. uses the palm of the hand to assess extremity skin temperature.

Correct Answer: B Rationale: The carotid pulses should never be palpated at the same time to avoid vagal stimulation, dysrhythmias, and decreased cerebral blood flow. The other assessment techniques also need to be corrected; however, they are not dangerous to the patient. Cognitive Level: Application Text Reference: p. 748 Nursing Process: Assessment NCLEX: Physiological Integrity

An 86-year-old patient lives alone and is on a fixed income. The patient is taking once-daily doses of metoprolol (Lopressor) and furosemide (Lasix) to control BP. The patient is able to tell the nurse the names of the medications and when they are to be taken but does not always take the medications regularly, so BP is not well controlled. The most appropriate action by the nurse will be to a. discuss the patient's possible confusion with a family member. b. ask the patient about whether the cost of the medications is too high. c. offer the patient teaching about long-term effects of hypertension. d. assist the patient with an easier dosing schedule to improve compliance.

Correct Answer: B Rationale: The cost of medications is a common cause of lack of medication compliance in older patients with fixed incomes. The patient is well-oriented and well-informed about the medications, and there is no indication that further education about the possible effects of chronic hypertension is needed. The once-daily dosing schedule is already in place and is optimal for assuring compliance with the medications. Cognitive Level: Application Text Reference: pp. 778, 780 Nursing Process: Implementation NCLEX: Physiological Integrity

During change-of-shift report, the nurse obtains all of this information about a hypertensive patient who received the first dose of nadolol (Corgard) during the previous shift. The information that will be of most concern to the nurse is that a. the patient's heart rate has dropped from 64 to 58 beats/min. b. the patient has developed wheezes throughout the lung fields. c. the patient complains that the fingers and toes feel quite cold. d. the patient's most recent BP is 156/94 mm Hg.

Correct Answer: B Rationale: The most urgent concern for this patient is the wheezes, which indicate that bronchospasm (a common adverse effect of the noncardioselective β-blockers) is occurring. The nurse should immediately obtain an oxygen saturation measurement, apply supplemental oxygen, and notify the health care provider. The mild decrease in heart rate and complaint of cold fingers and toes are associated with the β-receptor blockage caused by the nadolol but do not require any change in therapy. The BP reading may indicate that a change in medication type or dose may be indicated; however, this is not as urgently needed as addressing the bronchospasm. Cognitive Level: Application Text Reference: p. 774 Nursing Process: Evaluation NCLEX: Physiological Integrity

A patient is scheduled for a cardiac catheterization with coronary angiography. Before the test, the nurse informs the patient that a. a catheter will be inserted into a vein in the arm or leg and advanced to the heart. b. ECG monitoring will be required for 24 hours following the test to detect any dysrhythmias. c. a feeling of warmth may be experienced as the contrast material is injected into the catheter. d. it will be important to lie completely still during the coronary angiography procedure.

Correct Answer: C Rationale: A sensation of warmth or flushing is common when the iodine-based contrast material is injected, which can be anxiety-producing unless it has been discussed with the patient. The catheter is inserted in an artery (typically the femoral artery) and advanced to the openings for the coronary arteries at the aortic root. Dysrhythmias may occur during the procedure, but most patients are discharged a few hours after the coronary arteriogram or angiogram is completed. The patient is not required to be completely immobile during the procedure. Cognitive Level: Application Text Reference: pp. 755, 759 Nursing Process: Implementation NCLEX: Physiological Integrity

The nurse measures the BP of a 78-year-old patient and finds it to be 168/86 mm Hg in both arms. The nurse will plan to teach the patient that a. increased BP is a normal finding in older adults. b. prehypertension indicates the need for lifestyle changes. c. it is important to address the increased BP. d. there is a high probability of kidney and heart disease.

Correct Answer: C Rationale: Although an increase in systolic BP (SBP) is a common finding in older adults, the recommendations for treating elevated BP are unchanged. An SBP of >140 mm Hg is a more important cardiovascular risk factor than diastolic BP (DBP) in individuals older than 50. The diagnosis of prehypertension indicates a systolic BP between 120 and 139 and a DBP between 80 and 89. Kidney and heart disease are common complications of hypertension, but there are no data to support these as diagnoses for this patient. Cognitive Level: Application Text Reference: p. 779 Nursing Process: Planning NCLEX: Health Promotion and Maintenance

The nurse is has just finished medication teaching for a hypertensive patient who has a new prescription for quinapril (Accupril). The patient statement that indicates that more teaching is needed is a. "I will call the doctor if I notice that I have a frequent cough." b. "The medication may not work as well if I take any aspirin." c. "I won't worry if I have a little swelling around my lips and face." d. "The doctor may order a blood potassium level occasionally."

Correct Answer: C Rationale: Angioedema occurring with angiotension-converting enzyme (ACE)-inhibitor therapy is an indication that the ACE-inhibitor should be discontinued. The patient should be taught that if any swelling of the face or oral mucosa occurs, the health care provider should be immediately notified because this could be life threatening. The other patient statements indicate that the patient has an accurate understanding of ACE-inhibitor therapy. Cognitive Level: Application Text Reference: p. 776 Nursing Process: Evaluation NCLEX: Physiological Integrity

The nurse teaches a patient who is taking labetalol (Normodyne) for treatment of hypertension to change position slowly because this drug a. blocks the renin-angiotensin-aldosterone system (RAAS). b. paralyzes the smooth muscle of blood vessels. c. decreases sympathetic nervous system activity. d. prevents the movement of calcium into the cardiac cells.

Correct Answer: C Rationale: Labetalol decreases sympathetic nervous system activity by blocking both α- and β-receptors, leading to vasodilation and a decrease in heart rate, which lower BP. The angiotensin-converting enzyme (ACE)-inhibitors and angiotensin II blocking medications block the RAAS. Direct vasodilator medications inhibit the ability of vascular smooth muscle to contract and cause vasoconstriction. Calcium-channel blockers block the movement of calcium into cardiac muscle cells and lead to a decrease in heart rate and contractility, which will result in lower cardiac output.

While assessing a patient who has just arrived in the emergency department, the nurse notes a pulse deficit. The nurse will anticipate that the patient may require a. hourly blood pressure (BP) checks. b. a coronary arteriogram. c. electrocardiographic (ECG) monitoring. d. a 2-D echocardiogram.

Correct Answer: C Rationale: Pulse deficit is a difference between simultaneously obtained apical and radial pulses and indicates that there may be cardiac dysrhythmias that would be detected with ECG monitoring. Frequent BP monitoring, coronary arteriograms, and echocardiograms are used for diagnosis of other cardiovascular disorders but would not be as helpful in determining the reason for the pulse deficit. Cognitive Level: Application Text Reference: pp. 750-751 Nursing Process: Assessment NCLEX: Physiological Integrity

The nurse is obtaining a health history for a new patient with possible coronary artery disease. Which question would the nurse use when obtaining subjective data related to the patient's health perception-health management functional health pattern? a. "Do you every have any discomfort or indigestion resulting from exercise or activity?" b. "Have you had any recent episodes of sore throat, fever, or streptococcal infections?" c. "How frequently do you have your cholesterol level and blood pressure checked?" d. "Are there any symptoms that seem to occur when you are feeling very stressed?"

Correct Answer: C Rationale: The health perception-health management functional pattern includes information related to what the patient knows about coronary heart disease risk factors and actions the patient is taking to decrease risk. Any patient history of streptococcal infections or sore throat would also be included in this functional pattern, but this patient has possible coronary artery disease, not rheumatic heart disease. Information about discomfort caused by activity would be included in the activity-exercise pattern. The data about symptoms in response to stress would be documented in the coping-stress tolerance functional pattern. Cognitive Level: Application Text Reference: pp. 745-746 Nursing Process: Assessment NCLEX: Health Promotion and Maintenanc

During assessment of a patient with newly diagnosed stage 1 hypertension, the nurse finds that the patient uses a lot of salt on foods and is 30 pounds overweight. The patient states, "I thought high blood pressure was caused by stress, but I do not feel stressed at all." An appropriate nursing diagnosis for this patient is a. noncompliance related to lack of motivation and poor coping skills. b. situational low self-esteem related to new diagnosis of hypertension. c. ineffective health maintenance related to lack of knowledge about risk factors for hypertension d. ineffective denial related to complexity of management regimen and the associated lifestyle changes.

Correct Answer: C Rationale: This patient's subjective and objective assessment data indicate that lack of knowledge about hypertension will need to be addressed to allow the patient to improve the BP. There is no evidence for noncompliance, lack of motivation, or poor coping skills. The patient's statements do not indicate low self-esteem or denial. Cognitive Level: Application Text Reference: p. 778 Nursing Process: Diagnosis NCLEX: Health Promotion and Maintenance

While caring for a patient admitted with a hypertensive emergency and receiving sodium nitroprusside (Nipride), it will be essential for the nurse to a. insert an arterial line to obtain BP to ensure accurate BP measurements. b. assist the patient up in the chair for meals to avoid complications associated with immobility. c. titrate the rate of IV nitroprusside to avoid too-rapid reduction of BP to normal levels. d. place the patient on NPO status to prevent aspiration caused by nausea and the associated vomiting.

Correct Answer: C Rationale: When initiating treatment with rapidly acting medications such as sodium nitroprusside, the BP should not be decreased more than 25 % in the first 2 hours to prevent a sudden drop in cerebral perfusion. Frequent BP monitoring is required when treating hypertensive emergencies, an arterial line is often used; however, BP can also be monitored by using an automatic noninvasive BP device. Additionally, only a health care provider or an advanced practice nurse would insert an arterial line. When patients are receiving IV vasodilators, the BP is unable to adjust to changes in position, so the patients should remain on bed rest. There is no indication that this patient is nauseated or at risk for aspiration, so an NPO status is not appropriate. Cognitive Level: Application Text Reference: pp. 781-782 Nursing Process: Implementation NCLEX: Physiological Integrity

1. When assessing a 76-year-old woman, the nurse finds the following results: BP 146/102, resting HR 104, slightly irregular S4 heart sound, and a grade I/VI aortic systolic murmur. The nurse recognizes that common effects of aging may be responsible for the (Select all that apply.) a. HR. b. irregular pulse. c. S4 heart sound. d. systolic BP. e. diastolic BP. f. grade I/VI aortic systolic murmur.

Correct Answer: C, D, F Rationale: An S4 gallop, increased systolic BP, and aortic stenosis are associated with aging, although all these findings require further assessment or intervention. Increases in HR, irregular heart rhythms, and diastolic BP increases are not associated with increased age. Cognitive Level: Comprehension Text Reference: p. 744 Nursing Process: Assessment NCLEX: Physiological Integrity

A 52-year-old patient has no history of hypertension and no risk factors related to hypertension. During an annual physical examination, the BP is 188/106. After reconfirming the BP, it is appropriate for the nurse to tell the patient that a. a BP recheck should be scheduled in 2 months. b. there is an imminent danger of a stroke and immediate hospitalization is indicated. c. the dietary sodium and fat content should be decreased. d. more diagnostic testing may be needed to determine the cause of the hypertension

Correct Answer: D Rationale: A sudden increase in BP in a patient with no previous hypertension history or risk factors indicates that the hypertension may be secondary to some other problem. The BP will require more frequent monitoring than every 2 months. If the patient has no other risk factors, a stroke in the immediate future is unlikely. There is no indication that dietary salt or fat intake have contributed to this sudden increase in BP, and reducing intake of salt and fat alone will not be adequate to reduce this BP to an acceptable level. Cognitive Level: Application Text Reference: p. 765 Nursing Process: Implementation NCLEX: Health Promotion and Maintenance

During assessment of a patient who has stage 2 hypertension, the nurse recognizes that it is common for the patient to experience a. frequent nose bleeds. b. blurred vision. c. dyspnea on exertion. d. no symptoms.

Correct Answer: D Rationale: Hypertension is largely asymptomatic until damage to target organs has occurred. Frequent nosebleeds are not a common symptom of hypertension. Blurred vision and dyspnea on exertion indicate complications associated with damage to the retina and heart failure caused by chronic hypertension, but they are not common symptoms of hypertension. Cognitive Level: Comprehension Text Reference: pp. 766, 779 Nursing Process: Assessment NCLEX: Physiological Integrity

Which information obtained by the nurse who is admitting the patient for magnetic resonance imaging (MRI) will be most important to report to the health care provider before the MRI? a. The patient has a history of coronary artery disease. b. The patient took all the prescribed cardiac medications today. c. The patient has an allergy to shellfish and iodine. d. The patient has a permanent ventricular pacemaker in place.

Correct Answer: D Rationale: MRI is contraindicated for patients with implanted metallic devices such as pacemakers. The other information will also be reported to the health care provider but does not impact on whether or not the patient can have an MRI. Cognitive Level: Application Text Reference: p. 755 Nursing Process: Implementation NCLEX: Physiological Integrity

When reading the medical history on a patient's chart, the nurse notes that the patient has pseudohypertension. When assessing the patient, the nurse will anticipate a. an elevated SBP with a normal DBP. b. an increase in BP when the patient is stressed. c. that the patient may also be diabetic or have insulin resistance. d. that the patient may have few symptoms of target-organ damage.

Correct Answer: D Rationale: Pseudohypertension is caused by sclerosis of the larger arteries, resulting in BP readings that are falsely elevated. One indication of pseudohypertension is a high BP without any of the target-organ damage that would be expected. An elevated SBP with a normal DBP is common in older adults but does not define pseudohypertension. A BP increase with stress or associated with diabetes or insulin resistance may occur but is not defined as pseudohypertension. Cognitive Level: Application Text Reference: p. 765 Nursing Process: Assessment NCLEX: Health Promotion and Maintenance

While assessing a patient with heart failure, the nurse notes that the patient has jugular venous distension (JVD) when lying flat in bed. The nurse's next action will be to a. have the patient perform the Valsalva maneuver and observe the jugular veins. b. palpate the jugular veins and compare the volume and pressure on the both sides. c. use a centimeter ruler to measure and document accurately the level of the JVD. d. elevate the patient gradually to an upright position and examine for continued JVD.

Correct Answer: D Rationale: When assessing for and documenting JVD, the nurse should document the angle at which the patient is positioned. When the patient is lying flat, the jugular veins are at the level of the right atrium, so JVD is a common (but not clinically significant) finding. JVD that persists when the patient is sitting at a 30- to 45-degree angle or greater is significant. The nurse may use a ruler to determine the level of JVD above the heart if the JVD persists when the patient is at 30 to 45 degree angle or more. JVD is an expected finding when a patient performs the Valsalva maneuver because right atrial pressure increases. Comparison of the volume and pressure of the jugular veins is not included in jugular vein assessment. Cognitive Level: Application Text Reference: pp. 748-749 Nursing Process: Assessment NCLEX: Physiological Integrity

9. A 72-year-old patient is hospitalized for an aortic dissection of the abdominal aorta that stabilizes with treatment. The nurse develops a teaching plan for the patient's discharge that includes information about a. gradually increasing exercise to improve cardiac function and BP control. b. appropriate use of nonsteroidal antiinflammatory agents (NSAIDs) to control any abdominal pain. c. holding prescribed -blockers if dizziness or weakness occur to avoid injury. d. the use of antihypertensive medications to lower the risk of further dissection or bleeding.

D Rationale: Antihypertensive medications are prescribed to help control BP and prevent re-dissection, leaking, or rupture. Exercise will increase the BP and increase the risk for further dissection. NSAIDs decrease platelet function, and the patient should avoid use of this category of medications; in addition, the patient is taught to call the health care provider about any abdominal pain. The patient may experience some side effects of -blockade but should discuss any change in dose or medication with the health care provider. Cognitive Level: Application Text Reference: pp. 899-900 Nursing Process: Planning NCLEX: Physiological Integrity

23. A hospitalized patient with heart failure has a new order for captopril (Capoten) 12.5 mg PO. After administering the first dose and teaching the patient about captopril, which statement by the patient indicates that teaching has been effective? a. "I will need to include more high-potassium foods in my diet." b. "I will expect to feel more short of breath for the next few days." c. "I will be sure to take the medication after eating something." d. "I will call for help when I need to get up to the bathroom."

D Rationale: Captopril can cause hypotension, especially after the initial dose, so it is important that the patient not get up out of bed without assistance until the nurse has had a chance to evaluate the effect of the first dose. The ACE inhibitors are potassium sparring, and the nurse should not teach the patient to increase sources of dietary potassium. Increased shortness of breath is expected with initiation of -blocker therapy for heart failure, not for ACE-inhibitor therapy. ACE inhibitors are best absorbed when taken an hour before eating. Cognitive Level: Application Text Reference: p. 832 Nursing Process: Evaluation NCLEX: Physiological Integrity

22. A 42-year-old service-counter worker undergoes sclerotherapy for treatment of superficial varicose veins at an outpatient center. Before discharging the patient, the nurse teaches the patient that a. exercises such as walking or jogging cause recurrence of varicosities. b. sitting at the work counter, rather than standing, is recommended. c. taking one aspirin daily will help prevent clotting around venous valves. d. compression stockings should be applied before getting out of bed.

D Rationale: Compression stockings are applied with the legs elevated to reduce pressure in the lower legs. Walking is recommended to prevent recurrent varicosities. Sitting and standing are both risk factors for varicose veins and venous insufficiency. An aspirin a day is not adequate to prevent venous thrombosis and would not be recommended to the patient who had just had sclerotherapy. Cognitive Level: Application Text Reference: pp. 918-919 Nursing Process: Planning NCLEX: Health Promotion and Maintenance

29. While working in the outpatient clinic, the nurse notes that the chart states that a patient has intermittent claudication. Which of these statements by the patient would be consistent with this information? a. "My fingers hurt when I go outside in cold weather." b. "Sometimes I get tired when I climb a lot of stairs." c. "When I stand too long, my feet start to swell up." d. "My legs cramp whenever I walk more than a block."

D Rationale: Cramping that is precipitated by a consistent level of exercise is descriptive of intermittent claudication. Finger pain associated with cold weather is typical of Raynaud's phenomenon. Fatigue that occurs sometimes with exercise is not typical of intermittent claudication, which is reproducible. Swelling associated with prolonged standing is typical of venous disease. Cognitive Level: Application Text Reference: p. 900 Nursing Process: Assessment NCLEX: Physiological Integrity

13. The nurse plans discharge teaching for a patient with chronic heart failure who has prescriptions for digoxin (Lanoxin), hydrochlorothiazide (HydroDIURIL), and a potassium supplement. Appropriate instructions for the patient include a. avoid dietary sources of potassium because too much can cause digitalis toxicity. b. take the pulse rate daily and never take digoxin if the pulse is below 60 beats/min. c. take the hydrochlorothiazide before bedtime to maximize activity level during the day. d. notify the health care provider immediately if nausea or difficulty breathing occurs.

D Rationale: Difficulty breathing is an indication of acute decompensated heart failure and suggests that the medications are not achieving the desired effect. Nausea is an indication of digoxin toxicity and should be reported so that the provider can assess the patient for toxicity and adjust the digoxin dose, if necessary. Digoxin toxicity is potentiated by hypokalemia, rather than hyperkalemia. Patients should be taught to check their pulse daily before taking the digoxin and, if the pulse is less than 60, to call their provider before taking the digoxin. Diuretics should be taken early in the day to avoid sleep disruption. Cognitive Level: Application Text Reference: p. 835 Nursing Process: Implementation NCLEX: Health Promotion and Maintenance

18. A 55-year-old patient with inoperable coronary artery disease and end-stage heart failure asks the nurse whether heart transplant is a possible therapy. The nurse's response to the patient will be based on the knowledge that a. heart transplants are experimental surgeries that are not covered by most insurance. b. the patient is too old to be placed on the transplant list. c. the diagnoses and symptoms indicate that the patient is not an appropriate candidate. d. candidacy for heart transplant depends on many factors.

D Rationale: Indications for a heart transplant include inoperable coronary artery disease and refractory end-stage heart failure, but other factors such as coping skills, family support, and patient motivation to follow the rigorous post-transplant regimen are also considered. Heart transplants are not considered experimental; rather, transplantation has become the treatment of choice for patients who meet the criteria. The patient is not too old for a transplant. The patient's diagnoses and symptoms indicate that the patient may be an appropriate candidate for a heart transplant. Cognitive Level: Comprehension Text Reference: p. 837 Nursing Process: Planning NCLEX: Health Promotion and Maintenance

19. While caring for a patient with mitral valve prolapse with mild valvular regurgitation, the nurse determines that discharge teaching has been effective when the patient tells the nurse she will a. take 1 aspirin a day to prevent embolization from the valve. b. limit physical activity to avoid stressing the heart valves. c. schedule an appointment with the doctor every 6 months. d. discuss the diagnosis of mitral valve prolapse with the dentist.

D Rationale: Mitral valve prolapse with regurgitation is a risk factor for infective endocarditis, and the patient needs to discuss the need for antibiotic prophylaxis with the provider before any invasive medical or dental procedure is done. Anticoagulation, restriction of physical activity, and frequent medical appointments are not required for mild mitral valve prolapse. Cognitive Level: Application Text Reference: p. 880 Nursing Process: Evaluation NCLEX: Health Promotion and Maintenance

12. While obtaining an admission health history from a patient with possible rheumatic fever, which question will be most pertinent to ask? a. "Are you using any illegal IV drugs?" b. "Do you have any family history of congenital heart disease?" c. "Can you recall having any chest injuries in the last few weeks?" d. "Have you had a recent sore throat?"

D Rationale: Rheumatic fever occurs as a result of an abnormal immune response to a streptococcal infection. Although illicit IV drug use should be discussed with the patient before discharge, it is not a risk factor for rheumatic fever and would not be as pertinent when admitting the patient. Family history is not a risk factor for rheumatic fever. Chest injury would cause musculoskeletal chest pain rather than rheumatic fever. Cognitive Level: Application Text Reference: p. 875 Nursing Process: Assessment NCLEX: Physiological Integrity

28. A patient who is seen in the clinic tells the health care provider about experiencing cold, numb fingers when running during the winter and is diagnosed with Raynaud's phenomenon. The nurse will anticipate teaching the patient about tests for a. coronary artery disease. b. familial hyperlipidemia. c. high BP. d. immune disorders.

D Rationale: Secondary Raynaud's phenomenon may occur in conjunction with autoimmune diseases such as rheumatoid arthritis, and patients should be screened for autoimmune disorders. Raynaud's phenomenon is not associated with hyperlipidemia, hypertension, or coronary artery disease. Cognitive Level: Application Text Reference: p. 909 Nursing Process: Planning NCLEX: Physiological Integrity

11. During an assessment of a 63-year-old patient at the clinic, the patient says, "I have always taken an evening walk, but lately my leg cramps and hurts after just a few minutes of walking. The pain goes away after I stop walking, though." The nurse should a. ask about any skin color changes that occur in response to cold. b. check for the presence of tortuous veins bilaterally on the legs. c. assess for unilateral swelling, redness, and tenderness of either leg. d. attempt to palpate the dorsalis pedis and posterial tibial pulses.

D Rationale: The nurse should assess for other clinical manifestations of peripheral arterial disease in a patient who describes intermittent claudication. Changes in skin color that occur in response to cold are consistent with Raynaud's phenomenon. Tortuous veins on the legs suggest venous insufficiency. Unilateral leg swelling, redness, and tenderness point to deep vein thrombosis (DVT). Cognitive Level: Application Text Reference: pp. 900-901 Nursing Process: Assessment NCLEX: Physiological Integrity

4. Several hours following a surgical repair of an abdominal aortic aneurysm, the patient develops left flank pain and a urinary output of 20 ml/hr for 2 hours. The nurse notifies the health care provider and anticipates orders for a(n) a. additional antibiotic. b. increase in IV rate. c. complete blood count. d. blood urea nitrogen (BUN) and creatinine.

D Rationale: The pain and decreased urine output suggest a renal artery embolism, and monitoring of renal function is needed. The data are not consistent with the complications of infection, hypovolemia, or bleeding. Cognitive Level: Application Text Reference: p. 898 Nursing Process: Assessment NCLEX: Physiological Integrity

17. A patient admitted to the hospital with DVT has health care provider's orders for bed rest with the feet elevated. The best method for the nurse to use in elevating the patient's feet is to a. place two pillows under the calf of the affected leg. b. place the patient in the Trendelenburg position. c. elevate the bed at the knee and put pillows under the feet. d. put one pillow under the thighs and two pillows under the lower legs.

D Rationale: The purpose of elevation of the feet is to enhance venous flow from the feet to the right atrium, which is best accomplished by placing two pillows under the feet and one under the thighs. Placing the patient in the Trendelenburg position will lower the head below heart level, which is not indicated for this patient. Placing pillows under the calf or elevating the bed at the knee may cause blood stasis at the calf level. Cognitive Level: Application Text Reference: p. 916 Nursing Process: Implementation NCLEX: Physiological Integrity

Which information given by a patient admitted with chronic stable angina will help the nurse confirm this diagnosis? A) a. The patient rates the pain at a level 3 to 5 (0 to 10 scale). B) b. The patient states that the pain "wakes me up at night." C) c. The patient says that the frequency of the pain has increased over the last few weeks. D) d. The patient states that the pain is resolved after taking one sublingual nitroglycerin tablet.

D The patient states that the pain is resolved after taking one sublingual nitroglycerin tablet.

During a physical examination of a patient, the nurse palpates the point of maximal impulse (PMI) in the sixth intercostal space lateral to the left midclavicular line. The most appropriate action for the nurse to take next will be to A) a. document that the PMI is in the normal anatomic location. B) b. ask the patient about risk factors for coronary artery disease. C) c. auscultate both the carotid arteries for the presence of a bruit. D) d. assess the patient for symptoms of left ventricular hypertrophy.

D assess the patient for symptoms of left ventricular hypertrophy.

After noting a pulse deficit when assessing a patient who has just arrived in the emergency department, the nurse will anticipate that the patient may require A) a. a 2-D echocardiogram. B) b. a cardiac catheterization. C) c. hourly blood pressure (BP) checks. D) d. electrocardiographic (ECG) monitoring.

D electrocardiographic (ECG) monitoring.

When developing a health teaching plan for a 60-year-old man with the following risk factors for coronary artery disease (CAD), the nurse should focus on the A) a. family history of coronary artery disease. B) b. increased risk associated with the patient's gender. C) c. high incidence of cardiovascular disease in older people. D) d. elevation of the patient's serum low density lipoprotein (LDL) level.

D elevation of the patient's serum low density lipoprotein (LDL) level.

Nadolol (Corgard) is prescribed for a patient with angina. To determine whether the drug is effective, the nurse will monitor for A) a. decreased blood pressure and apical pulse rate. B) b. fewer complaints of having cold hands and feet. C) c. improvement in the quality of the peripheral pulses. D) d. the ability to do daily activities without chest discomfort.

D the ability to do daily activities without chest discomfort.

The nurse will suspect that the patient with stable angina is experiencing a side effect of the prescribed metoprolol (Lopressor) if A) a. the patient is restless and agitated. B) b. the blood pressure is 190/110 mm Hg. C) c. the patient complains about feeling anxious. D) d. the cardiac monitor shows a heart rate of 45.

D the cardiac monitor shows a heart rate of 45.

After the nurse teaches a patient with chronic stable angina about how to use the prescribed short-acting and long-acting nitrates, which statement by the patient indicates that the teaching has been effective? A) a. "I will put on the nitroglycerin patch as soon as I develop any chest pain." B) b. "I will check the pulse rate in my wrist just before I take any nitroglycerin." C) c. "I will be sure to remove the nitroglycerin patch before using any sublingual nitroglycerin." D) d. "I will stop what I am doing and sit down before I put the nitroglycerin under my tongue."

D) d. "I will stop what I am doing and sit down before I put the nitroglycerin under my tongue."

Which action should the nurse take when preparing for cardioversion of a patient with supraventricular tachycardia who is alert and has a blood pressure of 110/66 mm Hg? A) a. Turn the synchronizer switch to the "off" position. B) b. Perform cardiopulmonary resuscitation (CPR) until the paddles are in correct position. C) c. Set the defibrillator/cardioverter energy to 300 joules. D) d. Administer a sedative before cardioversion is implemented.

D) d. Administer a sedative before cardioversion is implemented.

. A patient who has begun to awaken after 30 minutes in the postanesthesia care unit (PACU) is restless and shouting at the nurse. The patient's oxygen saturation is 99%, and recent lab results are all normal. Which action by the nurse is most appropriate? A) a. Insert an oral or nasal airway. B) b. Notify the anesthesia care provider. C) c. Orient the patient to time, place, and person. D) d. Be sure that the patient's IV lines are secure.

D) d. Be sure that the patient's IV lines are secure.

Which topic will the nurse plan to include in discharge teaching for a patient with systolic heart failure and an ejection fraction of 38%? A) a. Need to participate in an aerobic exercise program several times weekly B) b. Use of salt substitutes to replace table salt when cooking and at the table C) c. Importance of making a yearly appointment with the primary care provider D) d. Benefits and side effects of angiotensin-converting enzyme (ACE) inhibitors

D) d. Benefits and side effects of angiotensin-converting enzyme (ACE) inhibitors

A patient's cardiac monitor shows sinus rhythm, rate 60 to 70. The P-R interval is 0.18 seconds at 1:00 AM, 0.20 seconds at 2:30 PM, and 0.23 seconds at 4:00 PM. Which action should the nurse take at this time? A) a. Prepare for possible temporary pacemaker insertion. B) b. Administer atropine sulfate 1 mg IV per agency protocol. C) c. Document the patient's rhythm and assess the patient's response to the rhythm. D) d. Call the health care provider before giving the prescribed metoprolol (Lopressor).

D) d. Call the health care provider before giving the prescribed metoprolol (Lopressor).

When analyzing the waveforms of a patient's electrocardiogram (ECG), the nurse will need to investigate further upon finding a A) a. T wave of 0.16 second. B) b. P-R interval of 0.18 second. C) c. Q-T interval of 0.34 second. D) d. QRS interval of 0.14 second.

D) d. QRS interval of 0.14 second.

Which laboratory result for a patient whose cardiac monitor shows multifocal premature ventricular contractions (PVCs) is most important for the nurse to communicate to the health care provider? A) a. Blood glucose 228 mg/dL B) b. Serum chloride 90 mEq/L C) c. Serum sodium 133 mEq/L D) d. Serum potassium 2.8 mEq/L

D) d. Serum potassium 2.8 mEq/L

A patient has a normal cardiac rhythm and a heart rate of 72 beats/minute, except that the PR interval is 0.24 seconds. The appropriate intervention by the nurse is to A) a. notify the patient's health care provider immediately. B) b. administer atropine per agency bradycardia protocol. C) c. prepare the patient for temporary pacemaker insertion. D) d. document the finding and continue to monitor the patient.

D) d. document the finding and continue to monitor the patient.

The perioperative nurse encourages a family member or a friend to remain with a patient in the preoperative holding area until the patient is taken into the operating room primarily to A) a. ensure the proper identification of the patient before surgery. B) b. protect the patient from cross-contamination with other patients. C) c. assist the perioperative nurse to obtain a complete patient history. D) d. help relieve the stress of separation for the patient and significant others.

D) d. help relieve the stress of separation for the patient and significant others.

A patient who has chronic heart failure tells the nurse, "I felt fine when I went to bed, but I woke up in the middle of the night feeling like I was suffocating!" The nurse will document this assessment information as A) a. pulsus alternans. B) b. two-pillow orthopnea. C) c. acute bilateral pleural effusion. D) d. paroxysmal nocturnal dyspnea.

D) d. paroxysmal nocturnal dyspnea.

The nurse obtains a monitor strip on a patient who has had a myocardial infarction and makes the following analysis: P wave not apparent, ventricular rate 162, R-R interval regular, P-R interval not measurable, and QRS complex wide and distorted, QRS duration 0.18 second. The nurse interprets the patient's cardiac rhythm as A) a. atrial fibrillation. B) b. sinus tachycardia. C) c. ventricular fibrillation. D) d. ventricular tachycardia.

D) d. ventricular tachycardia.

A patient in surgery receives a neuromuscular blocking agent as an adjunct to general anesthesia. At completion of the surgery, it is most important that the nurse monitor the patient for A) a. nausea. B) b. confusion. C) c. bronchospasm. D) d. weak chest-wall movement.

D) d. weak chest-wall movement.

The nurse has just finished teaching a hypertensive patient about the newly prescribed quinapril (Accupril). Which patient statement indicates that more teaching is needed? A) a. "The medication may not work as well if I take any aspirin." B) b. "The doctor may order a blood potassium level occasionally." C) c. "I will call the doctor if I notice that I have a frequent cough." D) d. "I won't worry if I have a little swelling around my lips and face."

D. "I won't worry if I have a little swelling around my lips and face."

A patient has just been diagnosed with hypertension and has a new prescription for captopril (Capoten). Which information is important to include when teaching the patient? A) a. Check BP daily before taking the medication. B) b. Increase fluid intake if dryness of the mouth is a problem. C) c. Include high-potassium foods such as bananas in the diet. D) d. Change position slowly to help prevent dizziness and falls.

D. Change position slowly to help prevent dizziness and falls.

After the nurse teaches the patient with stage 1 hypertension about diet modifications that should be implemented, which diet choice indicates that the teaching has been effective? A) a. The patient avoids eating nuts or nut butters. B) b. The patient restricts intake of dietary protein. C) c. The patient has only one cup of coffee in the morning. D) d. The patient has a glass of low-fat milk with each meal.

D. The patient has a glass of low-fat milk with each meal.

1) Which of the following is not found in acute decompensated heart failure? A. ORTHOPNEA B. TACHYCARDIA C. DYSPNEA D. UNPRODUCTIVE COUGH

D. UNPRODUCTIVE COUGH

A 52-year-old patient who has no previous history of hypertension or other health problems suddenly develops a BP of 188/106 mm Hg. After reconfirming the BP, it is appropriate for the nurse to tell the patient that A) a. a BP recheck should be scheduled in a few weeks. B) b. the dietary sodium and fat content should be decreased. C) c. there is an immediate danger of a stroke and hospitalization will be required. D) d. more diagnostic testing may be needed to determine the cause of the hypertension.

D. more diagnostic testing may be needed to determine the cause of the hypertension.

10) A patient with potential heart failure enters the emergency room. What symptom should the nurse not consider for heart failure? A. cyanosis, cold and clammy skin B. lung sounds-- crackling and wheezing C. orthopnea, shortness of breath D. tightness & burning from the chest

D. tightness & burning from the chest

You are called to the local airport to evaluate a 40-year-old obese woman who is complaining of pain in her left lower leg. She has just completed a 12-hour flight, and the pain developed as she got off the plane. Her leg is warm, swollen, and painful. You suspect:

Deep-vein thrombosis

impaired skin integrity related to peripheral edema.

During a visit to an elderly patient with chronic heart failure, the home care nurse finds that the patient has severe dependent edema and that the legs appear to be weeping serous fluid. Based on these data, the best nursing diagnosis for the patient is _________________

attempt to palpate the dorsalis pedis and posterial tibial pulses.

During an assessment of a 63-year-old patient at the clinic, the patient says, "I have always taken an evening walk, but lately my leg cramps and hurts after just a few minutes of walking. The pain goes away after I stop walking, though." The nurse should _____________

elevated right atrial pressure.

During assessment of a 72-year-old with ankle swelling, the nurse notes jugular venous distention (JVD) with the head of the patient's bed elevated 45 degrees. The nurse knows this finding indicates _____________

apical radial heart rate.

During change-of-shift report, the nurse learns that a patient with a large myocardial infarction has been having frequent PVCs. When monitoring the patient for the effects of PVCs, the nurse will check the patient's _____________

how to monitor anticoagulation therapy.

During postoperative teaching with a patient who had a mitral valve replacement with a mechanical valve, the nurse instructs the patient regarding ______________

a decrease in level of consciousness.

During the administration of the fibrinolytic agent to a patient with an AMI, the nurse should stop the drug infusion if the patient experiences _______________

dyspnea and a dry, hacking cough.

During the assessment of a patient with IE, the nurse would expect to find _________

Diagnostic tests for CAD/Angina

EKG (shows ischemia), cardiac catheterization (most definitive), blood lipid level, cardiac enzymes, stress test, holter monitor, echocardiogram, TEE, MUGA (ejection fraction)

A compensatory mechanism of the heart in the presence of chronic hypertension is to:

Enlarge the muscle mass of the heart

Three layers of the heart

Epicardium - outermost layer Myocardium - middle layer - muscle of heart Endocardium - innermost layer - lines inner chambers of heart and valves

Heart rate increases from 66 to 90 beats/min.

Following an AMI, a patient ambulates in the hospital hallway. When the nurse is evaluating the patient's response, which of these assessment data would indicate that the exercise level should be decreased?

angiotensin-converting enzyme (ACE) inhibitors, such as captopril (Capoten).

Following an acute myocardial infarction, a previously healthy 67-year-old patient develops clinical manifestations of heart failure. The nurse anticipates discharge teaching will include information about _______________

43. Immediate treatment for a patient with an acute myocardial infarction involves: a. Elevating the patient's legs 6 to 12 inches b. Reducing myocardial oxygen demand c. Giving oxygen via nonrebreathing mask d. Administering up to 3 doses of nitroglycerin

Giving oxygen via nonrebreathing mask

While assessing a patient you identify a carotid bruit. This leads you to believe that the patient:

Has atherosclerosis

identifying the need to call the emergency medical services (EMS) if chest pain persists 5 minutes after taking nitroglycerin.

In developing a teaching plan for a patient who has stable angina and is started on sublingual nitroglycerin (Nitrostat), the nurse identifies an expected patient outcome of

5. Hypocalcemia and hypomagnesemia would MOST likely result in: a. Decreased cardiac conduction b. Increased myocardial irritability c. A decrease in cardiac contractility d. Decreased myocardial automaticity

Increased myocardial irritability

An undesirable side effect of atropine is:

Increased myocardial oxygen demand

1. Which of the following is NOT a goal for decreasing the risk of cardiovascular disease? a. Increasing aerobic exercise b. Controlling high blood pressure c. Increasing LDL cholesterol level d. Increasing HDL cholesterol level

Increasing LDL cholesterol level

26. Jugular venous distension in a patient sitting at a 45 degree angle a. Is not clinically significant b. Is a sign of reduced preload c. Suggests left side heart failure d. Indicates right heart compromise

Indicates right heart compromise

Chest pain associated with MI:

Is constant

Defibrillation of patients in asystole

Is not recommended

3 Grades of Myocardial Insult

Ischemia - not enough O2 Injury - enzymes released Infarction - permanent irreversible damage

The organ(s) most at risk in a hypertensive crisis include the:

Kidneys

You are treating a patient with blood pressure of 200 over 140. The patient initially complained of headache and nausea. During your 3-hour transport, the patient began to seize and is now unresponsive to any stimulus. ment for this condition includes:

Labetalol

2. The point of maximal impulse (PMI) usually can be felt on the: a. Medial aspect of the chest, just below the third intercostal space b. Left lateral chest, in the midaxillary line, at the fourth intercostal space c. Left anterior chest, in the midaxillary line, at the fifth intercostal space d. Left anterior chest, in the midclavicular line, at the fifth intercostal space

Left anterior chest, in the midclavicular line, at the fifth intercostal space

An inferior-wall MI is usually caused by occlusion of the _____ artery.

Left anterior descending

The majority of acute myocardial infarctions involve the:

Left ventricle

If you see the outline of a small box implanted under skin in the left upper abdomen, you would suspect the patient has a(n):

Left-ventricular assist device

35. Which of the following statements MOST accurately describes an acute myocardial infarction (AMI)? a. Death of the myocardium secondary to the spasm of a major coronary artery b. Injury to a portion of the heart muscle secondary to atherosclerotic disease c. Damage to the left ventricle following occlusion of the left coronary artery d. Necrosis of a portion of the myocardium due to a prolonged lack of oxygen

Necrosis of a portion of the myocardium due to a prolonged lack of oxygen

decrease spasm of the coronary arteries.

Nifedipine (Procardia) is ordered for a patient with newly diagnosed Prinzmetal's (variant) angina. When teaching the patient, the nurse will include the information that Procardia will ____________

12. Cardiac-related chest pain often palliated by: a. Stress b. Exertion c. Nitroglycerin d. Mild exercise

Nitroglycerin

44. What is the MOST correct sequence of treatment for a patient with a suspected acute myocardial infarction? a. Oxygen, aspirin, nitroglycerin, morphine b. Oxygen, nitroglycerin, aspirin, morphine c. Aspirin, nitroglycerin, oxygen, morphine d. Morphine, oxygen, aspirin, nitroglycerin

Oxygen, aspirin, nitroglycerin, morphine

http://o.quizlet.com/CgjGGV.bVtB9R23arMzrBg_m.png

PVC

While assessing a patient, you note a pulsatile mass in the abdomen. Suddenly this mass is no longer palpable, and the patient's blood pressure begins to drop. You suspect that the:

Patient's aneurysm has ruptured

18. A patient with a medical condition that requires antiplatelet therapy would MOST likely be taking: a. Coreg b. Altace c. Zocor d. Plavix

Plavix

39. The pain associated with an acute myocardial infarction: a. Radiates to the left or right arm in majority of cases b. Is not influenced by deep breathing or body movement c. Is most often described as a sharp sensation in the chest d. Is often relieved by 2 or 3 doses of sublingual nitroglycerin

Radiates to the left or right arm in majority of cases

Baroreceptors

Specialized nerve endings located in the walls of the aortic arch and carotid sinuses. They are affected by changes in blood pressure. Increases in BP stimulate baroreceptors and heart rate and BP decrease. Decrease in BP leads to less stimulation of baroreceptors, vasoconstriction occurs and heart rate increases.

A blood pressure reading in an adult of 180/110 is considered:

Stage 3 hypertension

47. Which of the following situations would contraindicate the administration of nitroglycerin? a. Hypersensitivity to salicylates b. Systolic BP less than 90 mm Hg c. Levitra use within the last 72 hours d. Use of Plavix within the last 12 hours

Systolic BP less than 90 mm Hg

10. If you are unsure whether a conscious cardiac patient requires immediate transport versus continued assessment at the scene, you should: a. Continue with the focused assessment to obtain more information b. Take the patient's vital signs and then make your transport decision c. Begin immediate transport and perform a focused assessment en route d. Contact medical control and inform him or her of the patient's condition

Take the patient's vital signs and then make your transport decision

37. Cardiac arrhythmias following an acute myocardial infarction: a. Tend to originate from ischemic areas around the infarction b. Typically manifest as atrial fibrillation or atrial tachycardia c. Generally originate from the center of the infarcted tissues d. Are uncommon within the first 24 hours after the infarction

Tend to originate from ischemic areas around the infarction

Afterload

The force against which the heart has to pump to eject blood from the left ventricle.

avoid any IM medications to prevent localized bleeding.

The health care provider orders a continuous IV heparin infusion for a patient with swelling and pain of the upper leg caused by a DVT. While the patient is receiving the heparin infusion, the nurse should _____________

Contractility

The inherent ability of the myocardium to alter contractile force and velocity. Decreased contractility ---> decreased stroke volume ---> decreased cardiac output

every other QRS complex is wide and starts prematurely.

The nurse determines that a patient has ventricular bigeminy when the rhythm strip indicates that ________________

"I will be immune to further episodes of rheumatic fever after this infection."

The nurse establishes the nursing diagnosis of ineffective therapeutic regimen management related to lack of knowledge concerning long-term management of rheumatic fever when a patient recovering from rheumatic fever says, _______________

position the patient in Fowler's position, leaning forward on the overbed table.

The nurse has identified a nursing diagnosis of acute pain related to inflammatory process for a patient with acute pericarditis. The most appropriate intervention by the nurse for this problem is to ______________

Maintain bed rest as ordered.

The nurse has identified the collaborative problem of potential complication: pulmonary embolism for a patient with left-calf DVT. Which nursing action is appropriate to include in the plan of care?

"I should change my diet to include more green, leafy vegetables."

The nurse has initiated discharge teaching for a patient who is to be maintained on warfarin (Coumadin) following hospitalization for DVT. The nurse determines that additional teaching is needed when the patient says, _________________ __

A 51-year-old who has just returned to the unit after a coronary arteriogram and PCI

The nurse has just received change-of-shift report about these four patients. Which patient should the nurse assess first?

urine output less than 30 ml/hr.

The nurse identifies the nursing diagnosis of decreased cardiac output related to valvular insufficiency for the patient with IE based on the assessment finding of _____________

"I will use a heating pad on my feet at night to increase the circulation and warmth in my feet."

The nurse identifies the nursing diagnosis of ineffective peripheral perfusion related to decreased arterial blood flow for a patient with chronic PAD. In evaluating the patient outcomes following patient teaching, the nurse determines a need for further instruction when the patient says, ______________

The pain has persisted longer than 30 minutes.

The nurse is admitting a patient who is complaining of chest pain to the emergency department (ED). Which information collected by the nurse suggests that the pain is caused by an acute myocardial infarction (AMI)?

decreased dyspnea with the head of the bed at 30 degrees.

The nurse is caring for a patient receiving IV furosemide (Lasix) 40 mg and enalapril (Vasotec) 5 mg PO bid for ADHF with severe orthopnea. When evaluating the patient response to the medications, the best indicator that the treatment has been effective is ____________________

"Have you been to the dentist lately?"

The nurse obtains a health history from a patient with a prosthetic mitral valve who has symptoms of infective endocarditis. Which question by the nurse is most appropriate?

ventricular tachycardia.

The nurse obtains a monitor strip on a patient admitted to the coronary care unit with a myocardial infarction and makes the following analysis: P wave not apparent; ventricular rate 162, R-R interval regular; PR interval not measurable; and QRS complex wide and distorted, QRS duration 0.18 second. The nurse interprets the patient's cardiac rhythm as _______________

prolonged capillary refill.

The nurse performing an assessment with a patient who has chronic peripheral arterial disease (PAD) of the legs would expect to find _______________

administer IV antidysrhythmic drugs per protocol.

The nurse reviews data from the cardiac monitor indicating that a patient with a myocardial infarction experienced a 50-second episode of ventricular tachycardia before a sinus rhythm and a heart rate of 98 were re-established. The most appropriate initial action by the nurse is to ________________

calls when the weight increases from 124 to 130 pounds in a week.

The nurse working in the heart failure clinic will know that teaching for a 74-year-old patient with newly diagnosed heart failure has been effective when the patient ________________

Arterial pressure

The pressure of the blood against the arterial walls.

You are treating a 75-year-old woman who has a history of diabetes and atherosclerosis. Her chief complaint is persistent heartburn. You suspect:

This may be a cardiovascular problem

pericardial friction rub.

Three days after an MI, the patient develops chest pain that radiates to the back and left arm and is relieved by sitting in a forward position. On auscultation of the patient's chest, the nurse would expect to hear a ____________

place the diaphragm of the stethoscope at the lower left sternal border of the chest.

To assess the patient with pericarditis for the presence of a pericardial friction rub, the nurse should _____________

Assist the patient to modify favorite high-fat recipes by using monosaturated oils when possible.

To assist the patient with CAD to make the appropriate dietary changes, which of these nursing interventions will be most effective?

Cardiac output

Total volume of blood pumped through the heart in 1 minute. Stroke volume x heart rate.

PTCA/Angioplasty

Treatment for CAD - compresses plaque against walls and dilates vessel

situational low self-esteem related to perceived role changes.

Two days after having an MI, a patient tells the nurse, "I wish I had died when I had this heart attack. I won't be able to do anything now." The most appropriate nursing diagnosis is ___________________

33. Stable angina: a. Typically subsides within 10 to 15 minutes b. Occurs after a predictable amount of exertion c. Usually requires both rest and nitroglycerin to subside d. Is characterized by sharp chest pain rather than pressure

Typically subsides within 10 to 15 minutes

1 Turn the defibrillator on. 2 Select the appropriate energy level 3 Place the paddles on the patient's chest. 4 Check the location of other personnel and call out "all clear." 5 Deliver the electrical charge.

When a patient requires defibrillation, in which order will the nurse accomplish the following steps?

60

When analyzing an ECG rhythm strip of a patient with a regular cardiac rhythm, the nurse finds there are 25 small blocks from one R wave to the next. The nurse calculates the patient's heart rate as ______.

QRS interval of 0.14 second.

When analyzing the waveforms of a patient's ECG, the nurse will need to investigate further upon finding a _________

0.78 or 0.79

When assessing a patient with possible PAD, the nurse obtains a brachial BP of 140/80 and an ankle pressure of 110/70. The nurse calculates the patient's ankle-brachial index (ABI) as ______.

about the purpose of outpatient Holter monitoring.

When caring for a patient who has survived a sudden cardiac death (SCD) event and has no evidence of an AMI, the nurse will anticipate teaching the patient _______________

Chest pain level 8 on a 10-point scale

When caring for a patient with ACS who has returned to the coronary care unit after having a PCI, the nurse obtains these assessment data. Which data indicate the need for immediate intervention by the nurse?

dyspnea and hemoptysis.

When caring for a patient with mitral valve stenosis, it is most important that the nurse assess for ______________

dyspnea.

When caring for the patient with infective endocarditis of the tricuspid valve, the nurse will plan to monitor the patient for ____________

low activity level the patient reports.

When developing a health teaching plan for a 65-year-old patient with all these risk factors for coronary artery disease (CAD), the nurse will focus on the __________

positioning the patient in a high-Fowler's position with the feet horizontal in the bed.

When developing a plan to decrease preload in the patient with heart failure, the nurse will include actions such as ______________

"I will have incisions in my leg where they will remove the vein."

When evaluating the outcomes of preoperative teaching with a patient scheduled for a coronary artery bypass graft (CABG) using the internal mammary artery, the nurse determines that additional teaching is needed when the patient says, ________________

use the 3-second markers to count the number of QRS complexes in 6 seconds and multiply by 10.

When needing to estimate the ventricular rate quickly for a patient with a regular heart rhythm using an ECG strip, the nurse will ______________

milk, yogurt, and other milk products.

When teaching the patient with heart failure about a 2000-mg sodium diet, the nurse explains that foods to be restricted include ________________

passing of flatus with ambulation.

When the nurse is caring for a patient on the first postoperative day after an abdominal aortic aneurysm repair, the information that is most significant when the nurse is assessing for the return of peristalsis is ______________

hypertension eventually will lead to heart failure by overworking the heart muscle.

When the nurse is developing a teaching plan to prevent the development of heart failure in a patient with stage 1 hypertension, the information that is most likely to improve compliance with antihypertensive therapy is that _______________

The patient complains of chest pain associated with ambulation.

Which assessment information obtained by the nurse for a patient with aortic stenosis would be most important to report to the health care provider?

ST-segment elevation

Which electrocardiographic (ECG) change will be of most concern to the nurse when admitting a patient with chest pain?

The patient indicates that the pain is resolved after taking one sublingual nitroglycerin tablet.

Which information given by a patient admitted with chronic stable angina will help the nurse confirm this diagnosis?

The patient has crackles audible to the lung apices.

Which information obtained by the nurse when assessing a patient admitted with mitral valve stenosis should be communicated to the health care provider immediately?

Jugular vein distension (JVD) to the level of the jaw

Which of these assessment data obtained by the nurse when assessing a patient with acute pericarditis should be reported immediately to the health care provider?

Assist the patient in using a pillow to splint while coughing.

Which of these nursing actions in the care plan for a patient who had a repair of an abdominal aortic aneurysm 5 days previously is most appropriate for the nurse to delegate to an experienced nursing assistant?

Administration of the ordered metoprolol (Lopressor) and aspirin

Which of these nursing interventions included in the plan of care for a patient who had an AMI 3 days ago is most appropriate for the RN to delegate to an experienced LPN/LVN?

"I will tell my dentist about this surgery the next time I have an appointment."

Which statement by a patient who is being discharged 5 days after an abdominal aortic aneurysm repair and graft indicates that the discharge teaching has been effective?

referral to a home health care agency.

While admitting an 80-year-old patient with heart failure to the medical unit, the nurse obtains the information that the patient lives alone and sometimes confuses the "water pill" with the "heart pill." The nurse makes a note that discharge planning for the patient will need to include _____________

promote rest to decrease myocardial oxygen demand.

While caring for a patient with aortic stenosis, the nurse establishes a nursing diagnosis of pain related to decreased coronary blood flow. An appropriate intervention by the nurse is to ___________

discuss the diagnosis of mitral valve prolapse with the dentist.

While caring for a patient with mitral valve prolapse with mild valvular regurgitation, the nurse determines that discharge teaching has been effective when the patient tells the nurse she will _______________

"Have you had a recent sore throat?"

While obtaining an admission health history from a patient with possible rheumatic fever, which question will be most pertinent to ask?

"My legs cramp whenever I walk more than a block."

While working in the outpatient clinic, the nurse notes that the chart states that a patient has intermittent claudication. Which of these statements by the patient would be consistent with this information?

While the nurse is taking a health history from a patient with hypertrophic cardiomyopathy, information that the nurse recognizes as significant includes

a family history of cardiomyopathy

When interpreting an ECG, the nurse would keep in mind which of the following about the P wave? Select all that apply. a. Reflects electrical impulse beginning at the SA node. b. Indicated electrical impulse beginning at the AV node. c. Reflects atrial muscle depolarization. d. Identifies ventricular muscle depolarization. e. Has duration of normally 0.11 seconds or less.

a, c, e. In a client who has had an ECG, the P wave represents the activation of the electrical impulse in the SA node, which is then transmitted to the AV node. In addition, the P wave represents atrial muscle depolarization, not ventricular depolarization. The normal duration of the P wave is 0.11 seconds or less in duration and 2.5 mm or more in height.

The nurse is caring for a client who's experiencing sinus bradycardia with a pulse rate of 40 beats/minute. His blood pressure is 80/50 mm Hg and he complains of dizziness. Which medication would be used to treat his bradycardia? a. Atropine b. Dobutamine (Dobutrex) c. Amiodarone (Cordarone) d. Lidocaine (Xylocaine)

a. Atropine I.V. push atropine is used to treat symptomatic bradycardia. Dobutamine is used to treat heart failure and low cardiac output. Amiodarone is used to treat ventricular fibrillation and unstable ventricular tachycardia. Lidocaine is used to treat ventricular ectopy, ventricular tachycardia, and ventricular fibrillation.

The nurse teaches the client with angina about the common expected side effects of nitroglycerin, including: a. Headache b. High blood pressure c. Shortness of breath d. Stomach cramps

a. Because of the widespread vasodilating effects, nitroglycerin often produces such side effects as headache, hypotension, and dizziness. The client should lie or **** down to avoid fainting. Nitro does not cause shortness of breath or stomach cramps.

A client is having frequent premature ventricular contractions. A nurse would place priority on assessment of which of the following items? a. Blood pressure and peripheral perfusion b. Sensation of palpitations c. Causative factors such as caffeine d. Precipitating factors such as infection

a. Blood pressure and peripheral perfusion Premature ventricular contractions can cause hemodynamic compromise. The shortened ventricular filling time with the ectopic beats leads to decreased stroke volume and, if frequent enough, to decreased cardiac output. The client may be asymptomatic or may feel palpations. PVCs can be caused by cardiac disorders or by any number of physiological stressors, such as infection, illness, surgery, or trauma, and by the intake of caffeine, alcohol, or nicotine.

A client is admitted to an emergency department with chest pain that is being ruled out for myocardial infarction. Vital signs are as follows at 11am, P: 92 beats/min, RR: 24 breaths/min, BP: 140/88 mm Hg; 11:15am P: 96 beats/min, RR: 26 breaths/min, BP: 128/82 mm Hg, 11:30am P: 104 beats/min, RR: 28 breaths/min, BP: 104/68 mm Hg, 11:45am P: 118 beats/min, RR: 32, breaths/min, BP: 88/58 mm Hg. The nurse should alert the physician because these changes are most consistent with which of the following complications? a. Cardiogenic shock b. Cardiac tamponade c. Pulmonary embolism d. Dissecting thoracic aortic anyeurysm

a. Cardiogenic shock Cardiogenic shock occurs with severe damage (more than 40%) to the left ventricle. Classic signs include hypertension, a rapid pulse that becomes weaker, decreased urine output, and cool, clammy skin. Respiratory rate increases as the body develops metabolic acidosis from shock. Cardiac tamponade is accompanied by distant, muffled heart sounds and prominent neck vessels. Pulmonary embolism presents suddenly with severe dyspnea accompanying the chest pain. Dissecting aortic aneyurysms usually are accompanied by back pain.

A client enters the ER complaining of chest pressure and severe epigastric distress. His VS are 158/90, 94, 24, and 99*F. The doctor orders cardiac enzymes. If the client were diagnosed with an MI, the nurse would expect which cardiac enzyme to rise within the next 3 to 8 hours? a. Creatine kinase (CK or CPK) b. Lactic dehydrogenase (LDH) c. LDH-1 d. LDH-2

a. Creatine kinase (CK, formally known as CPK) rises in 3-8 hours if an MI is present. When the myocardium is damaged, CPK leaks out of the cell membranes and into the blood stream. Lactic dehydrogenase rises in 24-48 hours, and LDH-1 and LDH-2 rises in 8-24 hours.

Good dental care is an important measure in reducing the risk of endocarditis. A teaching plan to promote good dental care in a client with mitral stenosis should include demonstration of the proper use of: a. A manual toothbrush b. An electric toothbrush c. An irrigation device d. Dental floss

a. Daily dental care and frequent checkups by a dentist who is informed about the client's condition are required to maintain good oral health. Use of an electric toothbrush, an irrigation device, or dental floss may cause gums to bleed and allow bacteria to enter mucous membranes and the blood stream, increasing the risk of endocarditis.

A client is recovering from coronary artery bypass graft (CABG) surgery. Which nursing diagnosis takes highest priority at this time? a. Decreased cardiac output related to depressed myocardial function, fluid volume deficit, or impaired electrical conduction b. Anxiety related to an actual threat to health status, invasive procedures, and pain c. Disabled family coping related to knowledge deficit and a temporary change in family dynamics d. Hypothermia related to exposure to cold temperatures and a long cardiopulmonary bypass time

a. Decreased cardiac output related to depressed myocardial function, fluid volume deficit, or impaired electrical conduction For a client recovering from CABG surgery, Decreased cardiac output is the most important nursing diagnosis because myocardial function may be depressed from anesthetics or a long cardiopulmonary bypass time, leading to decreased cardiac output. Other possible causes of decreased cardiac output in this client include fluid volume deficit and impaired electrical conduction. The other options may be relevant but take lower priority at this time because maintaining cardiac output is essential to sustaining the client's life.

A client is recovering from coronary artery bypass graft (CABG) surgery. The nurse knows that for several weeks after this procedure, the client is at risk for certain conditions. During discharge preparation, the nurse should advise the client and family to expect which common symptom that typically resolves spontaneously? a. Depression b. Ankle edema c. Memory lapses d. Dizziness

a. Depression For the first few weeks after CABG surgery, clients commonly experience depression, fatigue, incisional chest discomfort, dyspnea, and anorexia. Depression typically resolves on its own and doesn't require medical intervention; however, family members should be aware that symptoms don't always resolve on their own. They should also be instructed about worsening symptoms of depression and when to seek care. Ankle edema seldom follows CABG surgery and may indicate right-sided heart failure; because this condition is a sign of cardiac dysfunction, the client should report ankle edema at once. Memory lapses reflect neurologic rather than cardiac dysfunction. Dizziness may result from decreased cardiac output, an abnormal condition after CABG surgery that warrants immediate physician notification.

A nurse notes 2+ bilateral edema in the lower extremities of a client with myocardial infarction who was admitted 2 days ago. The nurse would plan to do which of the following next? a. Review the intake and output records for the last 2 days. b. Change the time of diuretic administration from morning to evening. c. Request a sodium restriction of 1 g/day from the physician. d. Order daily weights starting the following morning.

a. Edema, the accumulation of excess fluid in the interstitial spaces, can be measured by intake greater than output and by a sudden increase in weight. Diuretics should be given in the morning whenever possible to avoid nocturia. Strict sodium restrictions are reserved for clients with severe symptoms.

Which signs and symptoms are present with a diagnosis of pericarditis? a. Fever, chest discomfort, and elevated erythrocyte sedimentation rate (ESR) b. Low urine output secondary to left ventricular dysfunction c. Lethargy, anorexia, and heart failure d. Pitting edema, chest discomfort, and nonspecific ST-segment elevation

a. Fever, chest discomfort, and elevated erythrocyte sedimentation rate (ESR) The classic signs and symptoms of pericarditis include fever, positional chest discomfort, nonspecific ST-segment elevation, elevated ESR, and pericardial friction rub. All other symptoms may result from acute renal failure.

In presenting a workshop on parameters of cardiac function, which conditions should the nurse list as those most likely to lead to a decrease in preload? a. Hemorrhage, sepsis, and anaphylaxis b. Myocardial infarction, fluid overload, and diuresis c. Fluid overload, sepsis, and vasodilation d. Third spacing, heart failure, and diuresis

a. Hemorrhage, sepsis, and anaphylaxis Preload is the volume in the left ventricle at the end of diastole. It's also referred to as end-diastolic volume. Preload is reduced by any condition that reduces circulating volume, such as hemorrhage, sepsis, and anaphylaxis. Hemorrhage reduces circulating volume by loss of volume from the intravascular space. Sepsis and anaphylaxis reduce circulating volume by increased capillary permeability. Diuresis, vasodilation, and third spacing also reduce preload. Preload would increase with fluid overload and heart failure.

A client has developed atrial fibrillation, which a ventricular rate of 150 beats per minute. A nurse assesses the client for: a. Hypotension and dizziness b. Nausea and vomiting c. Hypertension and headache d. Flat neck veins

a. Hypotension and dizziness The client with uncontrolled atrial fibrillation with a ventricular rate more than 150 beats a minute is at risk for low cardiac output because of loss of atrial kick. The nurse assesses the client for palpitations, chest pain or discomfort, hypotension, pulse deficit, fatigue, weakness, dizziness, syncope, shortness of breath, and distended neck veins.

Which sign or symptom suggest that a client's abdominal aortic aneurysm is extending? a. Increased abdominal and back pain b. Decreased pulse rate and blood pressure c. Retrosternal back pain radiating to the left arm d. Elevated blood pressure and rapid respirations

a. Increased abdominal and back pain Pain in the abdomen and back signify that the aneurysm is pressing downward on the lumbar nerve root and is causing more pain. The pulse rate would increase with aneurysm extension. Chest pain radiating down the arm would indicate myocardial infarction. Blood pressure would decrease with aneurysm extension, and the respiratory rate may not be affected.

A nurse is assessing an electrocardiogram rhythm strip. The P waves and QRS complexes are regular. The PR interval is 0.16 second, and QRS complexes measure 0.06 second. The overall heart rate is 64 beats per minute. The nurse assesses the cardiac rhythm as: a. Normal sinus rhythm b. Sinus bradycardia c. Sick sinus syndrome d. First-degree heart block.

a. Normal sinus rhythm measurements are normal, measuring 0.12 to 0.20 seconds and < .12 seconds, respectively.

Because a client has mitral stenosis and is a prospective valve recipient, the nurse preoperatively assesses the client's past compliance with medical regimens. Lack of compliance with which of the following regimens would pose the greatest health hazard to this client? a. Medication therapy b. Diet modification c. Activity restrictions d. Dental care

a. Preoperatively, anticoagulants may be prescribed for the client with advanced valvular heart disease to prevent emboli. Post-op, all clients with mechanical valves and some with bioprostheses are maintained indefinitely on anticoagulation therapy. Adhering strictly to a dosage schedule and observing specific precautions are necessary to prevent hemorrhage or thromboembolism. Some clients are maintained on lifelong antibiotic prophylaxis to prevent recurrence from rheumatic fever. Episodic prophylaxis is required to prevent infective endocarditis after dental procedures or upper respiratory, GI, or GU surgery.

When teaching a client about propranolol hydrochloride, the nurse should base the information on the knowledge that propranolol hydrochloride: a. Blocks beta-adrenergic stimulation and thus causes decreased heart rate, myocardial contractility, and conduction. b. Increases norepinephrine secretion and thus decreases blood pressure and heart rate. c. Is a potent arterial and venous vasodilator that reduces peripheral vascular resistance and lowers blood pressure. d. Is an angiotensin-converting enzyme inhibitor that reduces blood pressure by blocking the conversion of angiotensin I to angiotensin II.

a. Propranolol hydrochloride is a beta-adrenergic blocking agent. Actions of propranolol hydrochloride include reducing heart rate, decreasing myocardial contractility, and slowing conduction.

A client is wearing a continuous cardiac monitor, which begins to sound its alarm. A nurse sees no electrocardiogram complexes on the screen. The first action of the nurse is to: a. Check the client status and lead placement. b. Press the recorder button on the electrocardiogram console. c. Call the physician. d. Call a code blue.

a. Sudden loss of electrocardiogram complexes indicates ventricular asystole or possible electrode displacement. Accurate assessment of the client and equipment is necessary to determine the cause and identify the appropriate intervention.

After open-heart surgery a client develops a temperature of 102*F. The nurse notifies the physician, because elevated temperatures: a. Increase the cardiac output b. May indicate cerebral edema c. May be a forerunner of hemorrhage d. Are related to diaphoresis and possible chilling

a. Temperatures of 102*F or greater lead to an increased metabolism and cardiac workload.

What criteria should the nurse use to determine normal sinus rhythm for a client on a cardiac monitor? Check all that apply. a. The RR intervals are relatively consistent b. One P wave precedes each QRS complex c. Four to eight complexes occur in a 6 second strip d. The ST segment is higher than the PR interval e. The QRS complex ranges from 0.12 to 0.20 second.

a. The RR intervals are relatively consistent b. One P wave precedes each QRS complex The consistency of the RR interval indicates regular rhythm. A normal P wave before each complex indicates the impulse originated in the SA node. The number of complexes in a 6 second strip is multiplied by 10 to approximate the heart rate; normal sinus rhythm is 60 to 100. Elevation of the ST segment is a sign of cardiac ischemia and is unrelated to the rhythm. The QRS duration should be less than 0.12 second; the PR interval should be 0.12 to 0.20 second.

The nurse is evaluating a client who had a myocardial infarction (MI) 7 days ago. Which outcome indicates that the client is responding favorably to therapy? a. The client demonstrates the ability to tolerate increasing activity without chest pain. b. The client exhibits a heart rate above 100 beats/minute. c. The client verbalizes the intention of making all necessary lifestyle changes except for stopping smoking. d. The client states that sublingual nitroglycerin usually relieves chest pain.

a. The client demonstrates the ability to tolerate increasing activity without chest pain. The ability to tolerate increasing activity without chest pain indicates a favorable response to therapy in a client who is recovering from an MI or who has a history of coronary artery disease. The client should have a normal electrocardiogram with no arrhythmias and a regular heart rate of 60 to 100 beats/minute. Smoking is a cardiovascular risk factor that the client must be willing to eliminate. A client who responds favorably to therapy shouldn't have chest pain.

Which of the following symptoms should the nurse teach the client with unstable angina to report immediately to her physician? a. A change in the pattern of her pain b. Pain during sex c. Pain during an argument with her husband d. Pain during or after an activity such as lawnmowing

a. The client should report a change in the pattern of chest pain. It may indicate increasing severity of CAD.

After receiving shift report, the registered nurse in the cardiac step-down unit, must prioritize her client care assignment. She has an ancillary staff member available to help her care for her clients. Which of these clients should the registered nurse assess first? a. The client with heart failure who is having some difficulty breathing. b. The anxious client who was diagnosed with an acute myocardial infarction (MI) two days ago and who was transferred from the coronary care unit today. c. The demanding client who underwent coronary bypass surgery three days ago. d. The client admitted during the previous shift with new-onset controlled atrial fibrillation who has her call light on.

a. The client with heart failure who is having some difficulty breathing. The registered nurse should care for the client with heart failure who is experiencing difficulty breathing. Breathing takes precedence over the other client needs. The ancillary staff member can answer the call light of the client admitted with controlled atrial fibrillation. She can also attend to the demanding client who underwent coronary bypass surgery three days ago. Although anxiety can be detrimental to a client with an MI, anxiety doesn't take precedence over another client's breathing difficulty.

When the patient diagnosed with angina pectoris complains that he is experiencing chest pain more frequently even at rest, the period of pain is longer, and it take less stress for the pain to occur, the nurse recognizes that the patient is describing which type of angina? a. Unstable b. Intractable c. Refractory d. Variant

a. Unstable

A client with rapid rate atrial fibrillation asks a nurse why the physician is going to perform carotid massage. The nurse responds that this procedure may stimulate the: a. Vagus nerve to slow the heart rate b. Vagus nerve to increase the heart rate; overdriving the rhythm. c. Diaphragmic nerve to slow the heart rate d. Diaphragmic nerve to overdrive the rhythm

a. Vagus nerve to slow the heart rate Carotid sinus massage is one of the maneuvers used for vagal stimulation to decrease a rapid heart rate and possibly terminate a tachydysrhythmia. The others include inducing the gag reflex and asking the client to strain or bear down. Medication therapy often is needed as an adjunct to keep the rate down or maintain the normal rhythm.

Which of the following signs and symptoms would most likely be found in a client with mitral regurgitation? a. Exertional dyspnea b. Confusion c. Elevated creatine phosphokinase concentration d. Chest pain

a. Weight gain, due to fluid retention and worsening heart failure, causes exertional dyspnea in clients with mitral regurgitation. The rise in left atrial pressure that accompanies mitral valve disease is transmitted backward into pulmonary veins, capillaries, and arterioles and eventually to the right ventricle. Signs and symptoms of pulmonary and systemic venous congestion follow.

The nurse is caring for a cardiac client who requires various cardiac medications. When the nurse helps the client out of bed for breakfast, the client becomes dizzy and asks to lie down. The nurse helps the client lie down, puts up the side rails, and obtains the client's blood pressure, which is 84/50 mm Hg. It's time for the nurse to administer the client's medications: nitroglycerin, metoprolol (Lopressor), and furosemide (Lasix). Which action is best taken by the nurse? a. Withhold the medications and notify the physician. b. Administer the medications immediately. c. Encourage the client to sit up and eat breakfast. d. Administer the nitroglycerin and metoprolol and withhold the furosemide.

a. Withhold the medications and notify the physician. The nurse should withhold the three medications and notify the physician. Each of these medications has the potential to lower the client's blood pressure. Administering them together when the client is already hypotensive may severely lower the client's blood pressure. The client may continue to experience dizziness when sitting up so breakfast should be held until his blood pressure stabilizes.

A client with chest pain doesn't respond to nitroglycerin. On admission to the emergency department, the health care team obtains an electrocardiogram and begins infusing I.V. morphine. The physician also considers administering alteplase (Activase). This thrombolytic agent must be administered how soon after onset of myocardial infarction (MI) symptoms? a. Within 12 hours b. Within 24 hours c. Within 24 to 48 hours d. Within 5 to 7 days

a. Within 12 hours For the best chance of salvaging the client's myocardium, a thrombolytic agent must be administered within 12 hours after onset of chest pain or other signs or symptoms of MI. Within the first 24 hours after an MI, sudden death is most likely to occur. I.V. heparin therapy begins after administration of a thrombolytic agent and usually continues for 5 to 7 days.

An obese white male client, age 49, is diagnosed with hypercholesterolemia. The physician prescribes a low-fat, low-cholesterol, low-calorie diet to reduce blood lipid levels and promote weight loss. This diet is crucial to the client's well-being because his race, sex, and age increase his risk for coronary artery disease (CAD). To determine if the client has other major risk factors for CAD, the nurse should assess for: a. a history of diabetes mellitus. b. elevated high-density lipoprotein (HDL) levels. c. a history of ischemic heart disease. d. alcoholism.

a. a history of diabetes mellitus. Diabetes mellitus, smoking, and hypertension are other major risk factors for CAD. Elevated HDL levels aren't a risk factor for CAD; in fact, increased HDL levels seem to protect against CAD. Ischemic heart disease is another term for CAD, not a risk factor. Alcoholism hasn't been identified as a major risk factor for CAD.

A client reports recent onset of chest pain that occurs sporadically with exertion. The client also has fatigue and mild ankle swelling, which is most pronounced at the end of the day. The nurse suspects a cardiovascular disorder. When exploring the chief complaint, the nurse should find out if the client has any other common cardiovascular symptoms, such as: a. shortness of breath. b. insomnia. c. irritability. d. lower substernal abdominal pain.

a. shortness of breath. Common signs and symptoms of cardiovascular dysfunction include shortness of breath, chest pain, dyspnea, palpitations, fainting, fatigue, and peripheral edema. Insomnia seldom indicates a cardiovascular problem. Although irritability may occur if cardiovascular dysfunction leads to cerebral oxygen deprivation, this symptom more commonly reflects a respiratory or neurologic dysfunction. Lower substernal abdominal pain occurs with some GI disorders.

Antidysrhytmics and MI

amioderone, lidocaine

Patients with metal valve replacements require what type of therapy?

anticoagulant - lifetime

A client presents to the ED in atrial fibrillation, The client has no medical history of this. The nurse anticipates the patient will be initially treated with:

anticoagulant therapy

Before discharge, which instruction should the nurse give to a client receiving digoxin (Lanoxin)? a. "Take an extra dose of digoxin if you miss one dose." b. "Call the physician if your heart rate is above 90 beats/minute." c. "Call the physician if your pulse drops below 80 beats/minute." d. "Take digoxin with meals."

b. "Call the physician if your heart rate is above 90 beats/minute." The nurse should instruct the client to notify the physician if his heart rate is greater than 90 beats/minute because cardiac arrhythmias may occur with digitalis toxicity. To prevent toxicity, the client should be instructed never to take an extra dose of digoxin if a dose is missed. The nurse should show the client how to take her pulse and to call the physician if her pulse rate drops below 60 beats/minute — not 80 beats/minute, which is a normal pulse rate and doesn't warrant action. Digoxin shouldn't be administered with meals because this slows the absorption rate.

The home care nurse is visiting a left-handed client who has an automated implantable cardioverter-defibrillator implanted in his left chest. The client tells the nurse how excited he is because he's planning to go rifle hunting with his grandson. How should the nurse respond? a. "Be sure to enjoy your time with your grandson." b. "You cannot shoot a rifle left-handed because the rifle's recoil will traumatize the AICD site." c. "Being that close to a rifle might make your AICD fire." d. "You will need to take an extra dose of your antiarrhythmic before you shoot."

b. "You cannot shoot a rifle left-handed because the rifle's recoil will traumatize the AICD site." The recoil from the rifle can damage the AICD so the client should be warned against shooting a rifle with his left hand. Close proximity to a rifle won't cause the AICD to fire inadvertently. The client shouldn't take an extra dose of his antiarrhythmic.

During surgery, a client develops sinus bradycardia. The physician orders atropine sulfate. Which dose and route is the nurse most likely to administer? a. 0.6 mg I.M. b. 1 mg I.V. c. 2 mg I.M. d. 2 mg I.V.

b. 1 mg I.V. To reverse arrhythmias, bradycardia, or sinus arrest, the usual adult dosage of atropine is 0.4 to 1 mg I.V. every 2 hours as needed. The drug isn't administered I.M. for the treatment of bradycardia.

Following a percutaneous transluminal coronary angioplasty (PTCA), a client is monitored in the postprocedure unit. The client's heparin infusion was stopped 2 hours earlier. There is no evidence of bleeding or hematoma at the insertion site, and the pressure device is removed. The nurse should plan to remove the femoral sheath when the partial thromboplastin time (PTT) is: a. 25 seconds or less. b. 50 seconds or less. c. 75 seconds or less. d. 100 seconds or less.

b. 50 seconds or less. Heparin causes an elevation of the PTT and, thereby, increases the risk for bleeding. With a large cannulation such as a sheath used for angioplasty, the PTT should be 40 seconds or less before the sheath is removed. Removing the sheath prematurely can cause bleeding at the insertion site. The other PTT results are incorrect for determining when to remove the sheath.

A client has driven himself to the ER. He is 50 years old, has a history of hypertension, and informs the nurse that his father died of a heart attack at 60 years of age. The client is presently complaining of indigestion. The nurse connects him to an ECG monitor and begins administering oxygen at 2 L/minute per NC. The nurse's next action would be to: a. Call for the doctor. b. Start an intravenous line. c. Obtain a portable chest radiograph. d. Draw blood for laboratory studies.

b. Advanced cardiac life support recommends that at least one or two intravenous lines be inserted in one or both of the antecubital spaces. Calling the physician, obtaining a portable chest radiograph, and drawing blood are important but secondary to starting the intravenous line.

A 35-year-old male was knifed in the street fight, admitted through the ER, and is now in the ICU. An assessment of his condition reveals the following symptoms: respirations shallow and rapid, CVP 15 cm H2O, BP 90 mm Hg systolic, skin cold and pale, urinary output 60-100 mL/hr for the last 2 hours. Analyzing these symptoms, the nurse will base a nursing diagnosis on the conclusion that the client has which of the following conditions? a. Hypovolemic shock b. Cardiac tamponade c. Wound dehiscence d. Atelectasis

b. All of the client's symptoms are found in both cardiac tamponade and hypovolemic shock except the increase in urinary output.

The nurse is preparing a client for cardiac catheterization. The nurse knows that she must provide which nursing intervention when the client returns to his room after the procedure? a. Withhold analgesics for at least 6 hours after the procedure. b. Assess the puncture site frequently for hematoma formation or bleeding. c. Inform the client that he may experience numbness or pain in his leg. d. Restrict fluids for 6 hours after the procedure.

b. Assess the puncture site frequently for hematoma formation or bleeding. Because the diameter of the catheter used for cardiac catheterization is large, the puncture site must be checked frequently for hematoma formation and bleeding. The nurse should administer analgesics as prescribed and needed. If the femoral artery was accessed during the procedure, the client should be instructed to report any leg pain or numbness, which may indicate arterial insufficiency. Fluids should be encouraged to eliminate dye from the client's system.

A nurse is watching the cardiac monitor, and a client's rhythm suddenly changes. There are no P waves; instead there are wavy lines. The QRS complexes measure 0.08 second, but they are irregular, with a rate of 120 beats a minute. The nurse interprets this rhythm as: a. Sinus tachycardia b. Atrial fibrillation c. Ventricular tachycardia d. Ventricular fibrillation

b. Atrial fibrillation Atrial fibrillation is characterized by a loss of P waves; an undulating, wavy baseline; QRS duration that is often within normal limits; and an irregular ventricular rate, which can range from 60 to 100 beats per minute (when controlled with medications) to 100 to 160 beats per minute (when uncontrolled).

Which of the following is a modifiable risk factor for coronary artery disease ( CAD)? a. Race b. Diabetes mellitus c. Gender d. Increasing age

b. Diabetes mellitus

When a client is started on oral or I.V. diltiazem (Cardizem), the nurse should monitor for which potential complication? a. Flushing b. Heart failure c. Renal failure d. Hypertension

b. Heart failure The chief adverse effects of diltiazem are hypotension, atrioventricular blocks, heart failure, and elevated liver enzyme levels. Other reactions that have been reported include flushing, nocturia, and polyuria, but not renal failure. Although flushing may occur, it's an adverse reaction, not a potential complication. Heart failure is a life-threatening reaction.

A client who suffered blunt chest trauma in a car accident complains of chest pain, which is exacerbated by deep inspiration. On auscultation, the nurse detects a pericardial friction rub — a classic sign of acute pericarditis. The physician confirms acute pericarditis and begins appropriate medical intervention. To relieve chest pain associated with pericarditis, the nurse should encourage the client to assume which position? a. Semi-Fowler's b. Leaning forward while sitting c. Supine d. Prone

b. Leaning forward while sitting When the client leans forward, the heart pulls away from the diaphragmatic pleurae of the lungs, helping relieve chest pain caused by pericarditis. The semi-Fowler, supine, and prone positions don't cause this pulling-away action and therefore don't relieve chest pain associated with pericarditis.

A client is in the early stage of heart failure. During this time, which compensatory mechanism occurs? a. Decreased renal blood flow causes the renin-angiotensin-aldosterone system to lower the secretion of aldosterone and antidiuretic hormone. b. Low blood pressure triggers the baroreceptors to increase sympathetic nervous system stimulation. c. Decreased renal blood flow causes the renin-angiotensin-aldosterone system to raise aldosterone secretion. d. Low blood pressure triggers the baroreceptors to decrease sympathetic nervous system stimulation.

b. Low blood pressure triggers the baroreceptors to increase sympathetic nervous system stimulation. In the early stage of heart failure, low blood pressure triggers baroreceptors in the carotid sinus and aortic arch to increase sympathetic nervous system stimulation, causing a faster heart rate, vasoconstriction, and increased myocardial oxygen consumption. Decreased renal blood flow causes the renin-angiotensin-aldosterone system to increase, not reduce, secretion of aldosterone and antidiuretic hormone, in turn causing sodium and water retention and arterial vasoconstriction.

While caring for a client who has sustained an MI, the nurse notes eight PVCs in one minute on the cardiac monitor. The client is receiving an IV infusion of D5W and oxygen at 2 L/minute. The nurse's first course of action should be to: a. Increase the IV infusion rate b. Notify the physician promptly c. Increase the oxygen concentration d. Administer a prescribed analgesic

b. Notify the physician promptly PVCs are often a precursor of life-threatening dysrhythmias, including ventricular tachycardia and ventricular fibrillation. An occasional PVC is not considered dangerous, but if PVCs occur at a rate greater than 5 or 6 per minute in the post MI client, the physician should be notified immediately. More than 6 PVCs per minute is considered serious and usually calls for decreasing ventricular irritability by administering medications such as lidocaine. Increasing the IV infusion rate would not decrease the number of PVCs. Increasing the oxygen concentration should not be the nurse's first course of action; rather, the nurse should notify the physician promptly. Administering a prescribed analgesic would not decrease ventricular irritability.

A client with myocardial infarction has been transferred from a coronary care unit to a general medical unit with cardiac monitoring via telemetry. A nurse plans to allow for which of the following client activities? a. Strict bed rest for 24 hours after transfer. b. Bathroom privileges and self-care activities. c. Unsupervised hallway ambulation with distances under 200 feet. d. Ad lib activities because the client is monitored.

b. On transfer from the CCU, the client is allowed self-care activities and bathroom privileges. Supervised ambulation for brief distances are encouraged, with distances gradually increased (50, 100, 200 feet).

A client with end-stage heart failure is preparing for discharge. The client and his caregiver meet with the home care nurse and voice their concern that setting up a hospital bed in the bedroom will leave him feeling isolated. Which suggestion by the home care nurse best addresses this concern? a. Place a chair in the bedroom so guests can visit with the client. b. Set up the hospital bed in the family room so the client can be part of household activities. c. Set up the hospital bed in the bedroom so the client can rest in a quiet environment. d. Set up the hospital bed in the bedroom so the client can be assessed in a quiet environment.

b. Set up the hospital bed in the family room so the client can be part of household activities. The client should be kept actively involved in the household to prevent feelings of isolation. This can be accomplished by setting up the hospital bed in the family room. Placing a chair in the bedroom allows the client periods of isolation when visitors aren't present. It's important for the client to have periods of rest; however, that can be accomplished without keeping the client isolated in a bedroom. The needs of the client should be considered before the needs of the nurse who assesses the client during an occasional visit.

The nurse just received shift report for a group of clients on the telemetry unit. Which client should the nurse assess first. a. The client with a history of atrial fibrillation b. The client admitted with first-degree atrioventricular (AV) block whose cardiac monitor now reveals type II second-degree AV block c. The client with a history of heart failure who has bibasilar crackles and pitting edema in both feet d. The client with a demand pacemaker whose monitor shows normal sinus rhythm at a rate of 90 beats/minute

b. The client admitted with first-degree atrioventricular (AV) block whose cardiac monitor now reveals type II second-degree AV block The client whose cardiac rhythm now shows type II second-degree AV block should be assessed first. The client's rhythm has deteriorated from first-degree heart block to type II second-degree AV block and may continue to deteriorate into a lethal form of AV block (known as complete heart block). The client with a history of atrial fibrillation whose monitor reveals atrial fibrillation doesn't need to be assessed first. Because his rhythm is chronic, he has most likely been given an anticoagulant and isn't at immediate risk from this rhythm. The client with a history of heart failure may have chronic bibasilar crackles and pitting edema of both feet. Therefore, assessing this client first isn't necessary. The client's demand pacemaker fires only when the client's intrinsic heart rate falls below the pacemaker's set rate. In option 4, the pacemaker isn't firing because it most likely has been set at a slower rate than the client's intrinsic heart rate of 90 beats/minute.

The nurse is assigned with an ancillary staff member to care for a group of cardiac clients. Which client should the nurse address first? a. The client admitted with unstable angina who wants to be discharged. b. The client who suffered an acute myocardial infarction (MI) who is complaining of constipation. c. The client who had a pacemaker inserted yesterday and who is complaining of incisional pain. d. The client who has his call light on.

b. The client who suffered an acute myocardial infarction (MI) who is complaining of constipation. The client who suffered an acute MI who is complaining of constipation should be addressed first. If the client strains at stool after an MI, the vagal response may be stimulated causing bradycardia thereby provoking arrhythmias. The nurse should delegate answering the call light to the ancillary personnel. She can also delegate some of the discharge preparation, such as packing the client's belongings. After addressing the MI client with constipation, the nurse should promptly address the pain relief needs of the client who had a pacemaker inserted the previous day.

Before using a defibrillator to terminate ventricular fibrillation, the nurse should check the synchronizer switch. Why is this so important? a. The delivered shock must be synchronized with the client's QRS complex. b. The defibrillator won't deliver a shock if the synchronizer switch is turned on. c. The defibrillator won't deliver a shock if the synchronizer switch is turned off. d. The shock must be synchronized with the client's T wave.

b. The defibrillator won't deliver a shock if the synchronizer switch is turned on. The defibrillator won't deliver a shock to the client in ventricular fibrillation if the synchronizer switch is turned on because the defibrillator needs to recognize a QRS complex when the switch is turned on. The synchronizer switch should be turned on when attempting to terminate arrhythmias that contain QRS complexes, such as rapid atrial fibrillation that is resistant to pharmacologic measures.

A nurse notices frequent artifact on the ECG monitor for a client whose leads are connected by cable to a console at the bedside. The nurse examines the client to determine the cause. Which of the following items is unlikely to be responsible for the artifact? a. Frequent movement of the client b. Tightly secured cable connections c. Leads applied over hairy areas d. Leads applied to the limbs

b. Tightly secured cable connections Motion artifact, or "noise," can be caused by frequent client movement, electrode placement on limbs, and insufficient adhesion to the skin, such as placing electrodes over hairy areas of the skin. Electrode placement over bony prominences also should be avoided. Signal interference can also occur with electrode removal and cable disconnection.

A client who has been receiving heparin therapy also is started on warfarin. The client asks a nurse why both medications are being administered. In formulating a response, the nurse incorporates the understanding that warfarin: a. Stimulates the breakdown of specific clotting factors by the liver, and it takes 2-3 days for this to exert an anticoagulant effect. b. Inhibits synthesis of specific clotting factors in the liver, and it takes 3-4 days for this medication to exert an anticoagulant effect. c. Stimulates production of the body's own thrombolytic substances, but it takes 2-4 days for this to begin. d. Has the same mechanism of action as Heparin, and the crossover time is needed for the serum level of warfarin to be therapeutic.

b. Warfarin works in the liver and inhibits synthesis of four vitamin K-dependent clotting factors (X, IX, VII, and II), but it takes 3 to 4 days before the therapeutic effect of warfarin is exhibited.

A client with left-sided heart failure complains of increasing shortness of breath and is agitated and coughing up pink-tinged, foamy sputum. The nurse should recognize these as signs and symptoms of: a. right-sided heart failure. b. acute pulmonary edema. c. pneumonia. d. cardiogenic shock.

b. acute pulmonary edema. Because of decreased contractility and increased fluid volume and pressure in clients with heart failure, fluid may be driven from the pulmonary capillary beds into the alveoli, causing pulmonary edema. In right-sided heart failure, the client would exhibit hepatomegaly, jugular vein distention, and peripheral edema. In pneumonia, the client would have a temperature spike and sputum that varies in color. Cardiogenic shock would show signs of hypotension and tachycardia.

A client with substernal chest pain that radiates to the jaw is admitted to the coronary care unit. The client subsequently develops hypotension and suffers cardiac arrest. Which calcium preparation is injected into the ventricle during cardiac arrest? a. calcium carbonate (BioCal) b. calcium chloride c. calcium glubionate (Neo-Calglucon) d. calcium lactate

b. calcium chloride Calcium chloride is the only calcium preparation that should be injected into the ventricle during cardiac arrest, if appropriate.

A client is experiencing an acute myocardial infarction (MI) and I.V. morphine is prescribed. Morphine is given because it: a. eliminates pain, reduces cardiac workload, and increases myocardial contractility. b. lowers resistance, reduces cardiac workload, and decreases myocardial oxygen demand. c. raises the blood pressure, lowers myocardial oxygen demand, and eliminates pain. d. increases venous return, lowers resistance, and reduces cardiac workload.

b. lowers resistance, reduces cardiac workload, and decreases myocardial oxygen demand. When given to treat acute MI, morphine sulfate eliminates pain, reduces venous return to the heart, reduces vascular resistance, reduces myocardial workload, and reduces the oxygen demand of the heart. Morphine sulfate doesn't increase myocardial contractility, raise blood pressure, or increase venous return.

A client with a history of myocardial infarction is admitted with shortness of breath, anxiety, and slight confusion. Assessment findings include a regular heart rate of 120 beats/minute, audible third and fourth heart sounds, blood pressure of 84/64 mm Hg, bibasilar crackles on lung auscultation, and a urine output of 5 ml over the past hour. The nurse anticipates preparing the client for transfer to the intensive care unit and pulmonary artery catheter insertion because: a. the client is experiencing heart failure. b. the client is going into cardiogenic shock. c. the client shows signs of aneurysm rupture. d. the client is in the early stage of right-sided heart failure.

b. the client is going into cardiogenic shock. This client's findings indicate cardiogenic shock, which occurs when the heart fails to pump properly, impeding blood supply and oxygen flow to vital organs. Cardiogenic shock also may cause cold, clammy skin and generalized weakness, fatigue, and muscle pain as lactic acid accumulates from poor blood flow, preventing waste removal. Left-sided and right-sided heart failure eventually cause venous congestion with jugular vein distention and edema as the heart fails to pump blood forward. A ruptured aneurysm causes severe hypotension and a quickly deteriorating clinical status from blood loss and circulatory collapse; this client has low but not severely decreased blood pressure. Also, in ruptured aneurysm, deterioration is more rapid and full cardiac arrest is common.

The nurse teaches a patient who is taking metoprolol (Topral) for treatment of hypertension to change position slowly because this drug

blocks the normal sympathetic nervous system response of vasoconstriction and increased heart rate with position changes

Troponin level

blood test used to diagnose MI - level rises within 3 hours and remains elevated 7 - 10 days

A client with angina pectoris must learn how to reduce risk factors that exacerbate this condition. When developing the client's care plan, the nurse should include which expected outcome? a. "Client will verbalize an understanding of the need to call the physician if acute pain lasts more than 2 hours." b. "Client will verbalize the intention to avoid exercise." c. "Client will verbalize the intention to stop smoking." d. "Client will verbalize an understanding of the need to restrict dietary fat, fiber, and cholesterol."

c. "Client will verbalize the intention to stop smoking." The client with angina pectoris should stop smoking at once because smoking increases the blood carboxyhemoglobin level; this, in turn, reduces the heart's oxygen supply and may induce angina. The client must seek immediate medical attention if chest pain doesn't subside after three nitroglycerin doses taken 10 to 15 minutes apart; serious myocardial damage or even sudden death may occur if chest pain persists for 2 hours. To improve coronary circulation and promote weight management, the client should get regular daily exercise. The client should eat plenty of fiber, which may decrease serum cholesterol and triglyceride levels and minimize hypertension, in turn reducing the risk for atherosclerosis (which plays a role in angina).

A nurse is viewing the cardiac monitor in a client's room and notes that the client has just gone into ventricular tachycardia. The client is awake and alert and has good skin color. The nurse would prepare to do which of the following? a. Immediately defibrillate b. Prepare for pacemaker insertion c. Administer amiodarone (Cordarone) intravenously d. Administer epinephrine (Adrenaline) intravenously

c. Administer amiodarone (Cordarone) intravenously First-line treatment of ventricular tachycardia in a client who is hemodynamically stable is the use of anti-dysrhythmics such as amiodarone (Cordarone), lidocaine (Xylocaine), and procainamide (Pronestyl). Cardioversion also may be needed to correct the rhythm (cardioversion is recommended for stable ventricular tachycardia). Defibrillation is used with pulseless ventricular tachycardia. Epinephrine would stimulate and already excitable ventricle and is contraindicated.

For a patient who has undergone peripheral arteriography, how should the nurse assess the adequacy of peripheral circulation? a. By checking for cardiac dysrhythmias b. By observing the client for bleeding c. By checking peripheral pulses d. By hemodynamic monitoring

c. By checking peripheral pulses

A client with no history of cardiovascular disease comes into the ambulatory clinic with flu-like symptoms. The client suddenly complains of chest pain. Which of the following questions would best help a nurse to discriminate pain caused by a non-cardiac problem? a. "Have you ever had this pain before?" b. "Can you describe the pain to me?" c. "Does the pain get worse when you breathe in?" d. "Can you rate the pain on a scale of 1-10, with 10 being the worst?"

c. Chest pain is assessed by using the standard pain assessment parameters. Options 1, 2, and 4 may or may not help discriminate the origin of pain. Pain of pleuropulmonary origin usually worsens on inspiration.

A client is admitted to the emergency department after complaining of acute chest pain radiating down his left arm. Which laboratory studies would be indicated? a. Hemoglobin and hematocrit b. Serum glucose c. Creatinine phosphokinase (CPK) d. Troponin T and troponin I e. Myoglobin f. Blood urea nitrogen (BUN)

c. Creatinine phosphokinase (CPK) d. Troponin T and troponin I e. Myoglobin Levels of CPK, troponin T, and troponin I elevate because of cellular damage. Myoglobin elevation is an early indicator of myocardial damage. Hemoglobin, hematocrit, serum glucose, and BUN levels don't provide information related to myocardial ischemia.

The nurse is awaiting the arrival of a client from the emergency department. The client has a left ventricular myocardial infarction and is being admitted. In caring for this client, the nurse should be alert for which signs and symptoms of left-sided heart failure? a. Jugular vein distention b. Hepatomegaly c. Dyspnea d. Crackles e. Tachycardia f. Right upper quadrant pain

c. Dyspnea d. Crackles e. Tachycardia Signs and symptoms of left-sided heart failure include dyspnea, orthopnea, and paroxysmal nocturnal dyspnea; fatigue; nonproductive cough and crackles; hemoptysis; point of maximal impulse displaced toward the left anterior axillary line; tachycardia and S3 and S4 heart sounds; and cool, pale skin. Jugular vein distention, hepatomegaly, and right upper quadrant pain are all signs of right-sided heart failure.

A client comes to the emergency department complaining of chest pain. An electrocardiogram (ECG) reveals myocardial ischemia and an anterior-wall myocardial infarction (MI). Which ECG characteristic indicates myocardial ischemia? a. Prolonged PR interval b. Absent Q wave c. Elevated ST segment d. Widened QRS complex

c. Elevated ST segment Ischemic myocardial tissue changes cause elevation of the ST segment, a peaked or inverted T wave, and a pathological Q wave. A prolonged PR interval occurs with first-degree heart block, the least dangerous atrioventricular heart block; this disorder may arise in healthy people but sometimes results from drug toxicity, electrolyte or metabolic disturbances, rheumatic fever, or chronic degenerative disease of the conduction system. An absent Q wave is normal; an MI may cause a significant Q wave. A widened QRS complex indicates a conduction delay in the His-Purkinje system.

One hour after administering IV furosemide (Lasix) to a client with heart failure, a short burst of ventricular tachycardia appears on the cardiac monitor. Which of the following electrolyte imbalances should the nurse suspect? a. Hypocalcemia b. Hypermagnesemia c. Hypokalemia d. Hypernatremia

c. Furosemide is a potassium-depleting diuretic than can cause hypokalemia. In turn, hypokalemia increases myocardial excitability, leading to ventricular tachycardia.

A hospitalized client experiences digoxin- (Lanoxin-) induced premature ventricular contractions (PVCs). Which type of effect do such contractions represent? a. Toxic b. Secondary c. Iatrogenic d. Idiosyncratic

c. Iatrogenic Digoxin-induced PVCs are iatrogenic because the drug is mimicking a cardiac disorder. Because the client is experiencing an apparent pathological disorder, this effect isn't considered toxic, secondary, or idiosyncratic.

Which of the following nursing interventions is required to prepare a patient with cardiac dysrhythmia for an elective electrical cardioversion? a. Monitor blood pressure every 4 hours b. Administer digitalis and diuretics 24 hours before cardioversion c. Instruct the patient to restrict food and oral intake d. Facilitate CPR until the patient is prepared for cardioversion

c. Instruct the patient to restrict food and oral intake

The nurse would obtain serum levels of which electrolytes in a client with frequent episodes of ventricular tachycardia? a. Calcium and magnesium b. Potassium and calcium c. Magnesium and potassium d. Potassium and sodium

c. Magnesium and potassium Hypomagnesemia as well as hypokalemia and hyperkalemia are common causes of ventricular tachycardia. Calcium imbalances cause changes in the QT interval and ST segment. Alterations in sodium level don't cause rhythm disturbances.

Which of the following nursing interventions must a nurse perform when administering prescribed vasopressors to a patient with a cardiac dysrhythmia? a. Keep the patient flat for one hour after administration b. Administer every five minutes during cardiac resuscitation c. Monitor vital signs and cardiac rhythm d. Document heart rate before and after administration

c. Monitor vital signs and cardiac rhythm The nurse should monitor the patient's vital signs and cardiac rhythm for effectiveness of the medication and for side effects and should always have emergency life support equipment available when caring for an acutely ill patient. The side effects of vasopressor drugs are hypertension, dysrhythmias, pallor, and oliguria.

A 68-year-old woman is scheduled to undergo mitral valve replacement for severe mitral stenosis and mitral regurgitation. Although the diagnosis was made during childhood, she did not have any symptoms until 4 years ago. Recently, she noticed increased symptoms, despite daily doses of digoxin and furosemide. During the initial interview with the nice lady, the nurse would most likely learn that the client's childhood health history included: a. Chicken pox b. poliomyelitis c. Rheumatic fever d. meningitis

c. Most clients with mitral stenosis have a history of rheumatic fever or bacterial endocarditis.

During the previous few months, a 56-year-old woman felt brief twinges of chest pain while working in her garden and has had frequent episodes of indigestion. She comes to the hospital after experiencing severe anterior chest pain while raking leaves. Her evaluation confirms a diagnosis of stable angina pectoris. After stabilization and treatment, the client is discharged from the hospital. At her follow-up appointment, she is discouraged because she is experiencing pain with increasing frequency. She states that she is visiting an invalid friend twice a week and now cannot walk up the second flight of steps to the friend's apartment without pain. Which of the following measures that the nurse could suggest would most likely help the client deal with this problem? a. Visit her friend earlier in the day. b. Rest for at least an hour before climbing the stairs. c. Take a nitroglycerin tablet before climbing the stairs. d. Lie down once she reaches the friend's apartment.

c. Nitroglycerin may be used prophylactically before stressful physical activities such as stair climbing to help the client remain pain free. Visiting her friend early in the day would have no impact on decreasing pain episodes. Resting before or after an activity is not as likely to help prevent an activity-related pain episode.

As an initial step in treating a client with angina, the physician prescribes nitroglycerin tablets, 0.3mg given sublingually. This drug's principle effects are produced by: a. Antispasmotic effect on the pericardium b. Causing an increased mycocardial oxygen demand c. Vasodilation of peripheral vasculature d. Improved conductivity in the myocardium

c. Nitroglycerin produces peripheral vasodilation, which reduces myocardial oxygen consumption and demand. Vasodilation in coronary arteries and collateral vessels may also increase blood flow to the ischemic areas of the heart. Nitroglycerin decreases myocardial oxygen demand. Nitroglycerin does not have an effect on pericardial spasticity or conductivity in the myocardium.

The physician orders blood coagulation tests to evaluate a client's blood-clotting ability. The nurse knows that such tests are important in assessing clients at risk for thrombi, such as those with a history of atrial fibrillation, infective endocarditis, prosthetic heart valves, or myocardial infarction. Which test is used to determine a client's response to oral anticoagulant drugs? a. Bleeding time b. Platelet count c. Prothrombin time (PT) d. Partial thromboplastin time (PTT)

c. Prothrombin time (PT) PT determines a client's response to oral anticoagulant therapy. This test measures the time required for a fibrin clot to form in a citrated plasma sample after calcium ions and tissue thromboplastin are added and compares this time with the fibrin clotting time in a control sample. Anticoagulant dosages should be adjusted, as needed, to maintain PT at 1.5 to 2.5 times the control value. PTT determines the effectiveness of heparin therapy and helps evaluate bleeding tendencies. Roughly 99% of bleeding disorders are diagnosed from PT and PTT values. Bleeding time indicates how long it takes for a small puncture wound to stop bleeding. The platelet count reveals the number of circulating platelets in venous or arterial blood.

A client with mitral stenosis is scheduled for mitral valve replacement. Which condition may arise as a complication of mitral stenosis? a. Left-sided heart failure b. Myocardial ischemia c. Pulmonary hypertension d. Left ventricular hypertrophy

c. Pulmonary hypertension Mitral stenosis, or severe narrowing of the mitral valve, impedes blood flow through the stenotic valve, increasing pressure in the left atrium and pulmonary circulation. This may lead to low cardiac output, pulmonary hypertension, edema, and right-sided (not left-sided) heart failure. Other potential complications of mitral stenosis include mural thrombi, pulmonary hemorrhage, and embolism to vital organs. Myocardial ischemia may occur in a client with coronary artery disease. Left ventricular hypertrophy is a potential complication of aortic stenosis.

A client is experiencing tachycardia. The nurse's understanding of the physiological basis for this symptom is explained by which of the following statements? a. The demand for oxygen is decreased because of pleural involvement b. The inflammatory process causes the body to demand more oxygen to meet its needs. c. The heart has to pump faster to meet the demand for oxygen when there is lowered arterial oxygen tension. d. Respirations are labored.

c. The arterial oxygen supply is lowered and the demand for oxygen is increased, which results in the heart's having to beat faster to meet the body's needs for oxygen.

Sublingual nitroglycerin tablets begin to work within 1 to 2 minutes. How should the nurse instruct the client to use the drug when chest pain occurs? a. Take one tablet every 2 to 5 minutes until the pain stops. b. Take one tablet and rest for 10 minutes. Call the physician if pain persists after 10 minutes. c. Take one tablet, then an additional tablet every 5 minutes for a total of 3 tablets. Call the physician if pain persists after three tablets. d. Take one tablet. If pain persists after 5 minutes, take two tablets. If pain still persists 5 minutes later, call the physician.

c. The correct protocol for nitroglycerin used involves immediate administration, with subsequent doses taken at 5-minute intervals as needed, for a total dose of 3 tablets. Sublingual nitroglycerin appears in the blood stream within 2 to 3 minutes and is metabolized within about 10 minutes.

A client is at risk for pulmonary embolism and is on anticoagulant therapy with warfarin (Coumadin). The client's prothrombin time is 20 seconds, with a control of 11 seconds. The nurse assesses that this result is: a. The same as the client's own baseline level b. Lower than the needed therapeutic level c. Within the therapeutic range d. Higher than the therapeutic range

c. The therapeutic range for prothrombin time is 1.5 to 2 times the control for clients at risk for thrombus. Based on the client's control value, the therapeutic range for this individual would be 16.5 to 22 seconds. Therefore the result is within therapeutic range.

In order to prevent the development of tolerance, the nurse instructs the patient to: a. Apply the nitroglycerin patch every other day b. Switch to sublingual nitroglycerin when the patient's systolic blood pressure elevates to >140 mm Hg c. Apply the nitroglycerin patch for 14 hours each and remove for 10 hours at night d. Use the nitroglycerin patch for acute episodes of angina only

c. Tolerance can be prevented by maintaining an 8- to 12-hour nitrate-free period each day.

A client is admitted for treatment of Prinzmetal's angina. When developing the care plan, the nurse keeps in mind that this type of angina is triggered by: a. activities that increase myocardial oxygen demand. b. an unpredictable amount of activity. c. coronary artery spasm. d. the same type of activity that caused previous angina episodes.

c. coronary artery spasm. Prinzmetal's angina results from coronary artery spasm. Activities that increase myocardial oxygen demand may trigger angina of effort. An unpredictable amount of activity may precipitate unstable angina. Worsening angina is brought on by the same type or level of activity that caused previous angina episodes; however, anginal pain is increasingly severe.

A client with severe angina and ST-segment elevation on the electrocardiogram is being seen in the emergency department. In terms of diagnostic laboratory testing, it's most important for the nurse to advocate ordering a: a. creatine kinase level. b. hemoglobin (Hb) level. c. troponin level. d. liver panel.

c. troponin level. Troponin is a myocardial cell protein that is elevated in the serum when myocardial damage has occurred during a myocardial infarction (MI). It's the best serum indicator of MI and is more indicative of cardiac damage than creatine kinase. Hb values and liver panel components aren't as useful in the diagnosis of MI as a troponin level.

Treatment for sinus tachycardia

carotid massage ice

Angina

chest pain resulting from myocardial ischemia caused by inadequate myocardial blood/O2 supply - demand exceeds supply - conversion from aerobic to anaerobic respiration causes lactic acid build up which causes pain

Aspirin and MI

chew 325mg - antiplatelet

A client comes to the physician's office for a follow-up visit 4 weeks after suffering a myocardial infarction (MI). The nurse takes this opportunity to evaluate the client's knowledge of the prescribed cardiac rehabilitation program. Which evaluation statement suggests that the client needs more instruction? a. "Client performs relaxation exercises three times per day to reduce stress." b. "Client's 24-hour dietary recall reveals low intake of fat and cholesterol." c. "Client verbalizes an understanding of the need to seek emergency help if the heart rate increases markedly while at rest." d. "Client walks 4 miles in 1 hour every day."

d. "Client walks 4 miles in 1 hour every day." Four weeks after an MI, a client's walking program should aim for a goal of 2 miles in less than 1 hour. Walking 4 miles in 1 hour is excessive and may induce another MI by increasing the heart's oxygen demands. Therefore, this client requires appropriate exercise guidelines and precautions. The other options indicate understanding of the cardiac rehabilitation program. For example, the client should reduce stress, which speeds the heart rate and thus increases myocardial oxygen demands. Reducing dietary fat and cholesterol intake helps lower the risk of atherosclerosis. A sudden rise in the heart rate while at rest warrants emergency medical attention because it may signal a life-threatening arrhythmia and increase myocardial oxygen demands.

A paradoxical pulse occurs in a client who had a coronary artery bypass graft (CABG) surgery 2 days ago. Which of the following surgical complications should the nurse suspect? a. Left-sided heart failure b. Aortic regurgitation c. Complete heart block d. Pericardial tamponade

d. A paradoxical pulse (a palpable decrease in pulse amplitude on quiet inspiration) signals pericardial tamponade, a complication of CABG surgery. Left-sided heart failure can cause pulsus alternans (pulse amplitude alternation from beat to beat, with a regular rhythm). Aortic regurgitation may cause bisferious pulse (an increased arterial pulse with a double systolic peak). Complete heart block may cause a bounding pulse (a strong pulse with increased pulse pressure).

A nurse is assessing the blood pressure of a client diagnosed with primary hypertension. The nurse ensures accurate measurement by avoiding which of the following? a. Seating the client with arm bared, supported, and at heart level. b. Measuring the blood pressure after the client has been seated quietly for 5 minutes. c. Using a cuff with a rubber bladder that encircles at least 80% of the limb. d. Taking a blood pressure within 15 minutes after nicotine or caffeine ingestion.

d. BP should be taken with the client seated with the arm bared, positioned with support and at heart level. The client should sit with the legs on the floor, feet uncrossed, and not speak during the recording. The client should not have smoked tobacco or taken in caffeine in the 30 minutes preceding the measurement. The client should rest quietly for 5 minutes before the reading is taken. The cuff bladder should encircle at least 80% of the limb being measured. Gauges other than a mercury sphygnomanometer should be calibrated every 6 months to ensure accuracy.

Which of the following discharge instructions for self-care should the nurse provide to a patient who has undergone a percutaneous transluminal coronary angioplasty (PTCA) procedure? a. Refrain from sexual activity for one month. b. Cleanse the site with disinfectants and dress the wound appropriately. c. Normal activities of daily living can be resumed the first day post op. d. Cleanse the site with soap and water and remove any dressing.

d. Cleanse the site with soap and water and remove any dressing.

The nurse receives emergency laboratory results for a client with chest pain and immediately informs the physician. An increased myoglobin level suggests which of the following? a. Cancer b. Hypertension c. Liver disease d. Myocardial infarction

d. Detection of myoglobin is one diagnostic tool to determine whether myocardial damage has occurred. Myoglobin is generally detected about one hour after a heart attack is experienced and peaks within 4 to 6 hours after infarction (Remember, less than 90 mg/L is normal).

The nurse records a client's history and discovers several risk factors for coronary artery disease. Which cardiac risk factors are considered controllable? a. Diabetes, hypercholesterolemia, and heredity b. Diabetes, age, and gender c. Age, gender, and heredity d. Diabetes, hypercholesterolemia, and hypertension

d. Diabetes, hypercholesterolemia, and hypertension Controllable risk factors include hypertension, hypercholesterolemia, obesity, lack of exercise, smoking, diabetes, stress, alcohol abuse, and use of contraceptives. Uncontrollable risk factors for coronary artery disease include gender, age, and heredity.

Which of the following diagnostic tools is most commonly used to determine the location of myocardial damage? a. Cardiac catherization b. Cardiac enzymes c. Echocardiogram d. Electrocardiogram (ECG)

d. The ECG is the quickest, most accurate, and most widely used tool to determine the location of myocardial infarction. Cardiac enzymes are used to diagnose MI but can't determine the location. An echocardiogram is used most widely to view myocardial wall function after an MI has been diagnosed. Cardiac catherization is an invasive study for determining coronary artery disease and may also indicate the location of myocardial damage, but the study may not be performed immediately.

When assessing an ECG, the nurse knows that the P-R interval represents the time it takes for the: a. Impulse to begin atrial contraction b. Impulse to transverse the atria to the AV node c. SA node to discharge the impulse to begin atrial depolarization d. Impulse to travel to the ventricles

d. The P-R interval is measured on the ECG strip from the beginning of the P wave to the beginning of the QRS complex. It is the time it takes for the impulse to travel to the ventricle.

IV heparin therapy is ordered for a client. While implementing this order, a nurse ensures that which of the following medications is available on the nursing unit? a. Vitamin K b. Aminocaporic acid c. Potassium chloride d. Protamine sulfate

d. The antidote to heparin is protamine sulfate and should be readily available for use if excessive bleeding or hemorrhage should occur. Vitamin K is an antidote for warfarin.

The nurse prepares the client for insertion of a pulmonary artery catheter (Swan-Ganz catheter). The nurse teaches the client that the catheter will be inserted to provide information about: a. Stroke volume b. Cardiac output c. Venous pressure d. Left ventricular functioning

d. The catheter is placed in the pulmonary artery. Information regarding left ventricular function is obtained when the catheter balloon is inflated.

When preparing a client for discharge after surgery for a CABG, the nurse should teach the client that there will be: a. No further drainage from the incisions after hospitalizations b. A mild fever and extreme fatigue for several weeks after surgery c. Little incisional pain and tenderness after 3 to 4 weeks after surgery d. Some increase in edema in the leg used for the donor graft when activity increases

d. The client is up more at home, so dependent edema usually increases. Serosanguinous drainage may persist after discharge.

Which of the following types of pain is most characteristic of angina? a. Knifelike b. Sharp c. Shooting d. Tightness

d. The pain of angina usually ranges from a vague feeling of tightness to heavy, intense pain. Pain impulses originate in the most visceral muscles and may move to such areas as the chest, neck, and arms.

A client is scheduled for a cardiac catheterization using a radiopaque dye. Which of the following assessments is most critical before the procedure? a. Intake and output b. Baseline peripheral pulse rates c. Height and weight d. Allergy to iodine or shellfish

d. This procedure requires an informed consent because it involves injection of a radiopaque dye into the blood vessel. The risk of allergic reaction and possible anaphylaxis is serious and must be assessed before the procedure.

A 53-year-old client is about to undergo cardiac catheterization for which he signed an informed consent. As the nurse enters the room to administer sedation for the procedure, the client states, "I'm really worried about having this open heart surgery." Based on this statement, how should the nurse proceed? a. Medicate the client and document his comment. b. Medicate the client and notify the physician about the comment. c. Explain that cardiac catheterization doesn't involve open heart surgery, and then medicate the client. d. Withhold the medication and notify the physician immediately.

d. Withhold the medication and notify the physician immediately. The nurse should withhold the medication and notify the physician that the client doesn't understand the procedure. The physician then has the obligation to explain the procedure better to the client and determine whether or not the client understands. If the client doesn't understand, he can't give a true informed consent. If the medication is administered before the physician explains the procedure, the sedation may interfere with the client's ability to clearly understand the procedure. The nurse can't just medicate the client and document her finding; she must notify the physician.

Following coronary artery bypass grafting, a client begins having chest "fullness" and anxiety. The nurse suspects cardiac tamponade and prints a lead II electrocardiograph (ECG) strip for interpretation. In looking at the strip, the change in the QRS complex that would most support her suspicion is: a. narrowing complex. b. widening complex. c. amplitude increase. d. amplitude decrease.

d. amplitude decrease. Fluid surrounding the heart such as in cardiac tamponade, suppresses the amplitude of the QRS complexes on an ECG. Narrowing and widening complexes as well as an amplitude increase aren't what is expected on the ECG of an individual with cardiac tamponade.

A client is admitted to the acute care facility for treatment of heart failure. The nurse expects the physician to prescribe which drug? a. prednisone (Orasone) b. hydroxychloroquine sulfate (Plaquenil Sulfate) c. lidocaine (Xylocaine) d. furosemide (Lasix)

d. furosemide (Lasix) To maintain fluid balance — crucial for a client with heart failure — the physician typically prescribes a diuretic, such as furosemide; vasodilating agents; and drugs that increase contractility, such as digitalis glycosides. Prednisone, a corticosteroid, and hydroxychloroquine, an antimalarial agent, aren't indicated for heart failure. Lidocaine would be used only if the client also had ventricular ectopy.

A client with second-degree atrioventricular heart block is admitted to the coronary care unit. The nurse closely monitors the heart rate and rhythm. When interpreting the client's electrocardiogram (ECG) strip, the nurse knows that the QRS complex represents: a. atrial repolarization. b. ventricular repolarization. c. atrial depolarization. d. ventricular depolarization.

d. ventricular depolarization. The QRS complex on the ECG strip represents ventricular depolarization. Atrial repolarization usually occurs at the same time as ventricular depolarization and can't be distinguished on the ECG. The T wave represents ventricular repolarization. The P wave represents atrial depolarization.

The nurse closely monitors the fluid balance of a patient in congestive heart failure, with the knowledge that additional sodium and water retention occur in an already congested vascular system as a result of

decreased glomerular blood flow in the kidney

Nitrates (CAD/Angina)

dilate coronary arteries - decrease preload and afterload - side effects: headache, hypotension, flushing 1 sublingual q5min as needed - max 3 doses

Mitral valve insufficiency can lead to...

dilation and hypertrophy of left atrium and left ventricle - can cause Afib and CHF

Mitral valve insufficiency - signs and symptoms

dyspnea on exertion, orthopnea

Modifiable risk factors CAD

elevated lipid levels, hypertension, smoking, sedentary lifestyle, obesity, stress, diabetes, increased homocysteine levels

Myocardial injury is indicated on EKG by...

elevation or depression of ST segment, T wave inversion, or deep Q waves

Pericardial sac

encases the heart and protects from trauma and infection 2 layers - parietal (outer, tough/fibrous) and visceral (inner, thin)

Mitral valve prolapse - signs and symptoms

fatigue, atypical chest pain, palpitations, dizziness, tachycardia

Endocarditis - signs and symptoms

fever, anorexia, weight loss, fatigue, embolic complications, petechiae, splinter hemmorhages, Osler's nodes, Janeway lesions

The nurse monitors a patient receiving IV furosemide (Lasix) and enalapril (Vasotec) 5 mg po bid for an acute exacerbation of left sided congestive heart failure. The nurse determines that the treatment is effective upon finding

fewer crackles upon lung auscultation

Signs of Digoxin toxicity

green/yellow vision, bradycardia, nausea

While obtaining a nursing history from a 23-year-old man with rheumatic fever, the nurse recognizes that the most significant information relayed by the patient is that he

had an upper respiratory infection with a sore throat about 3 weeks ago

A patient with a blood pressure of 210/142 is admitted to the ICU with a diagnosis of hypertensive crisis. Clinical manifestations that the nurse would expect to find upon assessment include

headache, seizures, confusion, stupor, and coma

Ventricular tachycardia

heart rate > 100 bpm regular rhythm no p wave QRS > .12

When obtaining a health history from a patient with newly diagnosed coronary artery disease, the nurse recognizes that a modifiable major risk factor for coronary artery disease is present in the patient's history of

obesity

To assess for jugular vein distention in a patient with congestive heart failure, the nurse will:

observe the vertical distention of the viens as the patient is gradually elevated to an upright position

Stable angina

occurs with activities involving exertion or emotional stress - stable pattern of onset, duration, and intensity

A 78-year-old man is brought by his family to the emergency department because he developed severe weakness and difficulty breathing at home. During assessment of the patient's cardiovascular status, the nurse is aware that coronary artery disease in patients this age

often manifests without the usual chest pain and diaphoresis characteristic of myocardial ischemia

How is 3rd degree heart block treated?

pace maker

Cardiac tamponade

pericardial effusion - compresses heart restricting ventricular filling - cardiav output drops

Cardiac tamponade - treatment

pericardiocentesis - pericardial window for recurrent effusion

While a patient who survived a sudden cardiac death with a preceding MI is recovering in the hospital, the nurse would expect diagnostic evaluation and treatment to focus on

presence and prevention of coronary occlusion

Most cases of combined systolic and diastolic hypertension have no known cause and are therefore diagnosed as _____ hypertension.

primary

Sinus tachycardia

rate > 100 BPM regular rhythm P wave before each QRS PR interval .12 to .20 QRS < .12 usually related to SNS stimulation

Aortic stenosis

related to RHD - tissue thickens and narrows opening - left ventricular obstruction during systole leads to hypertrophy of ventricle

Mitral stenosis

related to RHD - tissue thickens and narrows the valve opening - valve cannot open fully - prevents blood flow from left atrium to left ventricle

Mitral valve prolapse

related to endocarditis and RHD - present in healthy individuals - can lead to mitral valve insufficiency - 5 times risk of bacterial endocarditis

The nurse administers IV nitroglycerin to a patient with a myocardial infarction. In evaluating the effect of this intervention, the nurse recognizes that an expected outcome of the administration of the drug is

relief of pain

QRS complex

represents ventricular depolarization/contraction - normal is < .12 seconds

T wave

represents ventricular repolarization/relaxation - should appear upright

Prinzmetal's angina

results from coronary artery spasm - may occur at rest - changes in ST segment on EKG - treat with calcium channel blockers

A patient with a myocardial infarction has rhythm disturbances involving the AV node. The nurse recognizes that the coronary artery most likely occluded is the

right coronary artery

A patient is diagnosed with pulmonary disease and elevated pulmonary vascular resistance. Which of the following heart failures is most likely to develop from this condition?

right heart failure

Multifocal PVCs

runs of PVCs - danger of decreased cardiac output - risk of V-tach or V-fib

Atrial flutter

saw tooth shaped flutter waves on EKG - artrial rate 250 - 300 - AV node selectively conducts impulse to ventricles - ventricular rate can be regular or variable

During early assessment of the patient with a myocardial infarction, the nurse is aware that the most important diagnostic test to determine the extent of an MI is

serial ECGs

After successful digitalization, a patient is to begin oral maintenance of digoxin (Lanoxin) and furosemide (Lasix) for control of congestive heart failure. To prevent digitalis toxicity, the nurse understands that it is most important to monitor the patient's

serum potassium

EKG and MI

shows either ST elevation, T wave inversion, or non-ST elevation - sometimes there is an abnormal q wave - ST and T waves return to normal within hours to days post MI - abnormal q waves are usually permanent

A 42-year-old businesswoman has hypertension for which she rarely takes her prescribed medication. She is socially active and usually has several cocktails each evening and smokes a pack of cigarettes daily. In teaching the patient about her risk factors, the nurse stresses that the factor that has been identified as the most powerful contributor to coronary artery disease in women her age is

smoking

The cardiac monitor alarms on a patient being monitored in the coronary care unit, and the nurse notes a cardiac pattern of undulations of varying contours and amplitude with no measurable ECG pattern. The patient is unconscious with no pulse or respirations. After calling for assistance, the nurse should

start basic CPR

During the physical examination of a 56 yr-old male, the nurse palpates the PMI in the sixth intercostal space lateral to the midclavicular line. The most approriate interpretation of this finding is:

the patient may have left ventricular hypertrophy

A patient with supraventricular tachycardia is to receive cardioversion. The nurse knows that when cardioversion is used

the patient should be sedated before the procedure, if possible

Your patient is taking Crestor (rovastatin). In explaining the risks and benefits of this medication, you would tell the patient:

this medication helps prevent atherosclerosis, but may cause severe muscle pain.

A diagnosis of acute coronary syndrome is the admission diagnosis for a patient transferred to the CCU. The nurse knows that this diagnosis indicates that the patient has experienced

unstable angina or a myocardial infarction

Lidocaine (cardiac)

ventricular arrhythmias, specifically PVCs

The nurse obtains a monitor strip on a patient admitted to the coronary care unit with a myocardial infarction and makes the following analysis: P wave not apparent; ventricular rate 142, R-R interval regular; PR interval not measurable; and QRS complex wide and distorted, >0.14 sec. The nurse interprets the patient's cardiac rhythm as

ventricular tachcardia

A 45 year old male is admitted to the ED after developing severe chest pain while doing yardwork. On admission, he has a normal EKG. The physician schedules the client for a cardiac catheretization with coronary angiography and possible percutaneous coronary intervention (PCI). The nurse prepares the client for the procedure by explaining that, in his case, it is used to:

visualize blockages in the coronary arteries and dilate these with a small balloon

During the hospitalization of a patient with a myocardial infarction, the nurse plans to start patient and family teaching about the disorder

while the patient is in the CCU

14. The nurse identifies the collaborative problem of potential complication: pulmonary edema for a patient in ADHF. When assessing the patient, the nurse will be most concerned about a. an apical pulse rate of 106 beats/min. b. an oxygen saturation of 88% on room air. c. weight gain of 1 kg (2.2 lb) over 24 hours. d. decreased hourly patient urinary output.

B Rationale: A decrease in oxygen saturation to less than 92% indicates hypoxemia. The nurse should administer supplemental oxygen immediately to the patient. An increase in apical pulse rate, 1-kg weight gain, and decreases in urine output also indicate worsening heart failure and require rapid nursing actions, but the low oxygen saturation rate requires the most immediate nursing action. Cognitive Level: Analysis Text Reference: pp. 829-830 Nursing Process: Assessment NCLEX: Physiological Integrity

when patient cardiac rehabilitation will begin.

For a patient who has been admitted the previous day to the coronary care unit with an AMI, the nurse will anticipate teaching the patient about _______________

A patient in ventricular fibrillation is successfully resuscitated and regains consciousness. She asks the nurse about what happened to her. The most appropriate response by the nurse is

"You had some a serious abnormal cardiac rythym and function, which treatment was able to reverse."

Small box on EKG =

0.04 seconds

Big box on EKG =

0.20 seconds

Normal Digoxin level

0.5 - 2

Review the preoperative assessment form for data about the pulses.

After repair of an abdominal aortic aneurysm, the nurse notes that the patient does not have popliteal, posterior tibial, or dorsalis pedis pulses. The legs are cool and mottled. Which action is appropriate for the nurse to take first?

22. A patient who is receiving dobutamine (Dobutrex) for the treatment of ADHF has all of the following nursing actions included in the plan of care. Which action will be best for the RN to delegate to an experienced LPN/LVN? a. Teach the patient the reasons for remaining on bed rest. b. Monitor the patient's BP every hour. c. Adjust the drip rate to keep the systolic BP >90 mm Hg. d. Call the health care provider about a decrease in urine output.

B Rationale: An experienced LPN/LVN would be able to monitor BP and would know to report significant changes to the RN. Teaching patients and making adjustments to the drip rate for vasoactive medications are RN-level skills. Because the health care provider may order changes in therapy based on the decrease in urine output, the RN should call the health care provider about the decreased urine output. Cognitive Level: Application Text Reference: pp. 827-829 Nursing Process: Planning NCLEX: Safe and Effective Care Environment

21. During postoperative teaching with a patient who had a mitral valve replacement with a mechanical valve, the nurse instructs the patient regarding a. the need to avoid high-voltage electrical fields. b. how to monitor anticoagulation therapy. c. the need for valve replacement in 7 to 10 years. d. how to check the radial pulse.

B Rationale: Anticoagulation with warfarin (Coumadin) is needed for a patient with mechanical valves to prevent clotting on the valve. There is no need to avoid high-voltage electrical fields. Mechanical valves are durable and would last longer than 7 to 10 years. Monitoring of radial pulse is not necessary after valve replacement. Cognitive Level: Comprehension Text Reference: p. 882 Nursing Process: Implementation NCLEX: Health Promotion and Maintenance

16. Which assessment information obtained by the nurse for a patient with aortic stenosis would be most important to report to the health care provider? a. A loud systolic murmur is audible along the right sternal border. b. The patient complains of chest pain associated with ambulation. c. The point of maximum impulse (PMI) is at the left midclavicular line. d. A thrill is palpable at the 2nd intercostal space, right sternal border.

B Rationale: Chest pain occurring with aortic stenosis is caused by cardiac ischemia, and reporting this information would be a priority. A systolic murmur and thrill would not be unusual for a patient with aortic stenosis. A PMI at the left midclavicular line is normal. Cognitive Level: Application Text Reference: pp. 879, 880 Nursing Process: Assessment NCLEX: Physiological Integrity

24. Which information obtained by the nurse when assessing a patient admitted with mitral valve stenosis should be communicated to the health care provider immediately? a. The patient has a loud diastolic murmur all across the precordium. b. The patient has crackles audible to the lung apices. c. The patient has a palpable thrill felt over the left anterior chest. d. The patient has 4+ peripheral edema in both legs.

B Rationale: Crackles that are audible throughout the lungs indicate that the patient is experiencing severe left ventricular failure and needs immediate interventions such as diuretics. A diastolic murmur and palpable thrill would be expected in a patient with mitral stenosis. Although 4+ peripheral edema indicates a need for a change in therapy, it does not need to be addressed urgently. Cognitive Level: Application Text Reference: p. 884 Nursing Process: Assessment NCLEX: Physiological Integrity

3. A patient has a 5-cm thoracic aortic aneurysm that was discovered during a routine chest x-ray. When obtaining a nursing history from the patient, the nurse will ask the patient about a. abdominal tenderness. b. difficulty swallowing. c. changes in bowel habits. d. dizziness or weakness.

B Rationale: Difficulty swallowing may occur with a thoracic aneurysm because of pressure on the esophagus. Abdominal tenderness or changes in bowel habits are consistent with an abdominal aneurysm. Dizziness or weakness may occur if there is blood loss from the aneurysm, but this aneurysm was discovered accidentally, not because the patient was symptomatic. Cognitive Level: Application Text Reference: p. 895 Nursing Process: Assessment NCLEX: Physiological Integrity

5. When the nurse is developing a teaching plan to prevent the development of heart failure in a patient with stage 1 hypertension, the information that is most likely to improve compliance with antihypertensive therapy is that a. hypertensive crisis may lead to development of acute heart failure in some patients. b. hypertension eventually will lead to heart failure by overworking the heart muscle. c. high BP increases risk for rheumatic heart disease. d. high systemic pressure precipitates papillary muscle rupture.

B Rationale: Hypertension is a primary cause of heart failure because the increase in ventricular afterload leads to ventricular hypertrophy and dilation. Hypertensive crisis may precipitate acute heart failure is some patients, but this patient with stage 1 hypertension may not be concerned about a crisis that happens only to some patients. Hypertension does not directly cause rheumatic heart disease (which is precipitated by infection with group A -hemolytic streptococcus) or papillary muscle rupture (which is caused by myocardial infarction/necrosis of the papillary muscle). Cognitive Level: Application Text Reference: p. 822 Nursing Process: Planning NCLEX: Health Promotion and Maintenance

7. A nursing action that is indicated for the collaborative problem of potential complication: cardiac dysrhythmia in a patient who has had a repair of a descending thoracic aortic aneurysm is to a. assess level of consciousness and orientation hourly. b. titrate oxygen to keep O2 saturation greater than 90%. c. turn the patient every 1 to 2 hours while on bed rest. d. monitor hourly fluid intake and urine output levels.

B Rationale: Hypoxemia may precipitate dysrhythmias in patients after aneurysm repair. Neurologic assessments, turning the patient, and monitoring intake and output are also appropriate nursing actions after aneurysm repair but will not have an effect on dysrhythmias. Cognitive Level: Application Text Reference: p. 897 Nursing Process: Implementation NCLEX: Physiological Integrity

8. A patient admitted to the hospital with an exacerbation of chronic heart failure tells the nurse, "I felt fine when I went to bed, but I woke up in the middle of the night feeling like I was suffocating!" The nurse can best document this assessment information as a. pulsus alternans. b. paroxysmal nocturnal dyspnea. c. two-pillow orthopnea. d. acute bilateral pleural effusion.

B Rationale: Paroxysmal nocturnal dyspnea is caused by the reabsorption of fluid from dependent body areas when the patient is sleeping and is characterized by waking up suddenly with the feeling of suffocation. Pulsus alternans is the alternation of strong and weak peripheral pulses during palpation. Orthopnea indicates that the patient is unable to lie flat because of dyspnea. Pleural effusions develop over a longer time period. Cognitive Level: Comprehension Text Reference: p. 825 Nursing Process: Assessment NCLEX: Physiological Integrity

16. After teaching a patient with newly diagnosed Raynaud's phenomenon about how to manage the condition, which behavior by the patient indicates that the teaching has been effective? a. The patient places the hands in hot water when they turn pale. b. The patient exercises indoors during the winter months. c. The patient takes pseudoephedrine (Sudafed) for cold symptoms. d. The patient avoids the use of aspirin and the NSAIDs.

B Rationale: Patients should avoid temperature extremes by exercising indoors when it is cold. To avoid burn injuries, the patient should use warm, rather than hot, water to warm the hands. Pseudoephedrine is a vasoconstrictor and should be avoided. There is no reason to avoid taking aspirin and NSAIDs with Raynaud's phenomenon. Cognitive Level: Application Text Reference: p. 909 Nursing Process: Implementation NCLEX: Health Promotion and Maintenance

11. When developing a plan to decrease preload in the patient with heart failure, the nurse will include actions such as a. administering sedatives to promote rest and decrease myocardial oxygen demand. b. positioning the patient in a high-Fowler's position with the feet horizontal in the bed. c. administering oxygen per mask or nasal cannula. d. encouraging leg exercises to improve venous return.

B Rationale: Positioning the patient in a high-Fowler's position with the legs dependent will reduce preload by decreasing venous return to the right atrium. The other interventions may also be appropriate for patients with heart failure but will not help in decreasing preload. Cognitive Level: Application Text Reference: pp. 827-828 Nursing Process: Planning NCLEX: Physiological Integrity

Atrial fibrillation

atria quiver at a rate of 300 - 600 BPM - irregular ventricular response - risk of mural thrombi

Mitral stenosis - signs and symptoms

dyspnea on exertion, orthopnea, paroxsymal nocturnal dyspnea, pulmonary edema/hemoptysis, JVD, peripheral edema, right ventricular failure and associated signs and symptoms

vegetative embolization.

A patient hospitalized with IE develops sharp left flank pain and hematuria. The nurse notifies the health care provider, recognizing that these symptoms may indicate ______________

Most new AEDs:

Use waveforms that are more effective at lower energy settings

Which of the following is most indicative of right ventricular infarct?

Peripheral edema

PVCs are treated with

IV Lidocaine

Treatment for a patient with left ventricular failure includes medications to:

Reduce afterload

Automaticity

The ability of cardiac cells to initiate impulse spontaneously and repetitively without external neurohormonal control.

Attach the cardiac monitor.

While admitting a patient with an AMI, which action should the nurse carry out first?

Amioderone

antidysrhytmic

Which of the following is also termed preinfarction angina? a. Stable angina b. Variant angina c. Unstable angina d. Silent ischemia

c. Unstable angina

Aortic insufficiency - signs and symptoms

dyspnea on exertion, orthopnea, palpitations, angina

During assessment of a patient with chest pain, the nurse recognizes that chest pain associated with unstable angina is

often unpredictable and may occur even at rest

SHORT ANSWER 1. When assessing a patient with possible PAD, the nurse obtains a brachial BP of 140/80 and an ankle pressure of 110/70. The nurse calculates the patient's ankle-brachial index (ABI) as ______.

0.78 or 0.79 Rationale: The ABI is calculated by dividing the ankle systolic BP by the brachial systolic BP. Cognitive Level: Application Text Reference: p. 901 Nursing Process: Implementation NCLEX: Physiological Integrity

A client is taking clonidine (Catapres) for treatment of hypertension. The nurse should teach the client about which of the following common adverse effects of this drug? Select all that apply. 1. Dry mouth. 2. Hyperkalemia. 3. Impotence. 4. Pancreatitis. 5. Sleep disturbance.

1, 3, 5. Clonidine (Catapres) is a central-acting adrenergic antagonist. It reduces sympathetic outflow from the central nervous system. Dry mouth, impotence, and sleep disturbances are possible adverse effects. Hyperkalemia and pancreatitis are not anticipated with use of this drug.

An industrial health nurse at a large printing plant finds a male employee's blood pressure to be elevated on two occasions 1 month apart and refers him to his private physician. The employee is about 25 lb overweight and has smoked a pack of cigarettes daily for more than 20 years. The client's physician prescribes atenolol (Tenormin) for the hypertension. The nurse should instruct the client to: 1. Avoid sudden discontinuation of the drug. 2. Monitor the blood pressure annually. 3. Follow a 2-g sodium diet. 4. Discontinue the medication if severe headaches develop.

1. Atenolol is a beta-adrenergic antagonist indicated for management of hypertension. Sudden discontinuation of this drug is dangerous because it may exacerbate symptoms. The medication should not be discontinued without a physician's order. Blood pressure needs to be monitored more frequently than annually in a client who is newly diagnosed and treated for hypertension. Clients are not usually placed on a 2-g sodium diet for hypertension.

The nurse is developing a care plan with an older adult and is instructing the client that hypertension can be a silent killer. The nurse should instruct the client to be aware of signs and symptoms of other system failures and encourage the client to report signs of which of the following diseases that are often a result of undeteced high blood pressure? 1. Cerebrovascular accidents (CVAs). 2. Liver disease. 3. Myocardial infarction. 4. Pulmonary disease.

1. Hypertension is referred to as the silent killer for adults, because until the adult has significant damage to other systems, the hypertension may go undetected. CVAs can be related to long-term hypertension. Liver or pulmonary disease is not generally associated with hypertension. Myocardial infarction is generally related to coronary artery disease.

stimulate a heart beat if the patient's own heart rate drops too low.

17. A patient has a permanent pacemaker inserted for treatment of chronic atrial fibrillation with slow ventricular response. The nurse teaches the patient that the pacemaker will ____________

45. A conscious and alert patient with chest pressure and a room air oxygen saturation of 98% can be given oxygen at: a. 6 to 8 L/min via face mask b. 1 to 2 L/min via nasal cannula c. 4 to 6 L/min via nasal cannula d. 12 to 15 L/min via nonrebreathing mask

12 to 15 L/min via nonrebreathing mask

24. Which of the following blood pressure reading is MOST suggestive of a patient who has arteriosclerosis? a. 140/90 mm Hg b. 150/80 mm Hg c. 160/70 mm Hg d. 180/100 mm Hg

180/100 mm Hg

Premature ventricular contraction

wide and distorted QRS complex on EKG - irritable focus in ventricle - result of increased automaticity of ventricular muscle cells - warning sign for sick heart

The initial pediatric defibrillation should occur at:

2 J/kg

The appropriate initial dose of morphine sulfate is:

2 to 4 mg

In teaching the hypertensive client to avoid orthostatic hypotension, the nurse should emphasize which of the following instructions? Select all that apply. 1. Plan regular times for taking medications. 2. Arise slowly from bed. 3. Avoid standing still for long periods. 4. Avoid excessive alcohol intake. 5. Avoid hot baths.

2, 3. Changing positions slowly and avoiding long periods of standing may limit the occurrence of orthostatic hypotension. Scheduling regular medication times is important for blood pressure management but this aspect is not related to the development of orthostatic hypotension. Excessive alcohol intake and hot baths are associated with vasodilation.

A client with hypertensive emergency is being treated with sodium nitroprusside (Nipride). In a dilution of 50 mg/ 250 mL, how many micrograms of Nipride are in each milliliter? ________________________ mcg.

200 mcg First, calculate the number of milligrams per milliliter: Next, calculate the number of micrograms in each milligram: CN: Pharmacological and parenteral

40. Which of the following patients would MOST likely present with atypical signs and symptoms of an acute myocardial infarction? a. 49 year old obese man b. 58 year old diabetic woman c. 60 year old man with anxiety d. 71 year old woman with hypertension

58 year old diabetic woman

Essential hypertension would be diagnosed in a 40-year-old male whose blood pressure readings were consistently at or above which of the following? 1. 120/ 90 mm Hg. 2. 130/ 85 mm Hg. 3. 140/ 90 mm Hg. 4. 160/ 80 mm Hg.

3. American Heart Association standards define hypertension as a consistent systolic blood pressure level greater than 140 mm Hg and a consistent diastolic blood pressure level greater than 90 mm Hg.

The client realizes the importance of quitting smoking, and the nurse develops a plan to help the client achieve this goal. Which of the following nursing interventions should be the initial step in this plan? 1. Review the negative effects of smoking on the body. 2. Discuss the effects of passive smoking on environmental pollution. 3. Establish the client's daily smoking pattern. 4. Explain how smoking worsens high blood pressure.

3. A plan to reduce or stop smoking begins with establishing the client's personal daily smoking pattern and activities associated with smoking. It is important that the client understands the associated health and environmental risks, but this knowledge has not been shown to help clients change their smoking behavior.

Second and subsequent defibrillations for pediatric patients should occur at:

4 J/kg

Which of the following terms is used to describe the amount of stretch on the myocardium at the end of diastole? a. Afterload b. Cardiac index c. Cardiac output d. Preload

4. Preload is the amount of stretch of the cardiac muscle fibers at the end of diastole. The volume of blood in the ventricle at the end of diastole determines the preload. Afterload is the force against which the ventricle must expel blood. Cardiac index is the individualized measurement of cardiac output, based on the client's body surface area. Cardiac output is the amount of blood the heart is expelling per minute.

A patient has a sinus arrest with a junctional escape rhythm. The nurse would expect the patient to have a pulse rate of

<40bpm

compression stockings should be applied before getting out of bed.

A 42-year-old service-counter worker undergoes sclerotherapy for treatment of superficial varicose veins at an outpatient center. Before discharging the patient, the nurse teaches the patient that ___________

candidacy for heart transplant depends on many factors.

A 55-year-old patient with inoperable coronary artery disease and end-stage heart failure asks the nurse whether heart transplant is a possible therapy. The nurse's response to the patient will be based on the knowledge that ____________

atherosclerotic plaques damage the artery and may lead to aneurysms.

A 69-year old patient is admitted to the hospital for elective repair of an abdominal aortic aneurysm. The history includes hypertension for 25 years, hyperlipidemia for 15 years, and smoking for 50 years. The patient asks the nurse what caused the aneurysm. The nurse's best response includes the information that ______________

41. The presence of dizziness in a patient with a suspected myocardial infarction is MOST likely the result of: a. Fear and anxiety b. The effects of nitroglycerin c. Acute left side heart failure d. A reduction in cardiac output

A reduction in cardiac output

38. Levine sign is MOST accurately defined as: a. Pushing on the sternum with the fingertips b. Rubbing the arm to which pain is radiating c. A subconsciously clinched fist over the chest d. A state of denial in patients with an acute MI

A subconsciously clinched fist over the chest

During a visit to a 72-year-old with chronic heart failure, the home care nurse finds that the patient has ankle edema, a 2-kg weight gain, and complains of "feeling too tired to do anything." Based on these data, the best nursing diagnosis for the patient is A) a. activity intolerance related to fatigue. B) b. disturbed body image related to leg swelling. C) c. impaired skin integrity related to peripheral edema. D) d. impaired gas exchange related to chronic heart failure.

A) a. activity intolerance related to fatigue.

A patient's family history reveals that the patient may be at risk for malignant hyperthermia (MH) during anesthesia. The nurse explains to the patient that A) a. anesthesia can be administered with minimal risks with the use of appropriate precautions and medications. B) b. as long as succinylcholine (Anectine) is not administered as a muscle relaxant, the reaction should not occur. C) c. surgery must be performed under local anesthetic to prevent development of a sudden, extreme increase in body temperature. D) d. surgery will be delayed until the patient is genetically tested to determine whether he or she is susceptible to malignant hyperthermia.

A) a. anesthesia can be administered with minimal risks with the use of appropriate precautions and medications.

Following an acute myocardial infarction, a previously healthy 67-year-old develops clinical manifestations of heart failure. The nurse anticipates discharge teaching will include information about A) a. angiotensin-converting enzyme (ACE) inhibitors. B) b. digitalis preparations. C) c. b-adrenergic agonists. D) d. calcium channel blockers.

A) a. angiotensin-converting enzyme (ACE) inhibitors.

Any patient guilt about having a therapeutic abortion may be identified when the nurse assesses the functional health pattern of A) a. value-belief. B) b. cognitive-perceptual. C) c. sexuality-reproductive. D) d. coping-stress tolerance.

A) a. value-belief.

31. Which of the following clinical findings is LEAST suggestive of a peripheral vascular disorder? a. A bruit heard over the carotid artery b. Pain in the calf muscle while walking c. Swelling and pain along the course of a vein d. An S3 sound during auscultation of the heart

An S3 sound during auscultation of the heart

9) How does a nurse assess for dysrhythmias? A. 12 lead EKG B. Listen to lung sounds C. blood test D. Urine sample

A. 12 lead EKG

A patient is diagnosed with hypertension and nadolol (Corgard) is prescribed. The nurse should consult with the health care provider before giving this medication upon finding a history of A) a. asthma. B) b. peptic ulcer disease. C) c. alcohol dependency. D) d. myocardial infarction (MI).

A. asthma.

Which of the following is true of abdominal aortic aneurysm (AAA)?

AAA may be asymptomatic as long as it is stable

The nurse obtains this information from a patient with prehypertension. Which finding is most important to address with the patient? A) a. Low dietary fiber intake B) b. No regular aerobic exercise C) c. Weight 5 pounds above ideal weight D) d. Drinks wine with dinner once a week

B No regular aerobic exercise

B-type natriuretic peptide (BNP).

An elderly patient with a 40-pack-year history of smoking and a recent myocardial infarction is admitted to the medical unit with acute shortness of breath; the nurse need to rule out pneumonia versus heart failure. The diagnostic test that the nurse will monitor to help in determining whether the patient has heart failure is ______________

A patient in left ventricular failure is expected to have:

Bradycardia

A patient asks you about his risk of cardiovascular disease. He is 50-years old and has diabetes, is overweight and smokes cigarettes. You advise him that:

He can modify his risk for cardiovascular disease by losing weight and not smoking

Pericarditis - treatment

Analgesics, antibiotics if bacterial, steroids, high fowler's leaning forward, monitor for signs of cardiac tamponade

application of external compression to the lower leg.

In planning care for a patient with a venous stasis ulcer on the right lower leg, the nurse understands that the most important intervention in promoting healing of the ulcer is

Dissections of the aorta are typically found:

In the ascending aorta

Risk factors for MI

Aterosclerosis, CAD, elevated cholesterol, smoking, hypertension, obesity, physical inactivity, impaired glucose tolerance, stress

Aneurysms are most commonly the result of:

Atherosclerotic disease

8. The MOST effective drug for the treatment of non-vagal-induced bradycardia is: a. Atropine b. Dopamine c. Epinephrine d. Metoprolol

Atropine

Treatment for sinus bradycardia

Atropine, transcutaneous or transvenous pacer

A patient with ST segment elevation in several electrocardiographic (ECG) leads is admitted to the emergency department (ED) and diagnosed as having an ST-segment-elevation myocardial infarction (STEMI). Which question should the nurse ask to determine whether the patient is a candidate for fibrinolytic therapy? A) a. "Do you take aspirin on a daily basis?" B) b. "What time did your chest pain begin?" C) c. "Is there any family history of heart disease?" D) d. "Can you describe the quality of your chest pain?"

B "What time did your chest pain begin?"

To determine the effects of therapy for a patient who is being treated for heart failure, which laboratory result will the nurse plan to review? A) a. Myoglobin B) b. Homocysteine (Hcy) C) c. Low-density lipoprotein (LDL) D) d. B-type natriuretic peptide (BNP)

D B-type natriuretic peptide (BNP)

Which information collected by the nurse who is admitting a patient with chest pain suggests that the pain is caused by an acute myocardial infarction (AMI)? A) a. The pain increases with deep breathing. B) b. The pain has persisted longer than 30 minutes. C) c. The pain worsens when the patient raises the arms. D) d. The pain is relieved after the patient takes nitroglycerin.

B The pain has persisted longer than 30 minutes.

When auscultating over the patient's abdominal aorta, the nurse hears a humming sound. The nurse documents this finding as a A) a. thrill. B) b. bruit. C) c. heave. D) d. murmur.

B bruit.

Amlodipine (Norvasc) is ordered for a patient with newly diagnosed Prinzmetal's (variant) angina. When teaching the patient, the nurse will include the information that amlodipine will A) a. reduce the "fight or flight" response. B) b. decrease spasm of the coronary arteries. C) c. increase the force of myocardial contraction. D) d. help prevent clotting in the coronary arteries.

B decrease spasm of the coronary arteries.

A 55-year-old with Stage D heart failure and type 2 diabetes asks the nurse whether heart transplant is a possible therapy. Which response by the nurse is appropriate? A) a. "Since you are diabetic, you would not be a candidate for a heart transplant." B) b. "The choice of a patient for a heart transplant depends on many different factors." C) c. "Your heart failure has not reached the stage in which heart transplants are considered." D) d. "People who have heart transplants are at risk for multiple complications after surgery."

B) b. "The choice of a patient for a heart transplant depends on many different factors."

. A patient who is scheduled for surgery in a week tells the nurse doing the preoperative assessment about an allergy to bananas, kiwifruit, and latex products. Which action is most important for the nurse to take? A) a. Notify the dietitian about the food allergies. B) b. Alert the surgery center about the latex allergy. C) c. Reassure the patient that all allergies are noted on the medical record. D) d. Ask whether the patient uses antihistamines to reduce allergic reactions.

B) b. Alert the surgery center about the latex allergy.

A patient has ST segment changes that indicate an acute inferior wall myocardial infarction. Which lead will be best for monitoring the patient? A) a. I B) b. II C) c. V6 D) d. MCL1

B) b. II

When a patient is transferred from the postanesthesia care unit (PACU) to the clinical surgical unit, the first action by the nurse on the surgical unit should be to A) a. assess the patient's pain. B) b. take the patient's vital signs. C) c. read the postoperative orders. D) d. check the rate of the IV infusion.

B) b. take the patient's vital signs.

While caring for a patient with abdominal surgery the first postoperative day, the nurse notices new bright-red drainage about 6 cm in diameter on the dressing. In response to this finding, the nurse should first A) a. reinforce the dressing. B) b. take the patient's vital signs. C) c. recheck the dressing in 1 hour for increased drainage. D) d. notify the patient's surgeon of a potential hemorrhage.

B) b. take the patient's vital signs.

5) Which of the following is not effective nursing management of heart failure? A. High Fowlers position B. Assisting with rigorous exercise 2x a day C. Daily weights, intake & output monitoring D. Continuous EKG monitoring

B. Assisting with rigorous exercise 2x a day

7) Which test is most important for the nurse to carry out if heart failure is suspected in a patient? A.12-lead EKG B. BNP C. ABG D. Exercise treadmill testing

B. BNP

27. When examining the chest of a patient who has an automated implanted cardioverter defibrillator (AICD), you would MOST likely find it: a. Just below the xiphoid process b. Below the left or right clavicle c. In the lower aspect of the chest d. Just lateral to the lower sternum

Below the left or right clavicle

Isoproterenol raises the heart rate by functioning as a:

Beta agonist

7. Epinephrine is used to treat patients in anaphylactic shock because of its effects of: a. Vasodilation and bronchoconstriction b. Bronchodilation and vasoconstriction c. Increased heart rate and automaticity d. Parasympathetic nervous system blockade

Bronchodilation and vasoconstriction

6. A patient hospitalized with IE develops sharp left flank pain and hematuria. The nurse notifies the health care provider, recognizing that these symptoms may indicate a. septicemia. b. acute pyelonephritis. c. vegetative embolization. d. glomerulonephritis.

C Rationale: The patient's clinical manifestations and history of IE indicate embolization. Sudden onset flank pain is not typical of pyelonephritis, septicemia, or glomerulonephritis. Cognitive Level: Application Text Reference: p. 869 Nursing Process: Assessment NCLEX: Physiological Integrity

After the nurse has finished teaching a patient about use of sublingual nitroglycerin (Nitrostat), which patient statement indicates that the teaching has been effective? A) a. "I can expect indigestion as a side effect of nitroglycerin." B) b. "I can only take the nitroglycerin if I start to have chest pain." C) c. "I will call an ambulance if I still have pain 5 minutes after taking the nitroglycerin." D) d. "I will help slow down the progress of the plaque formation by taking nitroglycerin."

C "I will call an ambulance if I still have pain 5 minutes after taking the nitroglycerin."

After giving a patient the initial dose of oral labetalol (Normodyne) for treatment of hypertension, which action should the nurse take? A) a. Encourage oral fluids to prevent dry mouth or dehydration. B) b. Instruct the patient to ask for help if heart palpitations occur. C) c. Ask the patient to request assistance when getting out of bed. D) d. Teach the patient that headaches may occur with this medication.

C Ask the patient to request assistance when getting out of bed.

The nurse working in the heart failure clinic will know that teaching for a 74-year-old patient with newly diagnosed heart failure has been effective when the patient A) a. uses an additional pillow to sleep when feeling short of breath at night. B) b. tells the home care nurse that furosemide (Lasix) is taken daily at bedtime. C) c. calls the clinic when the weight increases from 124 to 130 pounds in a week. D) d. says that the nitroglycerin patch will be used for any chest pain that develops.

C) c. calls the clinic when the weight increases from 124 to 130 pounds in a week.

During assessment of a 72-year-old with ankle swelling, the nurse notes jugular venous distention (JVD) with the head of the patient's bed elevated 45 degrees. The nurse knows this finding indicates A) a. decreased fluid volume. B) b. jugular vein atherosclerosis. C) c. elevated right atrial pressure. D) d. incompetent jugular vein valves.

C) c. elevated right atrial pressure.

After orienting a new staff member to the scrub nurse role, the nurse preceptor will know that the teaching was effective if the new staff member A) a. documents all patient care accurately. B) b. labels all specimens to send to the lab. C) c. keeps both hands above the operating table level. D) d. takes the patient to the postanesthesia recovery area.

C) c. keeps both hands above the operating table level.

23. All of the following medications are angiotensin-converting enzyme (ACE) inhibitors, EXCEPT: a. Calan b. Zestril c. Univasc d. Monopril

Calan

Clinical presentation of MI

Chest pain not relieved by nitroglycerine Pallor Diaphoresis Nausea and vomiting Dyspnea Impending doom Altered LOC

3. Injury to or disease of the ____ may cause prolapse of a cardiac valve leaflet, allowing blood to regurgitate from the ventricle into the atrium. a. Coronary sulcus b. Chordae tendineae c. Interatrial septum d. Coronary sinus

Chordae tendineae

17. Following an acute myocardial infarction, a previously healthy 67-year-old patient develops clinical manifestations of heart failure. The nurse anticipates discharge teaching will include information about a. digitalis preparations, such as digoxin (Lanoxin). b. calcium-channel blockers, such as diltiazem (Cardizem). c. -adrenergic agonists, such as dobutamine (Dobutrex). d. angiotensin-converting enzyme (ACE) inhibitors, such as captopril (Capoten).

D Rationale: ACE-inhibitor therapy is currently recommended to prevent the development of heart failure in patients who have had a myocardial infarction and as a first-line therapy for patients with chronic heart failure. Digoxin therapy for heart failure is no longer considered a first-line measure, and digoxin is added to the treatment protocol when therapy with other medications such as ACE-inhibitors, diuretics, and -adrenergic blockers is insufficient. Calcium-channel blockers are not generally used in the treatment of heart failure. The -adrenergic agonists such as dobutamine are administered through the IV route and are not used as initial therapy for heart failure. Cognitive Level: Application Text Reference: p. 832 Nursing Process: Implementation NCLEX: Physiological Integrity

3. During the assessment of a patient with IE, the nurse would expect to find a. substernal chest pain and pressure. b. splinter hemorrhages of the lips. c. dyspnea and a dry, hacking cough. d. a new regurgitant murmur.

D Rationale: New regurgitant murmurs occur in IE because vegetation on the valves prevents valve closure. Splinter hemorrhages occur on the nailbeds. Chest pain for pressure is not typical for the patient with IE and would be more consistent with angina or MI. Although dyspnea may occur as a result of heart failure, a moist cough would be expected rather than a dry, hacking cough. Cognitive Level: Comprehension Text Reference: p. 867 Nursing Process: Assessment NCLEX: Physiological Integrity

While doing the admission assessment for a thin 72-year-old patient, the nurse observes pulsation of the abdominal aorta in the epigastric area. Which action should the nurse take? A) a. Notify the hospital rapid response team. B) b. Instruct the patient to remain on bed rest. C) c. Teach the patient about aortic aneurysms. D) d. Document the finding in the patient chart.

D Document the finding in the patient chart.

A patient with a non-ST-segment-elevation myocardial infarction (NSTEMI) is receiving heparin. What is the purpose of the heparin? A) a. Platelet aggregation is enhanced by IV heparin infusion. B) b. Heparin will dissolve the clot that is blocking blood flow to the heart. C) c. Coronary artery plaque size and adherence are decreased with heparin. D) d. Heparin will prevent the development of new clots in the coronary arteries.

D Heparin will prevent the development of new clots in the coronary arteries.

Which information will the nurse include when teaching a patient who is scheduled to have a permanent pacemaker inserted for treatment of chronic atrial fibrillation with slow ventricular response? A) a. The pacemaker prevents or minimizes ventricular irritability. B) b. The pacemaker paces the atria at rates up to 500 impulses/minute. C) c. The pacemaker discharges if ventricular fibrillation and cardiac arrest occur. D) d. The pacemaker stimulates a heart beat if the patient's heart rate drops too low.

D) d. The pacemaker stimulates a heart beat if the patient's heart rate drops too low.

The nurse is caring for a patient who is receiving IV furosemide (Lasix) and morphine for the treatment of acute decompensated heart failure (ADHF) with severe orthopnea. When evaluating the patient response to the medications, the best indicator that the treatment has been effective is A) a. weight loss of 2 pounds overnight. B) b. hourly urine output greater than 60 mL. C) c. reduction in patient complaints of chest pain. D) d. decreased dyspnea with the head of bed at 30 degrees.

D) d. decreased dyspnea with the head of bed at 30 degrees.

A surgical patient received a volatile liquid as an inhalation anesthetic during surgery. Postoperatively the nurse should monitor the patient for A) a. tachypnea. B) b. myoclonia. C) c. hypertension. D) d. incisional pain.

D) d. incisional pain.

8) A nurse is assessing the client with left sided heart failure. The client states that he needs to use 3 pillows under the head and chest at night to be able to breathe comfortably while sleeping. The documents that the client is experience: A. ORTHOPNEA B. DYSPNEA at rest C. DYSPNEA on exertion D. Paroxysmal nocturnal dyspnea

D. Paroxysmal nocturnal dyspnea

Causes of PVCs

Digoxin toxicity, hypoxia, hypokalemia, fever, acidosis, exercise

Drugs used to teat valve disease

Digoxin, coumadin, amioderone, antibiotics

Treatment for A-Fib and A-Flutter

Digoxin, coumadin/pradaxa, calcium channel blockers/beta blockers, antiarrhythmics/amioderone

19. Spironolacton is a/an: a. Beta blocker b. Vasodilator c. Diuretic d. Antiarrhythmic

Diuretic

6. Which of the following drugs would be MOST effective when treating a patient with hypotension secondary to vasodilation? a. Dopamine b. Propranolol c. Isoproterenol d. Norepinephrine

Dopamine

A drug that may improve the symptoms of cardiogenic shock patients in the field is:

Dopamine

Cardiogenic shock is defined by shock symptoms after:

Hypovolemia and dysrhythmias have been corrected

A patient has received instruction on the management of her permanent pacemaker before discharge from the hospital. The nurse recognizes that teaching has been effective when the patient tells the nurse

I should take my pulse every day

The nurse hears a very faint murmur in the S1 heart sound in a patient with a stenosed mitral valve. The nurse records the murmur as a

I/VI systolic murmur

Endocarditis - treatment

IV antibiotics (maybe at home, PIC), rest, ROM, valve repair, anticoagulants

Which of the following interventions should be performed en route to the hospital during a lengthy transport of a patient with a suspected MI?

IV therapy and analgesia

Patients with pulseless ventricular tachycardia should be treated as though they have:

Ventricular fibrillation

The most common arrhythmia in sudden cardiac arrest is:

Ventricular fibrillation

Endocarditis - diagnostic tests

WBC/sed rate (elevated), blood cultures (+), TEE

Pericarditis - diagnostic tests

WBC/sed rate, blood culture, EKG, edho, BP, temp

20. Which of the following medications has a direct blood-thinning effect? a. Plavix b. Aspirin c. Accupril d. Warfarin

Warfarin

Jugular vein distention in cardiac patients should be evaluated with the patient positioned:

With the head elevated 45 degrees

Which signs cause the nurse to suspect cardiac tamponade after a client has cardiac surgery? Check all that apply. a. Tachycardia b. Hypertension c. Increased CVP d. Increased urine output e. Jugular vein distention

a, c, e. Blood in the pericardial sac compresses the heart so the ventricles cannot fill; this leads to a rapid thready pulse. Tamponade causes hypotension and a narrowed pulse pressure. As the tamponade increases, pressure on the heart interferes with the ejection of blood from the left ventricle, resulting in an increased pressure in the right side of the heart and the systemic circulation. As the heart because more inefficient, there is a decrease in kidney perfusion and therefore urine output. The increased venous pressure caused JVD.

The staff nurse is caring for a client who is a potential heart donor. The client's family is concerned that the recipient will have access to personal donor information. Which response by the nurse demonstrates knowledge of the organ donation process? a. "I will have the transplant coordinator speak with you to answer your questions." b. "There is never contact between the donor's family and the recipient." c. "The recipient is allowed to ask questions about the donor and have them answered." d. "It is important that the recipient know where to send Thank-You cards."

a. "I will have the transplant coordinator speak with you to answer your questions." The transplant coordinator, a specially trained person with knowledge of the donation, procurement, and transplantation process, typically speaks to family members and answers their questions. Contact is permitted after the procedure with consent from the donor's family and the recipient. Typically, the transplant organization coordinates the communication. Confidentiality of the potential donor is always maintained unless the recipient and donor families both sign confidentiality waivers.

Before discharge, which instruction should the nurse give to a client receiving flecainide (Tambocor) to reduce the risk of heart failure? a. "Limit your fluid intake." b. "Take a diuretic before going to bed." c. "Limit your potassium intake." d. "Have your serum electrolyte levels measured weekly."

a. "Limit your fluid intake." The nurse should tell the client receiving flecainide to limit fluid intake. The client shouldn't take a diuretic unless prescribed by the physician; if prescribed, the diuretic should be taken early in the day to prevent nocturia. Sodium (not potassium) should be limited because excessive sodium intake causes water retention. The client's electrolyte levels don't need to be measured weekly

After a myocardial infarction, a client develops a complication requiring a continuous infusion of lidocaine. To monitor the effectiveness of this infusion, the nurse should focus primarily on: a. electrocardiogram (ECG). b. urine output. c. creatine kinase (CK) and troponin levels. d. blood pressure and heart rate.

a. electrocardiogram (ECG). Lidocaine is an antiarrhythmic given to treat cardiac irritability and ventricular arrhythmias. The best indicator of its effectiveness is the reduction or disappearance of ventricular arrhythmias as seen on the ECG. Urine output is an indicator of pump effectiveness. CK and troponin levels monitor myocardial damage. Blood pressure and heart rate are too nonspecific to be indicators of lidocaine's effectiveness.

pace maker - sensing

ability to recognize spontaneous atrial or ventricular activity

Unmodifiable risk factors CAD

age, gender, race, family hx

The nurse on the telemetry unit is faced with various situations. Which situation takes priority? a. A client's cardiac monitor suddenly reveals sinus tachycardia with isolated premature ventricular contractions. b. A client's cardiac rhythm suddenly changes from normal sinus rhythm to uncontrolled atrial fibrillation. c. A client is requesting help to go to the bathroom. d. The cardiologist is asking the nurse to make rounds with him to his clients.

b. A client's cardiac rhythm suddenly changes from normal sinus rhythm to uncontrolled atrial fibrillation. The client whose cardiac rhythm suddenly changes from normal sinus rhythm to uncontrolled atrial fibrillation takes priority. This cardiac rhythm change may cause clots to shower from the atria placing the client at risk for a stroke. The client whose cardiac monitor reveals sinus tachycardia with isolated premature ventricular contractions isn't experiencing a life-threatening situation; therefore, he doesn't take priority. The nurse can ask her ancillary staff member to assist the client to the bathroom. Making rounds with the physician can wait until the nurse addresses the needs of the client in atrial fibrillation.

When do coronary arteries primarily receive blood flow? a. During inspiration b. During diastolic c. During expiration d. During systole

b. Although the coronary arteries may receive a minute portion of blood during systole, most of the blood flow to coronary arteries is supplied during diastole. Breathing patterns are irrelevant to blood flow.

When ventricular fibrillation occurs in a CCU, the first person reaching the client should: a. Administer oxygen b. Defibrillate the client c. Initiate CPR d. Administer sodium bicarbonate intravenously

b. Defibrillate the client Ventricular fibrillation is a death-producing dysrhythmia and, once identified, must be terminated immediately by precordial shock (defibrillation). This is usually a standing physician's order in a CCU.

The nurse is caring for a client experiencing dyspnea, dependent edema, hepatomegaly, crackles, and jugular vein distention. What condition should the nurse suspect? a. Pulmonary embolism b. Heart failure c. Cardiac tamponade d. Tension pneumothorax

b. Heart failure A client with heart failure has decreased cardiac output caused by the heart's decreased pumping ability. A buildup of fluid occurs, causing dyspnea, dependent edema, hepatomegaly, crackles, and jugular vein distention. A client with pulmonary embolism experiences acute shortness of breath, pleuritic chest pain, hemoptysis, and fever. A client with cardiac tamponade experiences muffled heart sounds, hypotension, and elevated central venous pressure. A client with tension pneumothorax has a deviated trachea and absent breath sounds on the affected side as well as dyspnea and jugular vein distention.

Which of the following ECG waveforms characterizes conduction of an electrical impulse through the left ventricle? a. P wave b. QRS complex c. PR interval d. QT interval

b. QRS complex

Considering a client's atrial fibrillation, the nurse must administer digoxin (Lanoxin) with caution because it: a. affects the sympathetic division of the autonomic nervous system, decreasing vagal tone. b. stimulates the parasympathetic division of the autonomic nervous system, increasing vagal tone. c. can induce hypertensive crisis by constricting arteries. d. can trigger proarrhythmia by increasing stroke volume.

b. stimulates the parasympathetic division of the autonomic nervous system, increasing vagal tone. The nurse must administer digoxin with caution in a client with atrial fibrillation because digoxin stimulates the parasympathetic division of the autonomic nervous system, increasing vagal tone. The vagal effect slows the heart rate, increases the refractory period, and slows conduction through the atrioventricular node and junctional tissue, thus increasing the potential for new arrhythmias to develop. Digoxin doesn't constrict arteries. Although it can trigger proarrhythmias, it does so by increasing vagal tone (not stroke volume).

Surgical treatments for valve disease

balloon valvuloplasty, aortic valve replacement, mitral valve replacement

Medications for sinus tachycardia

beta blockers - block beta receptors in heart and decrease heart rate - decrease force of contraction and decrease AV conduction

In order to be effective, percutaneous transluminal coronary angioplasty (PTCA) must be performed within what time frame, beginning with arrival at the emergency department after diagnosis of myocardial infarction? a. 6 to 12 months b. 9 days c. 60 minutes d. 30 minutes

c. 60 minutes

After cardiac surgery, a client's blood pressure measures 126/80. The nurse determines that the mean arterial pressure (MAP) is which of the following? a. 46 mm Hg b. 80 mm Hg c. 95 mm Hg d. 90 mm Hg

c. 95 mm Hg

When auscultating the apical pulse of a client who has atrial fibrillation, the nurse would expect to hear a rhythm that is characterized by: a. The presence of occasional coupled beats b. Long pauses in an otherwise regular rhythm c. A continuous and totally unpredictable irregularity d. Slow but strong and regular beats

c. A continuous and totally unpredictable irregularity In atrial fibrillation, multiple ectopic foci stimulate the atria to contract. The AV node is unable to transmit all of these impulses to the ventricles, resulting in a pattern of highly irregular ventricular contractions.

An 84-year-old male is returning from the operating room (OR) after inguinal hernia repair. The nurse notes that he has fluid volume excess from the operation and is at risk for left-sided heart failure. Which sign or symptom indicates left-sided heart failure? a. Jugular vein distention b. Right upper quadrant pain c. Bibasilar fine crackles d. Dependent edema

c. Bibasilar fine crackles Bibasilar fine crackles are a sign of alveolar fluid, a sequelae of left ventricular fluid, or pressure overload. Jugular vein distention, right upper quadrant pain (hepatomegaly), and dependent edema are caused by right-sided heart failure, usually a chronic condition.

The nurse is teaching a client who receives nitrates for the relief of chest pain. Which instruction should the nurse emphasize? a. Repeat the dose of sublingual nitroglycerin every 15 minutes for three doses. b. Store the drug in a cool, well-lit place. c. Lie down or sit in a chair for 5 to 10 minutes after taking the drug. d. Restrict alcohol intake to two drinks per day.

c. Lie down or sit in a chair for 5 to 10 minutes after taking the drug. Nitrates act primarily to relax coronary smooth muscle and produce vasodilation. They can cause hypotension, which makes the client dizzy and weak. Nitrates are taken at the first sign of chest pain and before activities that might induce chest pain. Sublingual nitroglycerin is taken every 5 minutes for three doses. If the pain persists, the client should seek medical assistance immediately. Nitrates must be stored in a dark place in a closed container. Sunlight causes the medication to lose its effectiveness. Alcohol is prohibited because nitrates may enhance the effects of the alcohol.

Balloon valvuloplasty

invasive non surgical procedure - catheter is placed in femoral vein to valve and balloon is inflated to enlarge orifice - monitor for bleeding from insertion site - monitor for signs of emboli - monitor cardiac rhythm, heart sounds, and cardiac output

A woman with severe mitral stenosis and mitral regurgitation has a pulmonary artery catheter inserted. The physician orders pulmonary artery pressure monitoring, including pulmonary capillary wedge pressures. The purpose of this is to help assess the: a. Degree of coronary artery stenosis b. Peripheral arterial pressure c. Pressure from fluid within the left ventricle d. Oxygen and carbon dioxide concentration is the blood

c. The pulmonary artery pressures are used to assess the heart's ability to receive and pump blood. The pulmonary capillary wedge pressure reflects the left ventricle end-diastolic pressure and guides the physician in determining fluid management for the client. The degree of coronary artery stenosis is assessed during a cardiac catherization. The peripheral arterial pressure is assessed with an arterial line.

The nurse is caring for a client with acute pulmonary edema. To immediately promote oxygenation and relieve dyspnea, the nurse should: a. administer oxygen. b. have the client take deep breaths and cough. c. place the client in high Fowler's position. d. perform chest physiotherapy.

c. place the client in high Fowler's position. The high Fowler's position will initially promote oxygenation in the client and relieve shortness of breath. Additional measures include administering oxygen to increase content in the blood. Deep breathing and coughing will improve oxygenation postoperatively but may not immediately relieve shortness of breath. Chest physiotherapy results in expectoration of secretions, which isn't the primary problem in pulmonary edema.

The nurse is caring for a patient with pericarditis, who has increasing complaints of chest pain. The nurse is aware that this may signify which of the following serious consequences of this diagnosis?

cardiac tamponade

How to calculate heart rate from EKG

count the number of R waves in a six second strip and multiply by 10

S2

created by closure of aortic and pulmonic valves

S1

created by the closure of the mitral and tricuspid valves - indicates beginning of ventricular systole

The monitor technician on the telemetry unit asks the charge nurse why every client whose monitor shows atrial fibrillation is receiving warfarin (Coumadin). Which response by the charge nurse is best? a. "It's just a coincidence; most clients with atrial fibrillation don't receive warfarin." b. "Warfarin controls heart rate in the client with atrial fibrillation." c. "Warfarin prevents atrial fibrillation from progressing to a lethal arrhythmia." d. "Warfarin prevents clot formation in the atria of clients with atrial fibrillation."

d. "Warfarin prevents clot formation in the atria of clients with atrial fibrillation." Blood pools in the atria of clients with atrial fibrillation. As the blood pools, clots form. These clots can be forced from the atria as the heart beats, placing the client at risk for stroke. Warfarin is prescribed in most clients with atrial fibrillation to prevent clot formation and decrease the risk of stroke, not to control heart rate. Digoxin is typically prescribed to control heart rate in atrial fibrillation. Atrial fibrillation doesn't typically progress to a lethal arrhythmia such as ventricular fibrillation.

How long after oral administration can the nurse expect to see digoxin's (Lanoxin) peak effect? a. 2 to 5 minutes b. 10 to 20 minutes c. 30 minutes to 2 hours d. 2 to 6 hours

d. 2 to 6 hours The peak effect of digoxin occurs 2 to 6 hours after an oral dose and 1 to 4 hours after an I.V. dose. Digoxin's onset of action ranges from 30 minutes to 2 hours after an oral dose and from 5 to 30 minutes after an I.V. dose.

Aortic stenosis - signs and symptoms

dyspnea on exertion, angina, syncope

The adaptations of a client with complete heart block would most likely include: a. Nausea and vertigo b. Flushing and slurred speech c. Cephalalgia and blurred vision d. Syncope and low ventricular rate

d. Syncope and low ventricular rate In complete atrioventricular block, the ventricles take over the pacemaker function in the heart but at a much slower rate than that of the SA node. As a result there is decreased cerebral circulation, causing syncope.

The nurse expects that a client with mitral stenosis would demonstrate symptoms associated with congestion in the: a. Aorta b. Right atrium c. Superior vena cava d. Pulmonary circulation

d. When mitral stenosis is present, the left atrium has difficulty emptying its contents into the left ventricle. Hence, because there is no valve to prevent backward flow into the pulmonary vein, the pulmonary circulation is under pressure.

A client with a bundle branch block is on a cardiac monitor. The nurse should expect to observe: a. Sagging ST segments b. Absence of P wave configurations c. Inverted T waves following each QRS complex d. Widening of QRS complexes to 0.12 second or greater.

d. Widening of QRS complexes to 0.12 second or greater. Bundle branch block interferes with the conduction of impulses from the AV node to the ventricle supplied by the affected bundle. Conduction through the ventricles is delayed, as evidenced by a widened QRS complex.

Normal sinus rhythm

heart rate 60 - 100 BPM regular rhythm P wave before each QRS PR interval .12 to .20 QRS < .12

Sinus bradycardia

rate < 60 BPM regular rhythm P wave before each QRS PR interval .12 to .20 QRS < .12

Atropine sulfate

used for bradycardia and first-degree AV blocks


Ensembles d'études connexes

Muscles that cross the Elbow Joint; Movements of Forearm

View Set

Seeley's Anatomy & Physiology Ch 24 - Digestive System

View Set

Macroeconomics Final Questions Possibilities

View Set

Nature of Contracts, Capacity, Consideration

View Set

Chapter 7 Intermediate Accounting : Review - Cash and Receivables

View Set